Dies ist ein Archiv der Seite Wikipedia:Auskunft. Es enthält alle Abschnitte, die in der Kalender-Woche 12 im Jahr 2012 begonnen wurden.

Möchtest du in einer neuen Diskussion auf dieses Archiv verweisen? Nutze folgenden Link und ersetze ________ durch den Namen des Abschnittes. Die Nummer im Inhaltsverzeichnis gehört nicht dazu:

[[Wikipedia:Auskunft/Archiv/2012/Woche 12#________]]


← vorherige Woche Gesamtarchiv nächste Woche →

Werkzeuge/Programme zur Glossarerstellung

Hallo. Ich möchte zu einem "Vereinszeitungs-Sammelband" eine Art Glossar erstellen. Dort sollen die wichtigsten Personen und Begriffe kurz erläutert werden. Ein Glossar halt :) Mir liegen die einzelnen Ausgaben der Zeitung digital vor. Um sie nicht alle einzeln nach Stichworten durchsuchen zu müssen, dachte ich mir, ich schicke sie einfach in die Cloud von Evernote. So kann ich einfach ein Stichwort eingeben und das Programm sagt mir in welchen Dokumenten dieses vorkommt. Einfacher gehts glaub ich nicht, oder doch?

Aber nun zur eigentlichen Frage: Ich würde parallel gern eine Liste mit den Stichwörtern und ihrer Häufigkeit erstellen. Dazu bräuchte man ja quasi so ein Programm oder ähnliches. Gibt es sowas auch als Freeware? Dies Excel-Programm scheint mir zu umständlich und gibt die Häufigkeit eines Stichworts nicht her. Diese Seite sortiert zwar fein, aber gibt auch keine Auskunft bezüglich der Stichwort-Häufigkeit. Kennt jemand eine gute Freeware-Lösung. Oder falls es keine Freeware gibt, eine kostengünstige Lösung? --BlueCücü (Diskussion) 09:23, 19. Mär. 2012 (CET)

TextStat, selber verwendet mit mehreren Millionen Wörtern. Works like a breeeeze! GEEZERnil nisi bene 10:18, 19. Mär. 2012 (CET)
Merci. --BlueCücü (Diskussion) 12:14, 19. Mär. 2012 (CET)
Anytime... Ergebnis lässt sich dann auch nach EXCEL exportieren... GEEZERnil nisi bene 12:25, 19. Mär. 2012 (CET)

Schriftart

Weiß jemand, wie die Schriftart heißt, die von der Computerzeitschrift Chip in ihrem Logo verwendet wird?--Uwe W. (Diskussion) 11:09, 19. Mär. 2012 (CET)

Es ist eine ge-render-te Version von Computer.ttf - also nach ähnlichen ttf-Namen suchen, wie z.B. ComputerFont Regular. Hier kann man mit Varianten davon herumspielen und sich Logos machen. GEEZERnil nisi bene 11:17, 19. Mär. 2012 (CET)
Danke für die schnelle Antwort.--Uwe W. (Diskussion) 11:39, 19. Mär. 2012 (CET)

Gibt es sowas wie den "National Security Letter" auch in Deutschland

In den USA dürfen die Geheimdienste Behörden, Firmen, Einzelpersonen und sogar Zeitungsverlage zwingen vor Gericht zu lügen (siehe en:National Security Letter und en:Gag order). Gibt es sowas auch in Deutschland? --109.193.145.79 10:07, 19. Mär. 2012 (CET)

ob es sowas wie eine Glaubhafte Abstreitbarkeit im Gesetz gibt ist mir nicht bekannt, vermutlich nicht. Da beruft man sich auf Geheimhaltung oder ähnliches; wie ein aktuelles Beispiel vermuten lässt....--gp (Diskussion) 13:27, 19. Mär. 2012 (CET)
"dürfen ... zwingen vor Gericht zu lügen" ist in en:National Security Letter und en:Gag order nicht zu finden und mit Sicherheit Unfug. Selbstverständlich gibt es das in Deutschland auch nicht. Beamte brauchen für Zeugenaussagen über dienstliche Vorgänge aber eine Genehmigung des Dienstherrn, s. z.B. ZPO § 376, StPO § 54, BBG § 68. --Vsop (Diskussion) 23:06, 19. Mär. 2012 (CET)

Notfalleinrichtungen...

Was ist konkret unter der Begrifflichkeit 'Notfalleinrichtung' zu verstehen? Danke und Grüße, --193.17.232.3 11:45, 19. Mär. 2012 (CET)

Da die Notfälle durchaus äußerst unterschiedlich sind, sind die entsprechend dafür oder dagegen vorgesehenen Einrichtungen das "in concreto" eben und notgedrungen auch. --G-Michel-Hürth (Diskussion) 12:38, 19. Mär. 2012 (CET)
Der Begriff ist weit gefasst, aber das sind alle Systeme und Vorrichtungen die im Falle eines Notfalles genutzt und eingesetzt werden können. Das reicht vom einfachen Hammer zum Zertrümmern der Scheibe im Omnibus bis zum Fluchttunnel in den Alpentunneln, im Flieger von den herunterfallenden Sauerstoffmasken bis zur Notrutsche in der Tür usw. Sofern Menschen dabei tätig werden müssen nennt man das aber eher Nothilfe.--212.23.103.40 14:42, 19. Mär. 2012 (CET)
Mal ein bisschen Theoriebildung betrieben würde ich folgendes sagen: Es lassen sich drei Arten von Notfalleinrichtung unterscheiden:
  1. Technische Einrichtungen: vom Warnlicht über die Sprinkleranlage bis hin zur Selbstabschaltung eines Kernkraftwerkes
  2. Organisatorische Einrichtungen: z. B. Notrufzentrale oder Krisenstab, auch Notfallpläne
  3. Einrichtungen der Nothilfe: z. B. Krankenhäuser und Notunterkünfte
Dabei gibt es natürlich auch Überschneidungen. Einen Krankenwagen etwa kann man gleichermaßen als Nothilfeeinrichtung wie auch als technische Einrichtung betrachten.
Ansonsten ist der Begriff Notfall in der Tat sehr weit gefasst. Ein technischer Defekt kann darunter fallen, ein Naturereignis wie ein Sturm, aber z. B. auch ein angekündigter Suizid oder ein drohender Krieg. --Duschgeldrache2 (Diskussion) 00:19, 20. Mär. 2012 (CET)

Krankheiten in Folge Taurinmangel bei Katzen

Hallo

Ich bin Tierpfleger mit jahrzehnte langer Erfahrung mit Katzen.

Kürzlich hatte ich, trotz „Premium-Futter“ einen DCM-Fall, bekanntlich, bei Katzen eine Folge von Taurinmangel. Nun gehe ich der Frage nach, ob die Taurinmengenangaben auf den Packungen u.U. nicht stimmen, oder ob generell von einem zu geringen Taurinbedarf der Katzen ausgegangen wird.

Wer kann mir Angaben zu Krankheiten in Folge Taurinmangel bei Katzen machen? Wo finde ich ausführliche Infos? Nebst DCM sind mir Niereninsuffizienz und Augenerkrankungen bekannt, vermute aber, dass es noch viel mehr sind, z.B. Stomatitis, Gingivitis, Zahnstein, Diabetes???

Vielen Dank im Voraus --Tierpfleger (Diskussion) 11:54, 19. Mär. 2012 (CET)

Taurinmangel ist nur eine Ursache bei DCM bei Katzen. Da gibt es noch eine ganze Reihe anderer Ursachen, die dafür auch infrage kommen. Also den behandelnden Tierarzt fragen und darauf hinweisen, dass genügend Taurin im Futter ist. Die Katze wäre sowieso gesünder, wenn sie Freigänger wäre und sich ihr natürliches Futter selbst fangen könnte, wenigstens mal hin und wieder.--212.23.103.40 14:29, 19. Mär. 2012 (CET)
Das sieht unser Abschnitt Dilatative Kardiomyopathie (Veterinärmedizin)#Katze zwar anders, deswegen ist der {{Gesundheitshinweis}} angebracht. --Rôtkæppchen68 14:38, 19. Mär. 2012 (CET)
Oh Gott ein gläubiger Wikileser. Da steht "meistens" und das ist auch im übertragenen Sinne bei 50% der Fälle schon richtig. Und ist es nicht ein bisschen absurd, bei Katzen einen menschlichen Gesundheitshinweis zu setzen. Wo bin ich hier hin geraten?--212.23.103.40 14:54, 19. Mär. 2012 (CET)
Der Hinweis ist nicht nur für Menschen. Klick doch einfach mal auf den Link und lies genau. --тнояsтеn 16:09, 19. Mär. 2012 (CET)
Sagt man das wirklich so: "Die dilatative Kardiomyopathie der Katze ist in den meisten Fällen nicht erblich, sondern eine Folge eines Taurinmangels. Sie wird seit der Einführung von kommerziell erhältlichem Katzenfutter nur noch sehr selten beobachtet und reagiert normalerweise gut auf die Gabe von Taurin." Die Myopathie reagiert gut ? Die Katze reagiert gut, oder? GEEZERnil nisi bene 15:11, 19. Mär. 2012 (CET)
"Man" sagt das normalerweise nicht so, Ärzte aber schon...--Coatilex 16:40, 19. Mär. 2012 (CET)
(BK) Unter Medizinern scheint das üblich zu sein, vgl. [1] - auf der ersten und zweiten Seite jeweils ein ähnlicher Treffer. ...und kann normalerweise ... gut behandelt werden, meint die OMA dazu --93.104.74.151 16:47, 19. Mär. 2012 (CET)
Hier wiederum in Wikipedia liesst sich das ganz anders. Diese Vielfalt der Aussagen ist es, warum ich Wikipedia ohne intensive Überprüfung als Wissenquelle nicht nutze. Besonders weil ich gelesen habe dass ein Automatenaufsteller die Wikipedia sozusagen beherrscht und der wissenschaftliche Gehalt dadurch beschränkt ist.--212.23.103.45 16:58, 19. Mär. 2012 (CET)
Die Wikipedia wird von einem Automatenaufsteller beherrscht? Wow. Hast du dazu mal nen Link? --88.130.168.16 19:57, 19. Mär. 2012 (CET)
Ich hoffe, du hast die Quelle, die dir das gesagt hat, ebenso kritisch geprüft.  Vorlage:Smiley/Wartung/:-d  --Eike (Diskussion) 21:16, 19. Mär. 2012 (CET)
@212.23.103.45, der Unterschied zwischen hypertroph und dilatativ ist Dir bekannt? --Rôtkæppchen68 21:34, 19. Mär. 2012 (CET)
Hier habe ich das gelesen. Gebe natürlich zu, dass ich das nicht überprüfen kann.--212.23.103.82 21:58, 19. Mär. 2012 (CET)
Doch, kannst du. Hier kannst du leicht feststellen, dass die von dir gemeinte Person überhaupt nicht mehr in der Leitung des Vereins tätig ist. (Wobei ich über die völlige Abstrusität der Idee, Wikimedia e.V. oder gar ein einzelnes Mitglied des (früher) Vorstands bzw. (jetzt) Präsidiums könne „die Wikipedia sozusagen beherrschen“, mal großzügig hinwegsehe.) --Jossi (Diskussion) 22:12, 19. Mär. 2012 (CET)
*gröhl* Ja, wenn man derart vertrauenswürdige Quellen hat, kann man die Wikipedia natürlich in die Tonne treten... --Eike (Diskussion) 22:42, 19. Mär. 2012 (CET)
Gut, der Automatenaufsteller gilt nicht mehr, aber da und in den angezogenen Links stehen Kritiken an der Wikipedia, die weiterhin gültig sein dürften. Als Welt-Online ist die Quelle, wenn auch bei der Presse immer Vorsicht geboten ist, doch einigermaßen seriös. Wenn jemand uneingelullt als nicht Jünger der Wikipedia seine Kritikfähigkeit bewahrt hat, ist er nicht gleich blöd und muß sich angröhlen lassen.--212.23.103.73 08:37, 20. Mär. 2012 (CET)
Man darf und soll allem gegenüber krtisch sein. Aber wenn einer einem Wikipedia-Autor privat hinterherspioniert, spricht das dafür, dass bei dem was im Argen ist, nicht dafür, dass bei der Wikipedia was im Argen ist. Abgesehen davon, Reality Check: Welches Interesse hat ein Automatenaufsteller an der Herrschaft über die Wikipedia? Und wie macht er das? Und warum steht, ganz nebenbei, im verlinkten Artikel kein Wort über Automatenaufsteller? Du musst mir die Fragen nicht beantworten. Es reicht, wenn du sie dir stellst. Du willst doch kein "eingelullter" "Jünger" der Wikipedia-Kritik werden, gelle? --Eike (Diskussion) 10:52, 20. Mär. 2012 (CET)
Irgendwie scheinst Du nicht ganz koscher zu sein. Wer hat hier "privat hinterherspioniert"? In dem Artikel ging es darum, dass die Wikipedia den wissenschaftlichen Ansprüchen nicht genügt und ein Automaten-Tabakhändler das nicht fördert, vermutlich weil er dem wissenschaftlichen Niveau nicht genügt. Mit gewissen stark gläubigen Leuten, wie z.B. mit Sektenmitgliedern soll man nicht diskutieren, also Schluß mit Dir. Um weitere Fantastereien vorzubeugen, ich habe nicht gesagt, dass du ein Sektenmitglied bist, du handelst nur so.--212.23.103.83 11:30, 20. Mär. 2012 (CET)
Vielleicht hattest du versucht, etwas anderes zu verlinken. In dem, was du oben tatsächlich verlinkt hast, kommt jedenfalls das Wort "Tabak" noch nicht mal vor. Es stammt auch nicht von der "Welt", sondern von einem Kanzlei-Blog. Von einem Autoren, der es für angemessen hält, Menschen privat hinterherzuspionieren. Jetzt kannst du dir die Finger in die Ohren stecken und laut summen, aber ändern wird das nichts. --Eike (Diskussion) 11:42, 20. Mär. 2012 (CET)
Die klassischen Taurinmangelsyndrome sind DCM und zentrale Netzhautdegeneration. Der Zusammenhang mit der DCM ist seit 1987 bekannt (PMID 3616607), es gibt so etwas auch beim American Cockerspaniel. Da Taurin bei einer Vielzahl von Prozessen beteiligt ist, sind weitere Erkrankungsbilder denkbar, bekannt sind beispielsweise Fortpflanzungsstörungen (geringe Welpenanzahl, Entwicklungsstörungen der Feten) und Funktionsstörungen der Leukozyten. Taurin wird kommerziellen Futtermitteln zwar zugesetzt, aber der Bedarf kann variieren und beim Einzeltier kann durchaus auch ein Taurinmangel auftreten, da ist am ehesten eine Taurinbestimmung im Serum angezeigt, der sollte über 30 µmol/l liegen (PMID 2658282). Uwe G. ¿⇔? RM 09:30, 20. Mär. 2012 (CET)

Verwaltungsakt "Einbürgerung": öffentlich einzusehen?

Hallo! Die Erlangung der deutschen Staatsangehörigkeit per Verwaltungsakt -hier: die Einbürgerung eines ausländischen Staatsbürgers auf Antrag- ist an gewisse Voraussetzungen gebunden. So ist wohl ein Einbürgerungstest (zwingend?) notwendig; Bestandteil davon ist ein Sprachtest. Der ausländische Bewerber muss laut Staatsangehörigkeitsgesetz, § 10 nachweisen, dass er "die Anforderungen der Sprachprüfung zum Zertifikat Deutsch (B1 des Gemeinsamen Europäischen Referenzrahmens für Sprachen) in mündlicher und schriftlicher Form erfüllt." Ausnahmsweise und unter bestimmten Bedingungen kann davon abgesehen werden ("Von den Voraussetzungen des Absatzes 1 Satz 1 Nr. 6 und 7 wird abgesehen, wenn der Ausländer sie wegen einer körperlichen, geistigen oder seelischen Krankheit oder Behinderung oder altersbedingt nicht erfüllen kann.") Ist ein solcher Verwaltungsakt für Dritte einsehbar (und damit quasi öffentlich)? Kann ich also beispielsweise überprüfen, ob bei einer bestimmten Einbürgerung ein Einbürgerungstest stattfand und eine Sprachprüfung bestanden wurde? Gruß --Sir James (Diskussion) 12:55, 19. Mär. 2012 (CET)

Das wäre ja noch schöner. --AndreasPraefcke (Diskussion) 14:12, 19. Mär. 2012 (CET)
(nach BK) Grundsätzlich: Nein. Eine Anspruchsgrundlage dafür sehe ich nicht; das Staatsangehörigkeitsgesetz enthält keine, das Verwaltungsverfahrensgesetz auch nicht. Vielmehr könnte eine solche Information sogar zu den zum persönlichen Lebensbereich gehörenden Geheimnissen gezählt werden, was eine unbefugte Offenbarung durch die Behörde ausschließt. --Snevern 14:15, 19. Mär. 2012 (CET)
Merci schon einmal. Verwaltungshandeln sollte nachvollziehbar sein. Grundsätzlich kann aber doch jedem Verwaltungsakt widersprochen werden; wenn auch nicht von jedem beliebigen Dritten, oder? Ich vermute daher, dass ein Verwaltungsgericht befugt wäre, eine solche Entscheidung (und deren Verfahren innerhalb einer Behörde) zu überprüfen. Gruß --Sir James (Diskussion) 14:48, 19. Mär. 2012 (CET)
In der Schweiz wird in den meisten Kantonen sogar ad personam abgestimmt, ob jemand eingebürgert werden soll, oder nicht. Da wird bei der Abstimmung sogar eine Kurz-Biographie mit allen relevanten Informationen öffentlich gemacht. In Staaten mit Diktaturerfahrung in der jüngeren Geschichte (zB D, Ö) ist dies ein undenkbarer Eingriff in die Persönlichkeitsrechte, obwohl eigentlich paradox, weil die Behörde weiß ja sowieso alles und sollte eine neue Diktatur kommen, sitzt die ja auf allen Behördenakten. --El bes (Diskussion) 15:27, 19. Mär. 2012 (CET)
Die vom Verwaltungsakt betroffenen können gegen die Entscheidung Widerspruch einlegen (oder manchmal auch direkt klagen). Das Verwaltungsgericht kann ohne Klage den Verwaltungsakt nicht überprüfen. Und es kann auch nicht jeder beliebige Dritte Widerspruch einlegen bzw. klagen. --Snevern 16:17, 19. Mär. 2012 (CET)
Sind die alten Staatsbürger denn nicht auch betroffen? --Eingangskontrolle (Diskussion) 21:52, 19. Mär. 2012 (CET)
Dazu müsstest du erst einmal nachweisen, dass die Einbürgerung eines Dritten ein dich belastender Verwaltungsakt ist. Dass es dich persönlich in deinen Rechten beschränkt oder benachteiligt, wenn Herr Dupont oder Frau Öztürk einen deutschen Pass bekommt, dürfte nicht ganz einfach zu beweisen sein. Und komm jetzt bloß nicht mit Belastung der Sozialsysteme oder so. Wenn das ziehen würde, dann dürftest du auch gegen jeden einzelnen hierzulande erlassenen Hartz IV-Bescheid Widerspruch einlegen. --Jossi (Diskussion) 22:21, 19. Mär. 2012 (CET)
+1. Die "alten" Staatsbürger sind von der Entscheidung ebensowenig betroffen wie von der Geburt eines Kindes (welches ja häufig in ebendiesem Moment ebenfalls die deutsche Staatsangehörigkeit erwirbt).
Zahlreiche bereits mit deutscher Staatsangehörigkeit geborene Mitbürger beherrschen bis zum Erreichen des Grundschulalters die deutsche Sprache nicht ausreichend. Das geht ganz ohne Verwaltungsakt vor sich, belastet aber durchaus "alte" (und junge) Staatsbürger in ganz erheblichem Maße (zum Beispiel die Mitschüler, weil das schwächste Glied die Stärke der Kette bestimmt, und die Lehrer, die erstmal grundlegende Kenntnisse der Sprache vermitteln müssen, bevor sie ihre eigentlichen Lehrinhalte an den Mann das Kind bringen können).
Der gemeine Landsmann wird also im Sinne des Verwaltungsverfahrensrechts weder durch die Entscheidung über die Einbürgerung belastet noch durch die Entscheidung, ob dieser Vorgang mit oder ohne Sprach- und Staatsbürgerkunde-Test erfolgte. Er kann daher weder durch Widerspruch noch durch Klageerhebung dagegen vorgehen und hat als Nicht-Beteiligter grundsätzlich auch kein Akteneinsichts- oder sonstwie geartetes Informationsrecht.
Und das ist auch gut so. --Snevern 22:40, 19. Mär. 2012 (CET)

Strickstrack

Im Zusammenhang der Bundespräsidentenwahl hörte ich den Familiennamen Strickstrack (ältester Wahlmann, aus Celle glaub ich) zum ersten Mal. Wer kann Angaben zur Herkunft dieses ungewöhnlichen und irgendwie lustig klingenden Namen machen? Angaben könnten, sofern belegt, auch in einen neu anzulegenden FN-Artikel einfließen (Strickstrack (Familienname)). mfg,Gregor Helms (Diskussion) 17:54, 19. Mär. 2012 (CET)

Bahlow (S. 504-505) legt Zusammensetzung aus Strick(er)(t) und Strack(e) nahe und gibt weitere lautmalerische Namen-Beispiele wie Trictrak, Schnickschnack, Pickenpack, Strickstramm (!), Hottenrott (alle ND). http://www.gen-evolu.de/ listet den Namen 24 x (Celle ist auch ein Strickstrack-Hotbed) LAUTMALERISCH: Na bitte... GEEZERnil nisi bene 18:03, 19. Mär. 2012 (CET)
Vielen Dank! Gregor Helms (Diskussion) 08:10, 20. Mär. 2012 (CET)

Deutsches Reich in den Grenzen vom 31. Dezember 1937

Kann jemand ausrechnen, die hoch die Gebietsverluste zwischen

prozentual ungefähr betragen? --Bergfex2011 (Diskussion) 18:48, 19. Mär. 2012 (CET)

Gebietsverluste: Deutsches Reich 1937 ggü.

BRD ohne Saarland 47,44 % BRD mit Saarland 46,90 % Deutschland nach 1990 23,82 % --HeinzWörth (Diskussion) 19:31, 19. Mär. 2012 (CET)

Im Zeitalter der endlich abklingenden Nationalstaaten muss man da aber nicht mehr von "Verlust" sprechen. Die Landschaften sind alle noch da. Dass sie heute nicht mehr unter der Regierung von 1937 stehen, haben sie mit Gegenden wie Hessen oder Thüringen gemeinsam. Kirschschorle (Diskussion) 19:37, 19. Mär. 2012 (CET)
<quetsch>In den 1980ern glaubte man schon einmal, das Zeitalter der Nationalstaaten gehe zu Ende. Das war damals ein Irrtum, und es ist auch jetzt einer. Aber im übrigen hast du natürlich recht. --Snevern 22:27, 19. Mär. 2012 (CET)</quetsch>
<quetschauch>Ja, ich muss auch ab und zu der Realität ins Auge und das Ende erst in weiter Ferne sehen. Es ist aber auch eine Frage des Kontextes. Jedenfalls sagte ich vorsichtshalber "abklingen", wie bei diesen Atommülldingern, die ja auch so schnell nicht völlig verschwinden. Aber abklingen tun sie schon; an manchen Nationalstaaten holt man sich noch üble Brandblasen, andere kann nach inzwischen (vielleicht nach etwas Training an einem alten Elektroherd) schon anfassen... Kirschschorle (Diskussion) 01:37, 20. Mär. 2012 (CET) </quetschwiederaus>
Ich meinte mit dem Wort Verlust die Minimierung der Territorien. Jedenfalls danke für die Infos, ich trage diese mal gutgläubig in den Artikel Deutsches Reich in den Grenzen vom 31. Dezember 1937 einpflegen. --Bergfex2011 (Diskussion) 20:20, 19. Mär. 2012 (CET)
Gemeinhin nennt man es "Veränderung des Gebietsstandes". Wenn dich statt der Prozentzahlen auch die konkreten Flächen in Quadratkilometern interessieren, dann schau mal ins Statistische Handbuch (ab Seite 6). -- Wiprecht (Diskussion) 20:59, 19. Mär. 2012 (CET)
Super Digitalisat, danke dafür. Ich bereite alles erstmal hier vor.--Bergfex2011 (Diskussion) 07:47, 20. Mär. 2012 (CET)
"Deutschland ist das Land mit den meisten internationalen Grenzen (9)." Das ist Quatsch. China und Russland haben beide schonmal mehr. --тнояsтеn 13:14, 20. Mär. 2012 (CET)
Was will man denn unter abklingenden Nationalstaaten verstehen? Es ist sicher so, dass das gegenseitige Köpfeeinschlagen zwischen europäischen Nationalstaaten stark abgenommen hat, aber im Grunde waren die Nationalstaaten nie so gefestigt wie heute. In den meisten Staaten wurden Sprachminderheiten ebenso wie die Dialekte der Nationalsprache weitgehend marginalisiert und wo vorher fließende Dialektkontinua zu den Nachbarstaaten bestanden, entstanden Sprachbarrieren. Insofern ist der Nationalstaat ziemlich erfolgreich.
Und die Kritik am Begriff Verlust ist auch daneben. Wenn ein Staat kleiner wird, dann hat er Gebiet verloren. Die Bundesrepublik ist jedenfalls Nachfolgestaat des Deutschen Reichs. Und auch wenn ein moderner Staat seine Bürger unabhängig ihrer Identität achten sollte, so hat der Staat doch auch heute noch den Namen Deutschland, weil er unter der Prämisse gegründet wurde, die von Deutschsprachigen besiedelten Regionen unter einer Regierung zu vereinen. Insofern ist die Abgabe und die Vertreibung der Deutschsprachigen aus diesen Gebieten ein Verlust im Sinne der Staatsprämisse. (Der natürlich ein enormer Bruch der eigenen Staatsprämisse vorausging, als das Deutsche Reich versuchte, riesige Gebiete unter seine Regierungsgewalt zu bringen, die nicht deutschsprachig waren. Insofern ist es wohl aus dem Blickwinkel des Staates nicht gerechtfertigt, diesen Verlust zu betrauern, denn aus dem Blickwinkel des Staates ist er selbstverschuldet. Aber der Verlust darf aus dem Blickwinkel der betroffenen Individuen sicherlich sehr wohl betrauert werden, da diese im Regelfall keine individuelle Schuld trifft.) --::Slomox:: >< 10:42, 20. Mär. 2012 (CET)

Hallo liebes Auskunftsteam,

mein Sohn (15) leidet unter dem Asperger-Syndrom. Ihm stört es enorm, dass er teils massiv geärgert wird und auf der anderen Seite auch ignoriert wird. In der Schule sowohl als auch in der Öffentlichkeit fällt er auf: Die Kinder aus diversen Schulen inklusive seiner eigenen gucken in komisch an und geben teils beleidigende Worte ab, die mein Sohn ignoriert, indem er weiterläuft, was welche noch mehr amüsiert. Seine eigene Klasse hat sich an dem „merkwürdigen“ Verhalten meines Sohns gewöhnt. Allerdings wird er dort derzeit ignoriert – mein Sohn hat Interesse innerhalb der Schulzeit, ein einigermaßen gutes Verhältnis mit seiner eigenen Klasse aufzubauen und nicht im schulischen als auch im öffentlichen Bereich negativ aufzufallen. Kinder zu sich nach Hause einzuladen lehnt er allerdings ab.

Ich hoffe ihr könnt mir für meinen Sohn Tipps geben, die ihm weiterhelfen in seiner recht unangenehmen Situation. Fachleute aus dem Bereich gaben mir eher unzufrieden stellende Antworten. So sagten sie: „Der Junge muss sich eben den anderen anpassen.“. Die Frage bleibt doch „Wie?!“. --93.131.78.121 18:57, 19. Mär. 2012 (CET)

Das ist leider häufig so, dass das "Anecken" von Autisten aller Sparten mit so dümmlich-ignoranten Sprüchen abgefertigt wird. Ein Autist KANN sich anderen nicht einfach mal so anpassen, das werden ihr Stolz und ihr "mentales Gefängnis" gar nicht erst zulassen. Und unsere Gesellschaft wird leider nie wirklich bereit sein, Autisten einfach zu akzeptieren. Sinnvoll und fördernd wäre es, Ihren Teenager in einer speziellen Schule/Einrichtung unterzubringen, wo er unter seinesgleichen wäre und weitestgehend professionell betreut und gefördert würde. Das allerdings müsste er aber auch wollen. Tun Sie niemals etwas, das seinem Wesen und seinem Stolz widerspricht! LG;--Nephiliskos (Diskussion) 19:05, 19. Mär. 2012 (CET)
Das beste wäre Kontakt mit anderen Eltern in der gleichen Situation aufzunehmen. Evtl. hier - am besten lokal. Da kommen manchmal Hinweise und Hilfestellungen, die aus dem praktischen Leben kommen und man kann sich "positionieren". Grüsse an den jungen Mann, er ist nicht allein. GEEZERnil nisi bene 19:11, 19. Mär. 2012 (CET)
Seltsam in der Frage sind ja die "Fachleute", die ein "Anpassen" fordern. Wurde tatsächlich Asperger diagnostiziert? Wenn ja, dann sind die Fachleute Diejenigen, die diese Diagnose erarbeitet haben. Und die sollten auch weiterhelfen können.
Unabhängig davon fehlen natürlich bei denjenigen Gleichaltrigen (Arschlöchern), die den Jungen ärgern und beleidigen, notwendige und eigentlich selbstverständliche Informationen und erzieherische Einwirkungen seitens der jeweiligen Eltern und Lehrer. Kirschschorle (Diskussion) 19:23, 19. Mär. 2012 (CET)
Ich denke nicht, daß die Gleichaltrigen pauschal als Arschlöcher abzustempeln sind. Das Problem wird doch hier vor allem im Alter (15) der Beteiligten liegen, wo nicht nur die Krankheit des Aspergers eine Rolle spielt, sondern das ganze Spektrum einer Gruppe Pubertierender. Und so hart es klingt, die Experten haben recht. Es bringt nichts mehr, wie bei Kindern an den Anpassungswillen der Umwelt zu appellieren, sondern der Kranke muss sich nunmal der Gesellschaft anpassen, was allgemein das Erwachsenwerden ausmacht. Da hilft keine Medizin und auch keine Einwirkung durch Autoritäten. Problem ist ja auch, daß Asperger heute viel zu häufig als "Modekrankheit" diagnostiziert wird, wobei es viele Symptome gibt, die schon immer Außenseiter prägten. Fehlende Fähigkeiten wie Willen zur Interaktion mit der Umwelt wird immer zur Ausgrenzung führen, genauso wie eigene Ablehnung gegenüber anderen Besonderheiten (Brille, Dicke, Kleine, Dumme, Ausländer, Körperbehinderungen). Die Lösung kann nur in der Hilfestellung bestehen, andere soziale Gruppen außerhalb der Schule zu finden, wo er beim Status Null anfangen kann. Mit 15 stehen da ja andere Möglichkeiten offen, als für einen Zehnjährigen.Oliver S.Y. (Diskussion) 19:49, 19. Mär. 2012 (CET)
Deshalb ja meine Frage nach der tatsächlichen Diagnose - war nicht klar?
Kirschschorle (Diskussion) 19:54, 19. Mär. 2012 (CET)
Wie von anderen bereits erwähnt, sind allerlei soziale Spannungen bei 15-jährigen mehr oder minder "normal" (siehe etwa Pubertät #Stimmungsschwankungen, Launenhaftigkeit oder Pubertät #Frühentwickler und Spätentwickler). Für einen jungen Menschen mit Asperger-Syndrom kann es aber besonders schwer sein, sich anzupassen, durchzusetzen, Freunde zu finden und ihm nicht Wohlgesonnene zu erkennen, also das soziale Geflecht und die Regeln zu durchschauen. Vielleicht hilft es ihm, ein soziales Kompetenztraining zu absolvieren. Dort kann er lernen, die eigene Position vertreten und eigene Rechte durchzusetzen, aber auch auf andere einzugehen und um Sympathie zu werben. Vielleicht hilft es dem Betroffenen, wenn soziales Verhalten so auf klare Regeln runtergebrochen wird. Ansonsten schließe ich mich Grey Geezer an: Hier ist eine Übersicht über Selbsthilfegruppen, wo man sich mit anderen Betroffenen austauschen kann. Gruß --stfn (Diskussion) 21:40, 19. Mär. 2012 (CET)
Falls es zu arg wird für den Sohn und die körperlichen Kräfteverhältnisse nicht zu sehr unterschiedlich sind... hat er die Option, in überraschender Weise (um als Sieger vom Platz zu gehen) dem mobbenden Gegner die Nase mit der Faust zu polieren. Es heißt zwar immer, Gewalt sein keine Lösung, das ist aber - leider - nicht unter allen Bedingungen richtig (denn in manchen Fällen ist ein nach Regeln [kein Schlagen oder Treten eines zu Boden gegangenen Gegners, keine Gegenstände als Waffen, nur Fäuste einsetzen] geführter Kampf ein schnelles und effektives Mittel zur Konfliktlösung). In diesem Fall (pubertierende Jugendliche) spielen Rangkämpfe meiner Meinung nach eine große Rolle (wer ist der stärkte, wer ist der beste, wer kommt als erstes an das weibliche Geschlecht ran...), und schafft es der Sohn, sich als körperlich starkes Individuum zu positionieren, so wird er einerseits in Ruhe gelassen und andererseits wahrscheinlich in eine Gruppe integriert werden. Durch die Partizipation an den zwischenmenschlichen Auseinandersetzungen um Macht und Rang fällt er schnell aus der Rolle als unverstandener Sonderling heraus, wodurch die Schulkameraden dann wahrscheinlich die mit Asperger oft einher gehende Phantasie und Begeisterungsfähigkeit entdecken und den Sohn akzeptieren werden. Ganz wichtige Voraussetzung - die Eltern dürfen vor etwaigen Beschwerden von anderen Eltern und Schulpersonal nicht einknicken, sondern müssen ihren Sohn darin bestärken, dass er von ihnen immer geliebt und unterstützt werden wird. Ich würde erwarten, dass er mit an Sicherheit grenzender Wahrscheinlichkeit vielleicht 3, 5 oder bis zu 8 Kämpfe auszufechten hat, er aber mitnichten zu einem Schläger werden wird, sondern danach eine angenehme Ruhe im Sozialverband der Klasse einkehrt und der Sohn mindestens leidlich gut integriert ist. Grüße, Grand-Duc (Diskussion) 22:08, 19. Mär. 2012 (CET) PS: falls pädagogisches Personal gegen solche Maßnahmen protestiert, so möge diesem Personal wärmstens die Lektüre von Irenäus Eibl-Eibesfeldt: Die Biologie des menschlichen Verhaltens. ISBN 978-3-492-03987-1. nahegelegt werden...
Wo hast Du den Tip her? Stand der in nem Machoheft aus den 70ern? Das mag vieleicht für Rangeleien in der Grundschule gelten, unter 15jährigen gibt es nicht mehr die Tradition des Faustrechts. Wen soll er denn verprügeln, um "soziales" Ansehen zu erwerben, andere Außenseiter? Tolle Idee zur Problemlösung, die anderen Gruppen dürften bereits feste Strukturen haben, und sicher schaut heute keiner ruhig zu, wenn ein Asperger seinen Kumpel verprügelt, dann hat er es bestenfalls mit mehreren Gegnern zu tun, und wird nur verprügelt. Es kann aber auch einfach jemand nen Messer zücken, und ihn abstechen. Und als dritte Option gibts dann noch die rechtliche Ahndung im Zeugnis oder durch Staatsanwalt und Polizei. Der Junge hat bereits den Außenseiterstatus, meinst, daß sind wirklich Optionen, die ihm helfen? Oliver S.Y. (Diskussion) 22:16, 19. Mär. 2012 (CET)
Jetzt wüsste ich bloß zu gerne, was ein Machoheft sein soll. Nein, nicht andere Außenseiter verprügeln, eher die ranghohen Mobber - und zwar wenn nötig per Überraschungsangriff, damit die ganze Geschichte in ca. 10 Sekunden positiv (für den Sohn) entschieden ist. Was die rechtlichen Probleme angeht: ständiges Hänseln kann gerne zur Körperverletzung nach §223 StGB werden, bei einem Akutgeschehen hat man dann ggf. die Möglichkeit der Notwehr (oder, wenn nicht, muss sich der Staatsanwalt dann mit den Problemen wie Notwehrexzess oder verminderter Schuldfähigkeit durch die Behinderung "Asperger-Syndrom" auseinandersetzen, vermutlich ist das zu viel Aufwand für eine blutige Nase). Außerdem habe ich nichts von "verprügeln" geschrieben, das dauert nämlich zu lange und ist zu riskant, der Überasschungseffekt und das Zeigen des Mutes, gegen die Hänseleien vorzugehen, sind wichtiger. Grüße, Grand-Duc (Diskussion) 22:26, 19. Mär. 2012 (CET)
Die Eskalation von verbaler zu körperlicher Gewalt müsste er sich trotzdem zurechnen lassen. Vor 50 Jahren konnte ein Lehrer in bestimmten Situationen dem Quäler, Hänsler und Stichler, der verdientermaßen eins auf die Nase bekommen hatte, noch sagen: „Geschieht dir ganz recht“, heute könnte sich das kein Pädagoge mehr erlauben. Zuschlagen ist als Mittel der Konfliktlösung gesellschaftlich nicht mehr akzeptiert. --Jossi (Diskussion) 22:36, 19. Mär. 2012 (CET)
Besser eine selbst kontrollierte Eskalation als monatelanges Leiden in der Schule! Wegen der Gesellschaft: Deswegen ja der Verweis auf I. Eibl-Eibesfeldt... Fast alle ganzen schönen Pädagogikkonzepte versagen meiner Meinung nach im Angesicht der psychischen Brutalität von pubertierenden Menschen, für effektive Alternativen (Schulrausschmiss der Mobber, Strafanzeige der Eltern der betroffenen Schüler gegen die Täter o.ä.) sind wohl die meisten Lehrer und Schulleiter zu schwach, zu demotiviert oder zu sehr wegschauend ("Mobbing? Sowas gibt es bei uns nicht!" währenddessen im Klassenraum hinter ihnen beispielsweise die Schultasche von einem etwas pummeligen Mädchen erst durch den Klassenraum und dann in die Schultoilette geworfen wird). Als Junge kann wenigstens noch der Vorteil bestehen, eine körperliche Überlegenheit herzustellen. Scheiß auf die gesellschaftliche Meinung wenn es um die eigene psychische Gesundheit geht, und irgendwann ist man dann aus der giftigen Atmosphäre raus und schon hat die Gesellschaft vergessen, dass auf dem Weg daraus 3 Nasenbeine zu Bruch gingen. Grüße, Grand-Duc (Diskussion) 22:46, 19. Mär. 2012 (CET)
Wie gesagt, ich weiß wirklich nicht, aus welchem Keller Deinen Ansichten zum Gruppenverhalten und Einzelverhalten stammen. Der Junge hier hat zumindest ein diagnostiziertes Aspergersyndrom, auch wenn das keine Erkrankung wäre, die Verhaltensstörung bzw. -auffälligkeit ist vorhanden. Selbst wenn die vorgeschlagene Gewalt eine Lösung gegenüber anderen Personen wäre, sein verändertes Verhalten würde sich durch solchen 10 Sekundenausbruch nicht ändern, und wir sind keine Berggorillas, wo sich geschlechtsreife Weibchen von solchem Überfall beeindrucken lassen. Und wie sagte schon mein Opa, wenn Du jemanden unten hast, verletzte ihn so, daß er nicht mehr aufsteht, denn ansonsten schwebst Du fortan in Gefahr bzw. hast zusätzlich zum Mobbing durch den Rest noch die Angst vor der Rache Deines Gegners. Denn was Du kannst, kann er auch, nur mit sehr viel mehr Mut, und Gewalt hat den Nachteil, daß sie meist unkontrolliert eskaliert. Und am Ende steht der Satz "Anschließend richtete er die Waffe gegen sich selbst". Da klingt für mich der Vorschlag, Integration in anderen Gruppen wie Sportverein, Tanzverein oder Sozialarbeit zu suchen wesentlich zielführender. Oder anders gefragt, wer von diesen Vieren hat vermutlich Asperger [2]. Ich tippe mal auf Nr.3, aber den mobbt sicher niemand deswegen. Oliver S.Y. (Diskussion) 23:04, 19. Mär. 2012 (CET)
cocooning? --Heimschützenzentrum (?) 23:07, 19. Mär. 2012 (CET)
Bei Gorilla sp. lassen sich Weibchen nicht unbedingt beeindrucken, denn ein Männchen, dass den herrschenden Silberrücken entthront hat, muss sich erst die Zuneigung verdienen. AFAIK hat Dian Fossey bei solchen Geschehnissen beobachtet, dass einige Weibchen dem scheidenden Silberrücken hinterher gewandert sind und sich so die Gruppe teilte. Außerdem meine ich, dass es sich empfiehlt, vorsichtig mit Affengleichnissen zu sein, denn Pan troglodytes führt Vernichtungskriege gegen Nachbarn, der Pan paniscus hat eine Sexualmoral, die Papa Ratzo so gar nicht gefallen dürfte, Gorilla sp. dürfte der Alptraum jeglichen Sportwagenbauers sein (extrem kleine Hoden - es muss schließlich kein Weibchen "ausgespült" werden, wenn man Befruchtungserfolg haben will) und der Homo sapiens reiht sich da irgendwie passend ein... Und ja, ich fände es schöner, wenn Sportvereine, Interessengemeinschaften für Kunst, Naturwissenschaften, soziale Dienste etc. ausreichend Ausgleich bieten könnten. Wenn das nicht reicht, bleibt nur noch Gewalt - oder der Betroffene zerbricht wahrscheinlich innerlich am psychischen Leidensdruck. Grüße, Grand-Duc (Diskussion) 23:20, 19. Mär. 2012 (CET)
Ob Gorillaweibchen sich von Gewalt beeindrucken lassen, weiß ich nicht, aber Menschenweibchen offensichtlich schon. Dass eher die "bösen" Jungs, die gewalttätigen Männer, deutlich mehr "Weibchen" um sich scharen, ist ja wohl offenkundig. Auch wenn man davor vielleicht gern die Augen verschließen möchte, weil auch dieses Stück Realität nicht zu der Welt passt, die man sich mit gebetsmühlenartig wiederholten Gehirnwäschefloskeln wie "Gewalt ist keine Lösung" und "Zuschlagen ist als Mittel der Konfliktlösung gesellschaftlich nicht mehr akzeptiert" herbeiphantasieren will. --178.202.39.166 00:10, 20. Mär. 2012 (CET)
Anpassung vs Gewaltanwendung - welche Lösung hilft einem kranken Menschen durch solche Lebensphase? Wer aus seiner Lebenserfahrung wirklich den Schluss zieht, daß Gewalt eine Lösung ist, hat in meinen Augen entweder noch nicht die finale Eskalation erlebt, oder ist genau solch ein Täter, der oben als Mobber geschildert wird. [3] zeigt, wohin es führt, wenn Gewalt als Lösung von allen Seiten akzeptiert wird. Ein Messerstich in Notwehr, oder ein Messerstich als Körperverletzung, Grautöne gibt es da nicht.Oliver S.Y. (Diskussion) 02:58, 20. Mär. 2012 (CET)
Es ist immer wieder hübsch zu sehen, auf wie viele Arten man missverstanden werden kann. Der Satz "Zuschlagen ist als Mittel der Konfliktlösung gesellschaftlich nicht mehr akzeptiert" war von mir als Tatsachenaussage im Sinne einer neutralen Zustandsbeschreibung gemeint. Was daran eine "gebetsmühlenartig wiederholte Gehirnwäschefloskel" sein soll, mit der man sich etwas "herbeiphantasieren" will, ist mir schleierhaft. Oder bist du der Meinung, dass Zuschlagen als Mittel der Konfliktlösung hierzulande gesellschaftlich akzeptiert sei? Dann bewegst du dich offenbar in anderen Gesellschaftskreisen als ich. --Jossi (Diskussion) 11:58, 20. Mär. 2012 (CET)

Übrigens, ein sehenswerter Film zur Thematik von Dauermobbing und Asperger: Ben X; ist allerdings nict unbedingt etwas für Menschen mit schwachen Nerven. Als Lösungen bieten sich an, a) Lehrer informieren und darauf drängen, dass die sich die Mobber mal vornehmen; wenn das nichts hilft, weil die Lehrer trotz wiederholter Ansprache das Thema nicht ernst nehmen, b) Schule wechseln. --88.73.28.255 07:57, 20. Mär. 2012 (CET)

→ Siehe auch die Frage kürzlich, scheint vom gleichen Fragesteller zu stammen: Wikipedia:Auskunft/Archiv/2012/Woche 09#Asperger-Syndrom-Frage --тнояsтеn 13:21, 20. Mär. 2012 (CET)

Papierpreislisten im Supermarkt (erl.)

Für einige Waren (warum nur einige?) gibt es bei vielen Supermarktkassen(plätzen) in Deutschland eine Vorrichtung, die um 45 Grad geneigt ist. Das Herzstück besteht aus mehreren doppelseitigen Kunststoffblättern mit transparenten Folien, die Rahmenfarben sind unterschiedlich. Hinter den Folien sind papierne Preis- und/oder EAN-Listen im Format A 4 hineingesteckt. Der Kassier (richtige Bezeichnung?) kann ganz schnell nachschlagen. Die letze gewählte Doppelseite verbleibt aufgrund der Neigung und Gravitation. Das Objekt steht auf einem Minimasten. Wie heißt dieser Gegenstand?

--77.4.95.214 20:01, 19. Mär. 2012 (CET) Ich hatte die Frage vor ca. 1/4 Jahr hier mal gestellt, finde den Beitrag jedoch nicht mehr in der Archiv-Suche.

Die Firma Durable in Iserlohn beispielsweise bietet die Dinger als Sichttafeln an: http://www.durable.de/de/produkte/7/77/informations-und-praesentationssysteme/sherpa-sichttafel-systeme.html LG --Gwexter (Diskussion) 20:14, 19. Mär. 2012 (CET) Hatte grad einen betriebsunfall hier? Durch BK?
Sichttafel scheint korrekt zu sein (ganz schön hochpreisig), Danke!!! --77.4.95.214 20:55, 19. Mär. 2012 (CET)
Guckstdu: Wikipedia:Auskunft/Archiv/2012/Woche 05#Stabiles Heft (erl.) --тнояsтеn 21:00, 19. Mär. 2012 (CET)


Auf diesen Listen befinden sich übrigens meist PLUs und keine EAN-Codes. --Dubaut (Diskussion) 09:36, 20. Mär. 2012 (CET)

Ist die Energieabsoption von CO2 Temperatur abhängig?

Hallo, ich kenne jemanden der meint das die "Energieabsorption" von CO2 erst ab einer Temperatur von -78 Grad Celsius beginnt, und somit CO2 nicht für den Anstieg der Temperatur verantwortlich ist^^ Leider find ich dazu so gut wie gar nix, ich fürchte es handelt sich hierbei um "geschwurbel" trotzdem hätte ich gerne die genaueren Hintergründe^^ Gruß --Lexikon-Duff (Diskussion) 21:08, 19. Mär. 2012 (CET)

Schau mal hier die "Zweite Behauptung": [4]. Desweiteren [5]. --тнояsтеn 21:15, 19. Mär. 2012 (CET)
Hier hast Du ein Infrarot-Absorptionsspektrum von CO2. Die Abszisse ist in cm−1 unterteilt, siehe Wellenzahl. Ich sehe deutliche Banden bei 2300 cm-1, 3600 cm-1 und 3700 cm-1 (4300, 2800 und 2700 nm). --Rôtkæppchen68 21:29, 19. Mär. 2012 (CET)

Alkoholmarken in Musikvideos

Hallo Auskunft. Ist das zurzeit ein Trend, dass in aktuellen Musikvideos Product-Placements von Alkoholmarken dazu gehören?

Ist das sonst jemandem aufgefallen? Welche Regelungen (Werbeverbote) werden dadurch ausgehebelt? Schöne Grüße --Heiko (Diskussion) 00:53, 20. Mär. 2012 (CET)

330.000 Hits mit Google => product placement alcohol music videos <=
Nach der Lektüre von Keith Richards Biographie kann man spielend das Vordiplom (Bachelor?) in Pharmazie machen... GEEZERnil nisi bene 07:16, 20. Mär. 2012 (CET)
Des Staatsexamens erster Theyl. --78.50.225.145 15:12, 20. Mär. 2012 (CET)
Ich habe erst mal nachsehen müssen, was "smack" bedeutet ... --Zerolevel (Diskussion) 11:33, 20. Mär. 2012 (CET)

Karl I. (England), Privatbriefe an ältesten Sohn in den Niederlanden

Wo finde ich die gerade auf die Schnelle? --77.188.159.169 01:04, 20. Mär. 2012 (CET)

vllt. in dem Buch The letters, speeches and proclamations of King Charles I? Catfisheye (Diskussion) 02:37, 20. Mär. 2012 (CET)

was ist ein virtuelles Laufwerk?

--193.170.149.104 08:11, 20. Mär. 2012 (CET)

Das ist der Titel eines Wikipedia-Artikels. --Rôtkæppchen68 08:16, 20. Mär. 2012 (CET)

ist das auch ein USB Stick??? (nicht signierter Beitrag von 193.170.149.104 (Diskussion) 09:07, 20. Mär. 2012 (CET))

Nein. In ein real existentes Laufwerk kannst du eine real existierende CD/DVD stecken. Wenn du von der DVD eine virtuelle Kopie (z.B.: Images im *.iso Format) erstellst, kannst du die DVD als eine Datei auf dem PC Speichern. Das ist dann eine virtuelle DVD. Damit du die abspielen kannst, musst du dem PC irgendwie vorgaukeln, dass du sie in ein DVD-Laufwerk einlegst. Das ist dann das virtuelle Laufwerk. Es ist im System wie ein echtes Hardware-Laufwerk hinterlegt und wird auch so angesteuert, in wahrheit verbingt sich dahinter jedoch Software. -- 208.48.242.106 09:25, 20. Mär. 2012 (CET)
Du kannst natürlich eine virtuelle DVD auf deinen USB-Stick speichern, also die *.iso Datei. Das virtuelle Laufwerk ist aber eine Art Treiber in deinem System. -- 208.48.242.106 09:29, 20. Mär. 2012 (CET)

In der kernigen alten Zeit vor Windows, als man den Rechner noch mit einzelnen Befehlen auf der DOS-Ebene steuern musste und die Betriebssysteme Namen wie MS-DOS, PC-DOS oder DR-DOS trugen, konnte man mit HIMEM.SYS im "'High Memory" oberhalb des adressierbaren Limits von 640 kB ein "virtuelles Laufwerk" einrichten. Bei einem Speicherstein von 1 MB waren das 360 kB, d.h. es passte ein 5,25 "-Floppy-Inhalt gut hinein. Wenn die echte HDD unter C: lief, konnte man das virtuelle Laufwerk z.B. als D: ansprechen. Vorteil: Es war viel schneller als das Floppy-Laufwerk; Gefahr: Wenn der PC ausgeschaltet wurde, war der Inhalt unwiderruflich weg. --Zerolevel (Diskussion) 10:51, 20. Mär. 2012 (CET)

Das ist eine RAM-Disk, ein Sonderfall eines virtuellen Laufwerks. Himem.sys hat übrigens nur mittels der berüchtigten A20-Leitung den Speicherbereich von 0x100000 bis 0x10FFEF (knapp 64 KiB) im Realmode adressierbar gemacht. Für die von Dir beschriebenen UMBs brauchte man emm386.exe. --Rôtkæppchen68 11:43, 20. Mär. 2012 (CET)

Morbus Fabry

für Morbus Fabry bestimmung wird eine Blutuntersuchung notwendig, welcher Wert wird benötigt.

Vielschichtig, so nicht zu beantworten => Morbus_Fabry#Diagnose. GEEZERnil nisi bene 09:56, 20. Mär. 2012 (CET)

Film gesucht

Hallo, eine Frage von geringer Dringlichkeit. Ein vermutlich französischer Film aus den 2000er Jahren. Protagonist ist ein Personaler, der die Aufgabe hat, soundsoviel Menschen zu entlassen und letzten Endes entlässt er sich selbst, um die Zahl zu erfüllen. Nebenbei bröckelt sein Leben, seine Freundin verläßt ihn, als sie ihn auf der Toilette sitzend eine Wurst essend mit dem Notebook auf den Knien sieht.

Jemand eine Idee? Merci. Catfisheye (Diskussion) 00:19, 20. Mär. 2012 (CET)

Ich weiß nicht warum aber ich muß da an Up in the Air denken. Zwar USA, aber Thema.Oliver S.Y. (Diskussion) 02:42, 20. Mär. 2012 (CET)
Leider nein, aber danke. Catfisheye (Diskussion) 03:12, 20. Mär. 2012 (CET)
Die Story klingt nach "Der Schnee am Kilimandscharo" (F 2011, Regie: Robert Guédiguian, Darsteller: Ariane Ascaride, Jean-Pierre Darroussin). Allerdings ist der erst vorige Woche in Deutschland angelaufen. --Snevern 08:26, 20. Mär. 2012 (CET)
Gesehen habe ich ihn circa 2007 auf DVD im italienischen Fernsehen, war aber nahezu sicher _kein_ italienischer Filmund ich bin mir nicht sicher, ob überhaupt auf Deutsch. :/ Catfisheye (Diskussion) 18:39, 20. Mär. 2012 (CET)
Könnts dieser sein? --BlueCücü (Diskussion) 19:00, 20. Mär. 2012 (CET)
Da gehts auch um oben schon genannten Kilimandscharo-Film. Sorry. War zu voreilig. --BlueCücü (Diskussion) 19:02, 20. Mär. 2012 (CET)
Spontan würde ich auf Ressources humaines (Der Jobkiller) von Laurent Cantet tippen. --César (Diskussion) 21:15, 20. Mär. 2012 (CET)

Hallo, ressources humaines war es leider auch nicht. Der Film war schon eher in der 2000er-Jahre-Ästhetik, also "cleaner/stylisch". Ich habe derweil fr:Catégorie:Film sur le monde du travail durchgesehen. Da war er nicht dabei. Catfisheye (Diskussion) 21:40, 20. Mär. 2012 (CET)

Hmm ... Dann vielleicht mal beim Oracle nachfragen? Oder im Allocine-Forum?--César (Diskussion) 22:01, 20. Mär. 2012 (CET)

Sei Dir niemals in etwas sicher. War doch ein italienischer Film... it:Volevo solo dormirle addosso. Sein Chef sollte Franzose sein. Schlechte Ausrede, ich weiß. Aber danke, mein Gehirn hat in Panik davor, irgendetwas auf frz. schreiben zu müssen, mich nochmal auf Italienisch statt auf Französisch suchen lassen. Danke Euch allen! Catfisheye (Diskussion) 22:21, 20. Mär. 2012 (CET)

Archivierung dieses Abschnittes wurde gewünscht von: тнояsтеn 09:49, 21. Mär. 2012 (CET)

Kommt jemand an das Archiv von The Independent vom 17. Juli 1995? Oder hat jemand Zugriff aufs FAZ-Archiv?

Es geht um den Artikel William Pearson (Bariton), der Belege braucht. Es gab einen Nachruf, http://www.highbeam.com/doc/1P2-4727890.html. Ich habe vergeblich versucht, den Nachruf im Archiv der Zeitung zu finden, siehe hier und scheitere technisch. Kann jemand helfen, bitte, und es an meiner Stelle versuchen? Gruß und vielen Dank, --Liesbeth 15:18, 21. Mär. 2012 (CET)

Hiersteht ja auch unten „To read the full text of this article and others like it, try us out for 7 days, FREE!“. Zugang zum vollen Artikel also scheinbar nur nach Anmeldung. -- 208.48.242.106 15:31, 21. Mär. 2012 (CET)
Hilfe zur Selbsthilfe: Bewirb dich für ein Jahr Highbeamzugriff. --88.130.176.102 15:34, 21. Mär. 2012 (CET)
Ich finde in der Artikelübersicht des besagten Tages den Gewünschten Nachruf garnicht, er steht einfach nicht drin. -- 208.48.242.106 15:38, 21. Mär. 2012 (CET)
Ich schätze, wenn du dich anmeldest, erscheint der gesamte Text auf der oben von dir verlinkten Seite. Da brauchst du kein Archiv. -- TZorn 15:55, 21. Mär. 2012 (CET)

Zum FAZ-Archiv: http://www.seiten.faz-archiv.de/faz/19950705/f19950705pearson100.html --Liesbeth 15:48, 21. Mär. 2012 (CET)

Danke auf den Hinweis auf „Bewirb dich für ein Jahr Highbeamzugriff“ - aber das dauert ja... Ich habe mich jedenfalls beworben. --Liesbeth 16:19, 21. Mär. 2012 (CET)

Solche Anfragen werden auf WP:BIBA meist schnell und unkompliziert erledigt. --тнояsтеn 16:28, 21. Mär. 2012 (CET)

Danke, das wusste ich nicht und habe mich dort gemeldet. Gruß --Liesbeth 16:47, 21. Mär. 2012 (CET)
Na, habe ich zu viel versprochen? Das könnte dich auch interessieren: http://www.deseretnews.com/article/428059/AVANT-GARDE-US-OPERA-SINGER-DIES-IN-GERMANY-AT-61.html --тнояsтеn 17:55, 21. Mär. 2012 (CET)
Nein, hast du nicht. Ganz großartig! Dank deines Tipps ist der Artikel jetzt ganz in Ordnung und belegt. Vielen herzlichen Dank, auch für den weiteren Nachruf. Gruß --Liesbeth 18:40, 21. Mär. 2012 (CET)
Archivierung dieses Abschnittes wurde gewünscht von: тнояsтеn 21:31, 21. Mär. 2012 (CET)

Huckle Buff

Weiß jemand, was Huckle Buff für ein Getränk ist, bzw. mit welchen Zutaten es gemixt wird? Ist scheinbar ein Drink aus dem 19. Jahrhunder, welcher aus England oder Wales kommt. -- 208.48.242.106 16:49, 21. Mär. 2012 (CET)

Gugelbux weiß es. --Rôtkæppchen68 16:53, 21. Mär. 2012 (CET)
Oh, danke! :) Mit der reinen Gugelsuche war ich nicht so weit gekommen. -- 208.48.242.106 16:55, 21. Mär. 2012 (CET)
Archivierung dieses Abschnittes wurde gewünscht von: тнояsтеn 21:33, 21. Mär. 2012 (CET)

Symbole auf einem Tachografenblatt

 
 

Kann mir jemand die Symbole auf diesen Tachografenblättern beschreiben also das "Bett", Hammer und Stössel, das Druchgestrichen Quadrat und das A. Bzw auf der Rückseite sind die noch mal zu finden und dann gibt es noch irgend welche e Nummern.--Sanandros (Diskussion) 07:17, 19. Mär. 2012 (CET)

<huuuust> Tachograph#Bedienung GEEZERnil nisi bene 07:30, 19. Mär. 2012 (CET)
Ah habe ich ganz übersehen, und die e Nummern auf der Rückseite?--Sanandros (Diskussion) 09:48, 19. Mär. 2012 (CET)
Siehe hier, Punkt 31. Onkel Google hats gefunden ;) --BlueCücü (Diskussion) 09:52, 19. Mär. 2012 (CET)
Schönes Dokument! Bereits im Artikel verlinkt? Wenn nicht ... gibt es hier barmherzige Seelen ...? GEEZERnil nisi bene 10:22, 19. Mär. 2012 (CET)
Also bei mir kamen`irgendwelche EWG Nummern und die E-Nummern der Lebensmittelzusatzstoffe, aber ihr wisst schon das Google die Ergebnisse individualisiert?--Sanandros (Diskussion) 11:33, 20. Mär. 2012 (CET)
Es gibt verschiedenste Tachographenblätter, z.B. bis 120km/h, bis 160km/h, ...etc. legt man beispielsweise ein 120km/h Blatt in einen Tachographen der Blätter bis 160km/h benötigt, so sind logischerweise die aufgezeichnete Geschwindigkeiten nicht richtig. Bei den E-Nummern gehts um den Typ des Tachographen, auf jedem Tachographen i.d.Regel auf dem Einschub, wo das Schaublatt eingelegt wird steht eine solche Nummer. Anhand dieser und der auf den Schaublättern aufgedruckten Nummern läßt sich überprüfen, ob das jeweilige Schaublatt zum Gerät paßt und verwendet werden kann bzw. darf. --Btr 14:40, 20. Mär. 2012 (CET)
OK vielen Dank--Sanandros (Diskussion) 20:00, 20. Mär. 2012 (CET)

Lichtwert und ISO in der Fotografie

Beim Artikel http://de.wikipedia.org/wiki/Lichtwert steht unter Lichtwert und Helligkeit: "Wenn bei einer Filmempfindlichkeit von ISO 100/21° eine Aufnahme mit LW 12 (zum Beispiel f/8, 1/60 s) zur korrekten Belichtung des Filmes führen würde, so müsste dasselbe Motiv bei derselben Helligkeit bei einer Filmempfindlichkeit von ISO 400/27° mit LW 14 aufgenommen werden."

Sollte das nicht LW 10 heißen?

--91.143.101.247 08:59, 19. Mär. 2012 (CET)

Frag am besten hier, hier oder hier nochmal nach. --BlueCücü (Diskussion) 09:25, 19. Mär. 2012 (CET)
Ich denke, das was da steht, stimmt hinten und vorne nicht. Mein Belichtungsmesser zeigt zunächst einmal einen Lichtwert an, und der ist völlig unabhängig davon, welche Empfindlichkeit eingestellt ist. Erst nach Einstellen der Empfindlichkeit ergibt sich dann eine oder mehrere Zeit/Blendenkombinationen. -- Gerd (Diskussion) 17:19, 19. Mär. 2012 (CET)
@91.143.101.247, ein Film mit 400 ISO ist empfindlicher als ein Film mit 100 ISO. Für dasselbe Motiv brauchst Du also weniger Licht. Bei gleicher Blende brauchst Du also eine kürzere Belichtungszeit. Gemäß der im Artikel Lichtwert gegebenen Formel ist der Lichtwert damit größer. Der Zusammenhang ist im Artikel IMHO korrekt dargestellt. --Rôtkæppchen68 17:58, 19. Mär. 2012 (CET)
Die Darstellung im Artikel ist richtig. Man kann es sich so klar machen:
  • Höherer Lichtwert bei gleichem Film: Hellere Szene, mehr Licht, höhere Dämpfung im Objektiv.
Bei empfindlicherem Film und gleicher Szene muss Dein Objektiv mehr dämpfen, also auf höheren Lichtwert gestellt werden.
Je heller das Objekt in Bezug auf die Filmempfindlichkeit ist, desto größer ist der einzustellende Lichtwert. --Pyrometer (Diskussion) 11:13, 20. Mär. 2012 (CET)

Unmittelbar vor dem vom Fragesteller zitierten Satz steht: Beispiel: "LW 12" bezeichnet keine bestimmte Helligkeit, "LW 12 bei ISO 100/21°" hingegen schon. Wie ich bereits weiter oben erwähnte, ist es meinem Belichtungsmesser völlig wurscht, welche Empfindlichkeit ich einstelle. Er zeigt immer densalben Lichtwert an, der ausschließlich von der Motivhelligkeit abhängig ist. Mit dem Artikel stimmt also irgendwas nicht. -- Gerd (Diskussion) 17:25, 20. Mär. 2012 (CET)

Genauso ist es, der Artikel ist falsch. Aber man darf ja nichts mehr korrigieren, ohne drölfzig Quellen dranzuhängen...--Marcela   17:30, 20. Mär. 2012 (CET)
 
ISO - Lichtwertsprung
Um auf die Ausgangsfrage zurückzukommen, aus meiner Sicht ist nicht erkennbar, was an der Artikelformulierung fehlerhaft sein soll. Ich habe den in Frage stehenden Text versucht, in der Tabelle nachzuvollziehen (Abb.) und finde das Ergebnis korrekt. Es deckt sich mit der praktischen Fotografiererfahrung, dass ich bei der Erhöhung der ISO an meiner Kamera von 100 auf 400 mit der Belichtungszeit von 1/60 auf 1/250 gehen oder auch die Blende von 8 auf 16 verringern kann. Dass dabei der Lichtwert gleich bleibt, kann man daraus schließen, dass es in allen diesen Fällen zu korrekt belichteten Fotos kommt - abseit aller abstrakten Überlegungen. Habe ich etwas übersehen? Grüße --losch (Diskussion) 20:05, 20. Mär. 2012 (CET)
Der Fehler ist, daß diese Tabelle ausschließlich für ISO 100/21° gilt. Man kann auf ihr also garkeine Belichtung für andere Empfindlichkeiten finden. Diese Tabelle stellt also ausschließlich die möglichen Zeit-/Blendenkombinationen für einige gängige Lichtwerte dar. Für andere Empfindlichkeiten brauchten wir andere Tabellen. -- Gerd (Diskussion) 08:45, 21. Mär. 2012 (CET)
In der Tabelle selbst sehe ich keine Festlegung auf einen ISO-Wert. In der Beschreibung wurde das lediglich beispielhaft auf ISO100 festgelegt. Die Tabelle würde sonst ja auch nur bei einer definiert festgelegten Motivhelligkeit Sinn machen. Insofern verstehe ich sie so, dass jeder LW mit jeder ISO bei passender Motivhelligkeit zutreffend sein kann und halte das für richtig. --losch (Diskussion) 09:13, 21. Mär. 2012 (CET)
Nochmal: Der Lichtwert ist die Motivhelligkeit, lediglich in das System von Blende, Belichtungszeit, und Empfindlichkeit übertragen. Und die Tabelle macht nicht bei einer definierten Motivhelligkeit Sinn, sondern bei einer definierten Film-/Sensorempfindlichkeit. Ich hab den Artikel derweil mit dem Überarbeiten-Baustein verziert. -- Gerd (Diskussion) 10:13, 21. Mär. 2012 (CET)
Irgendwie reden wir aneinander vorbei. Aus meiner Sicht ist es so, dass bei gegebener Motivhelligkeit (LW) entweder mit Blende, Verschlusszeit oder ISO-Empfindlichkeit eine Anpassung zur korrekten Belichtung vorgenommen werden kann. Richtig ist jedoch auch, dass bei fester ISO die Anpassung durch Blende, Verschlusszeit oder Motivhelligkeit (Scheinwerfer, Blitz...) erfolgen kann. Also können ALLE Werte variabel sein und nichts ist fix. Genau das lese ich auch aus dem Artikel heraus. Was müsste da deiner Meinung nach geändert werden? Grüße --losch (Diskussion) 10:36, 21. Mär. 2012 (CET)
Dem zweiten und dritten Satz Deiner letzten Aussage kann ich bedenkenlos zustimmen. Deiner Schlußfolgerung, dies stünde so im Artikel, jedoch nicht. Lies Dir doch einfach noch mal durch, was ich weiter oben geschrieben habe, dann muß ich es hier nicht nochmal wiederholen. Grüße -- Gerd (Diskussion) 11:20, 21. Mär. 2012 (CET)

Schließbare Fächer in Pörtschach Hauptbahnhof?

Weiß jemand, ob es in Pörtschach Hauptbahnhof in Kärtnen, Österreich schließbare Fächer gibt, da ich mein Gepäck hineinschließen könnte? JIP (Diskussion) 07:42, 20. Mär. 2012 (CET)

Vermutlich nicht: [6]. --Wrongfilter ... 07:55, 20. Mär. 2012 (CET)
In Klagenfurt Hauptbahnhof gibt es 33 Schließfächer [7]. --Pp.paul.4 (Diskussion) 09:50, 20. Mär. 2012 (CET)
Nach der letzten Bahnfahrt solltest du aber die neue Kamera mitnehmen :( --Incognito.ergo.possum (Diskussion) 20:48, 20. Mär. 2012 (CET)

GIF-Animationen - Funktion und Programme

Angenommen, in einer Animation bewegt sich eine Figur über einen Hintergrund von links (1. Bild) nach rechts (2. Bild). Um dies zu bewerkstelligen, sehe ich drei Methoden:

  1. ein komplettes neues 2. Bild wird über das 1. Bild gelegt
  2. das 2. Bild wird aufgebaut, indem das 1. Bild mit einem Ausschnitt des Hintergrundes überlagert wird, der gerade groß genug ist, um die Figur zu verdecken. Außerdem wird die Figur an der neuen Stelle dargestellt
  3. die Figur wird einfach an der alten Position gelöscht, der Hintergrund kommt wieder zum Vorschein. Außerdem wird die Figur an der neuen Stelle dargestellt

Methode 1 ist trivial. Methode 2 scheint mir die gängige Methode zu sein. Wenn ich die Beschreibung von GIMP richtig verstanden habe, dann arbeitet dessen Animationsoptimierung nach dieser Methode.

Aber was ist mit Methode 3? Hat das GIF-Format überhaupt die technischen Voraussetzungen, um diese Methode anzuwenden? Wenn ja, welche Programme können GIF-Animationen nach dieser Methode erstellen?

Letztendlich geht es mir um folgendes: ich habe einen Hintergrund H, drei Figuren A, B und C sowie 18 Figuren 1 bis 18. Die Einzelbilder sollen nun darstellen: H+A+1, H+B+2, H+C+3, H+A+4, H+B+5, H+C+6, H+A+7, H+B+8, H+C+9 usw. Nach Methode 2 müsste ich die Figuren A, B und C jeweils 6 Mal in der Gesamtdatei abspeichern, nach Methode 3 jedoch nur 1 Mal, was die Dateigröße stark reduzieren würde. --Plenz (Diskussion) 10:24, 20. Mär. 2012 (CET)

Ich wusste gar nicht, dass Animated GIF sowas Kompliziertes wie 2) beherrscht, aber du hast Recht. en:Graphics_Interchange_Format#Animated_GIF beschreibt das. Wenn du etwas Schlaueres haben willst, müsstest du aber wohl auf modernere Formate setzen. Animated GIF ist ein anscheinend nicht mal standardisiertes Verfahren aus frühen Netscape-Zeiten... --Eike (Diskussion) 10:34, 20. Mär. 2012 (CET)
Bei einem animierten GIF muss der Folge-Frame nicht das vollständige Bild überschreiben; es kann auch ein kleinerer (rechteckiger!) Ausschnitt definiert werden, der neu dargestellt wird; der Rest bleibt somit aus dem vorhergehenden Frame sichtbar. Einzelne Figuren können aber nicht abgelegt und nach Bedarf ein- und ausgeblendet werden. Es handelt sich somit um eine Modifizierung von Methode 1; für Methode 2 müssten zwei Bereiche definiert und überschrieben werden, wofür GIF aber entweder zwei Schritte (= 2 Frames) oder ein entsprechend größeres Rechteck zum Überschreiben beider Bereiche benötigen würde. --Snevern 10:43, 20. Mär. 2012 (CET)
Sehr guter Hinweis, danke! Wenn also z.B. die Figuren B und 2 weit auseinanderliegen, würde das eine riesige Fläche bedeuten, und es wäre demnach günstiger, jeder der beiden Figuren ein Extrabild zu spendieren mit 0 ms Verzögerung dazwischen. --Plenz (Diskussion) 16:33, 20. Mär. 2012 (CET)
Variante drei dürfte nicht möglich sein, weil GIF keine Layers hat. Wenn du eine Figur auf einen Hintergrund setzt, dann geht der Hintergrund dort in diesem Frame verloren. --Hareinhardt (Diskussion) 10:49, 20. Mär. 2012 (CET)
Um es noch deutlicher zu sagen: Animated GIF ist eine Krücke aus dem Internet-Kreidezeit. Leider bilden sich die Alternativen erst langsam heraus. HTML 5 und H.264 scheinen da vielversprechend zu sein. --Eike (Diskussion) 11:01, 20. Mär. 2012 (CET)
@Plenz, Du kannst bei GIF auch binäre Transparenz nutzen und so in den einzelnen Frames nur die Differenz zum vorherigen Frame neu zeichnen und den Rest transparent lassen. Gute GIF-Animations-Programme übernehmen sogar die Optimierung für Dich. --Rôtkæppchen68 11:37, 20. Mär. 2012 (CET)
Du könntest auch deine Figuren A, B, C und 1-18 je als animiertes GIF mit transparentem Drumherum machen und diese über ein separates Hintergrundbild H legen (z.B. auf einer Webseite mit Hilfe von Javascript). Dann hast du insgesamt nur 22 Dateien und volle Flexibilität. Allerdings nicht garantiert, dass alle Browser das auch richtig anzeigen. --Neitram 11:59, 20. Mär. 2012 (CET)
Würde ich nur unter bestimmten Umständen empfehlen. Bei einem GIF kennt der Alpha-Kanal (meines Wissens) nur 1 und 0. Das Problem ist dann, dass du z.B. an den Rändern deiner Figuren nicht mit 50%, 30% oder 20% (usw.) Transparenz arbeiten kannst. Möglicherweise hat man das Problem auch bei Methode 2... --Maxkhl (Diskussion) 13:54, 20. Mär. 2012 (CET)
Die teilweise Transparenz (Alphakanal) muss im GIF-Format selbstverständlich schon beim Rendern berücksichtigt. Bei der Codierung und Optimierung der GIF-Datei ist nur binäre Transparenz möglich, die halbtransparenten Pixel müssen also dem opaken Vordergrund zugerechnet werden. --Rôtkæppchen68 15:40, 20. Mär. 2012 (CET)

Danke für alle Antworten. Dass es Alternativen zu GIF gibt, ist schon klar, aber mir geht es um die Erstellung einer Datei, die ich bei Wikipedia hochladen kann. Auf einer eigenen Webseite kann ich natürlich ganz andere Dinge machen. Das mit den binären Transparenzen klingt interessant, aber bringt das eine deutliche Dateiverkleinerung? Und welche Programme beherrschen diese Optimierung? --Plenz (Diskussion) 16:29, 20. Mär. 2012 (CET)

Die GIF-Erstellung ist bei mir eine ganze Weile her, deshalb nenne ich die Programme nur unter Vorbehalt: Das ist der Ulead GIF Animator und Jasc Animation Shop, der bei Paint Shop Pro 5 mit dabei war. Möglicherweise gibt es längst bessere Programme oder freie Alternativen. --Rôtkæppchen68 19:38, 20. Mär. 2012 (CET)
.svg kann man bei Wikipedia hochladen. .svg kann man animieren (über den Internet Explorer reden wir nicht). --88.130.132.26 22:46, 20. Mär. 2012 (CET)
Ich habe schon mit dem "GIF Animator LE" (erhältlich bei den Downloads von ct.de) rumgespielt und schöne Ergebnisse bekommen. --PeterFrankfurt (Diskussion) 03:23, 21. Mär. 2012 (CET)

Dienstkleidung

Ist es strafbar, eine (nicht zu kaufende) Dienstkleidung eines Unternehmens zu besitzen? Ist es strafbar, sich als ein Angehöriger des Unternehmens auszugeben? (ich meine keine hoheitlichen Stelle wie Polizei, Zoll, Feuerwehr) --85.180.145.232 13:03, 20. Mär. 2012 (CET)

1. Würde ich jetzt eher mal verneinen. 2. Kann durch aus sein, dass es strafbar ist. Kommt immer darauf an was man mit „sich als ein Angehöriger des Unternehmens auszugeben“ erreichen will. Die dabei ausgeübten Handlung können dann durchaus als strafbar angesehen werden. Z.B. Erschleichung einer Leistung, sich verschaffen des Zutritt zu einem nicht öffentlichen Bereich usw.. Da gibt es einiges an strafbare Handlungen, die durch Dienstkleider möglich sind. Es kommt hier in erster Linie darauf an, ob damit jemand geschädigt wird, bzw,. geschädigt werden könnte.
Sich an einer privaten Party als Pilot auszugeben, unter anderem in dem man ein originalen Pilotenanzug trägt ist aber sicher nicht strafbar. --Bobo11 (Diskussion) 13:12, 20. Mär. 2012 (CET)
Man darf sich aber keine Dienstkleidung der lokalen Stadtwerke anziehen und sich dann bei alten Omas als falscher Gasmann ausgeben. --El bes (Diskussion) 13:41, 20. Mär. 2012 (CET)
Bzgl. des Pilotenanzugs kommt es darauf an: Wer auf einer Party zB unbefugt die Uniform eines Bundeswehrpiloten trägt, macht sich nach § 132a StGB strafbar. Da muss dann, anders als beim bloßen benutzen des Blaumanns der Stadtwerke, auch nichts anderes (zB eine Täuschungsabsicht mit dem Ziel einer Vermögensverfügung) dazu kommen. -- Ian Dury Hit me  13:56, 20. Mär. 2012 (CET)
Wenn es die Dienstkleidung einer staatlichen Organisation ist, ist es Amtsanmaßung (z.B.: Polizei, Militär, ...). Ich schätz, auch wenn du beispielsweise auf einer Großveranstaltung im Sanitäterkostüm rumrennst obwohl du keine entsprechende Ausbildung aufweisen kannst, gibt das mächtig Ärger, weil wenn jemand im Notfall zuerst zu dir kommt, du daran schuld wärest, dass sich die Zeit bis zur Erstversorgung durch einen Sanitäter erhöht. -- 208.48.242.106 14:02, 20. Mär. 2012 (CET)
Du wirst bitte keine DB-Uniform erwerben und Schaffner spielen. Und das Mobile Terminal und den Zangendrucker gibst Du bitte auch zurück. -- 188.105.53.57 14:11, 20. Mär. 2012 (CET)
Nicht?! Och menno! -.-
Alternativ darf man natürlich gern die Dienskleidung der Stadtreinigung erwerben und Beete und Gehwege von Müll befreien. -- 208.48.242.106 14:29, 20. Mär. 2012 (CET)

Das Führen von (geschützten) Marken ist als Antragsdelikt strafbar. --Marcela   17:32, 20. Mär. 2012 (CET)

Es ging in der Fragestellung ausdrücklich nicht um hoheitliche Uniformen, sondern um Dienstkleidung von Privatfirmen. Und der Erwerb solcher Kleidungsstücke ist für sich genommen in Deutschland ganz klar nicht strafbar. Das heißt nicht zwangsläufig, dass es erlaubt ist, aber es ist eben keine Straftat.
Sich als Angehöriger eines solchen Privatunternehmens auszugeben, unter Zuhilfenahme der unbefugt (und damit möglichweise widerrechtlich) erworbenen Dienstkleidung, ist für sich allein genommen auch noch keine Straftat. Es gibt aber eigentlich nur sehr wenige denkbare Handlungen, die man auf diese Weise begehen kann, die nicht auf die eine oder andere Weise gegen Rechte des betroffenen Unternehmens verstoßen und damit zumindest rechtswidrig sind (Unterlassungs- und/oder Schadensersatzansprüche des Unternehmens = Zivilrecht). Ohne weitere Tatbestandsmerkmale ist aber auch das noch keine Straftat (= Strafrecht). --Snevern 19:28, 20. Mär. 2012 (CET)
Straftat und strafbar sind alles andere als identisch. Ob man etwas legal oder illegal erworben hat ist unerheblich. Das führen von registrierten Marken ist verboten. Es wird allerdings nur auf Antrag verfolgt. Wenn ich mir das Logo eines bekannten Colaherstellers auf die Stirn male und nackt durch die Stadt laufe, kann mich die Firma belangen. Wenn ich mir das Logo eines Autoherstellers aus der Wikipedia herunterlade und auf ein Shirt drucke und damit etwas mache, was der Firma nicht gefällt, dann kann sie mich belangen. Wenn ich offiziell ein Merchandisingtrikot eines Fußballvereins gekauft habe und damit betrunken Passanten anpöble, kann mich der Verein belangen. Ob das in solchen Fällen geschieht, steht auf einem anderen Blatt. --Marcela   20:15, 20. Mär. 2012 (CET)
Der Unterschied zwischen "Straftat" und "strafbar" ist in erster Linie grammatikalischer Natur: Das eine ist ein Substantiv, das eine Handlung beschreibt, das andere ein Adjektiv, das sich auf eben diese Handlung bezieht. Der Jurist unterscheidet natürlich noch zwischen der Erfüllung des Tatbestands mit allen seinen (objektiven und subjektiven) Merkmalen und der tatsächlichen Strafbarkeit, die zusätzlich insbesondere das Vorliegen von Rechtswidrigkeit und Schuld erfordert (also das Fehlen von Rechtfertigungsgründen und Schuldausschließungsgründen) - aber darauf wolltest du vermutlich nicht heraus. Falls du auf den Unterschied zwischen verboten einerseits und strafbar andererseits hinauswolltest: Das habe ich oben schon versucht darzustellen (ist mir scheinbar misslungen).
Sagst du uns noch, nach welcher Vorschrift es (auf Antrag) strafbar ist, mit einem legal erworbenen Fussballtrikot Passanten anzupöbeln oder mit einem auf die Stirn gemalten Cola-Emblem nackt durch die Stadt zu laufen (das Anpöbeln bzw. nackt durch die Stadt laufen an sich ist ja nicht gemeint - es geht ausschließlich um den Missbrauch der geschützten Marken und insbesondere die Strafbarkeit dieses Missbrauchs)? --Snevern 21:57, 20. Mär. 2012 (CET)
§ 14 MarkenG --Marcela   22:41, 20. Mär. 2012 (CET)
Das ist keine Strafnorm, sondern eine Verbotsnorm (eine Strafnorm wäre § 143, aber die ist hier genausowenig einschlägig).
Verboten sind nach der von dir genannten Norm bestimmte Handlungen im geschäftlichen Verkehr. Die von dir geschilderten Fälle haben damit nichts zu tun. --Snevern 22:49, 20. Mär. 2012 (CET)

Wer weiß was zu... ?

Hallo, wer weiß was zu dieser Steinplatte. Zu finden am Strassen-rand in Niederschlesien.

http://bystrzyca.info.pl/forum/files/p1010356_892.jpg

Danke. (nicht signierter Beitrag von Skarabeusz84 (Diskussion | Beiträge) 19:00, 20. Mär. 2012 (CET))

Das sieht nach einem Gedenkstein aus. Entweder ist dort jemand umgekommen, oder jemand hat das aus einem Gelübde heraus dort aufgestellt. Näheres kann man ohne genaue Ortsangabe nicht sagen. --Sr. F (Diskussion) 20:11, 20. Mär. 2012 (CET)
+1, siehe Sühnestein. --Concord (Diskussion) 01:01, 21. Mär. 2012 (CET)

Abschluss für PVC-Boden

Ich möchte in einer "amerikanischen Küche", also nur in einem Teil eines Zimmers, PVC-Boden über dem Laminat verlegen. Für den Abschluss gibt es Schienen, die man verschrauben kann. Ich würde aber lieber den Boden unversehrt lassen und zum Beispiel eine Gummi-Kante kaufen, die man über die PVC-Kante stülpt und vielleicht noch am Boden festklebt. Gibt es sowas? --87.170.107.36 19:42, 20. Mär. 2012 (CET)

bei mir wurde das mit silikon aus einer kartusche gemacht, nachdem der belag am estrich(?) festgeklebt wurde... --Heimschützenzentrum (?) 20:09, 20. Mär. 2012 (CET)
Klar gibbet sowas, hat jeder bessere Baumarkt, in Weich-PVC oder Gummi. -- Janka (Diskussion) 20:16, 20. Mär. 2012 (CET)
Meine Studibude hatte auch so eine Küche. Das Übergangsprofil war aus Kunststoff, mit dem Messer leicht zuschneidbar (auch auf Gehrung) sehr robust und hielt alles aus :o). Sah ähnlich aus wie das PVC-Profil auf dieser Seite (das zweite von unten) und hatte den Vorteil, dass es elegant Teppichboden und PVC verband und mit großer Fläche auf dem Boden klebte (im Gegensatz zu dem darunter). -- Ian Dury Hit me  21:45, 20. Mär. 2012 (CET)

Neodym-Pflanzenlampen

Lassen diese Neodym-Lampen die Farben von Pflanzen nur besser aussehen, oder haben die auch wachstumsfördernde Eigenschaften? -- 92.226.209.198 22:00, 20. Mär. 2012 (CET)

Diese Lampen haben ein blau eingefärbtes Glas, wodurch der Rotanteil gegenüber einer normalen Glühlampe stark verringert wird. Dadurch erscheint das Licht kälter, näher am Tageslicht. Gegenüber normalem Glühlampenlicht selber Leistung ist das nicht wachstumsfördernder, nur halt besser für die Farbwiedergabe. Wachstumsfördernd ist immer nur mehr Licht als die Pflanze in der dusteren Wohnung ohne Zusatzlicht kriegen würde. Pflanzen gehören ja eigentlich ins Freie, und die Lichtstärke ist da locker 100 bis 10000 mal so hoch wie bei gleicher Wetterlage im Zimmer.
Jetzt stellt sich natürlich die Frage: Wie kriege ich das Zimmer so hell wie draußen, ohne dass es mir ein Loch in den Geldbeutel brennt. Und da sind Glühlampen eine schlechte Wahl. Die meisten Pflanzen stehen auf blaues und rotes Licht. Grünes Licht wird reflektiert bzw. durchgelassen, das kann die Pflanze nicht nutzen. Wenn man eine Glühlampe als Basis nimmt, ist es aber sowieso wumpe welche Lichtfarbe man nimmt, denn die Glühlampe erzeugt grundsätzlich immer das ganze Spektrum. Ein Beschichtung in der Lampe würde also nur das Filtern vorwegnehmen, das die Pflanze sowieso vornimmt. Man hat also Gegenüber einer normalen Glühlampe keine Vorteile - außer vielleicht, dass die Pflanzen "hell" stehen, während man selbst das Zimmer als relativ dunkel empfindet - schließlich sind unsere Augen ja besonders Grünempfindlich, weil wir ja hauptsächlich das ganze grüne, von den Pflanzen ungenutze Licht zum Sehen verwenden.
Nützlich sind spezielle Pflanzlichtlampen nur dann, wenn die Lampe ohnehin nur die benötigten Farben, also rot und blau, erzeugt. Das geht mit Leuchstoffröhren (und ESL) mit passenden Leuchtstoffen, mit Hochdrucklampen und mit LEDs. Da spart man sich die Energie für die Grünerzeugung. Außerdem werden solche Lampen nicht so heiß, wodurch man näher an die Pflanzen herangehen kann ohne sie zu quälen, im Effekt weniger Streulicht und mehr Licht an die Pflanzen, da wo es hinsoll. -- Janka (Diskussion) 02:14, 21. Mär. 2012 (CET)

Bahnsteigen 2 und 3b bei Klagenfurt Hauptbahnhof

Weiß jemand, ob die Bahnsteigen 2 und 3b bei Klagenfurt Hauptbahnhof in Kärtnen, Österreich genau neben einander sind, so man von einen Bahnsteige zu den Anderen nur kurz durchgehen braucht, ohne Treppe, Fahrstühle oder so was nutzen zu brauchen? JIP (Diskussion) 22:03, 20. Mär. 2012 (CET)

Antwort ist hier --91.2.125.225 22:07, 20. Mär. 2012 (CET)
Ja, die Gleise 2 und 3 liegen am selben Mittelbahnsteig. Auf Webseite [8] unter «Barrierefreie Bahnhofsausstattung» ist unter Bemerkungen die Bahnanlage erklärt. Und Nein @IP, dass steht eben nicht deutlich im Artikel. (Gut JETZT -nach meiner Bearbeitung- schon)---Bobo11 (Diskussion) 22:12, 20. Mär. 2012 (CET)
Hat die IP doch auch gar nicht behauptet. --тнояsтеn 22:19, 20. Mär. 2012 (CET)
Gut! Dann scheint es möglich zu sein, von ein Zug an den Bahnsteig 3b zu ein Anderen an den Bahnsteig 2 in 13 Minuten zu wechseln, wenn der erste Zug nicht zu spät ankommt. Und weil er ein S1-Regionalzug von Pörtschach nach Klagenfurt ist, glaube ich, das er nicht zu spät ankommen wird. JIP (Diskussion) 22:26, 20. Mär. 2012 (CET)
Ja, 13 Minuten sollten in dem Bahnhof reichen. Selbst bei Bahnsteigwechsel mit zweimaliger Liftbenutzung. Kurzum auch für jemand der in seiner Mobilität eingeschränkt ist, sollte das Umsteigen innerhalb 13 Minuten in Klagenfurt machbar sein.--Bobo11 (Diskussion) 22:45, 20. Mär. 2012 (CET)

Landtagsauflösung NRW 2012

Hallöchen! Kleine Denksportaufgabe an die Verfassungsjuristen:

Nach diesem „lustigen“ Bericht von RP Online war die Auflösung des NRW-Landtags nach der zweiten Lesung wohl verfassungswidrig, es hätte eine dritte Lesung geben müssen – meint jedenfalls dieser Herr. Und auch der Staatsrechtslehrer Ralph Alexander Lorz haut gegenüber der FAZ in dieselbe Kerbe. Offensichtlich hat die Landtagsverwaltung da wohl etwas verwechselt, dass in NRW andere Regelungen gelten als im Bund.

Aber mal gesetzt den Fall, die Auflösung war verfassungswidrig, wie legal sind dann überhaupt die kommenden Wahlen? Und wie legal ist der solchermaßen gewählte Landtag? Kann ein daraus hervorgegangener Landtag überhaupt gültige Beschlüsse fassen? Welchen Wert haben Gesetze, die ein solcher Landtag beschließt? --Duschgeldrache2 (Diskussion) 01:51, 21. Mär. 2012 (CET)

Nein, der Artikel der RP wurde völlig falsch verstanden. Von Verfassungswidrigkeit des Auflösungsbeschlusses ist dort keine Rede, sondern bloß davon, dass die ihm zugrunde liegende Annahme fragwürdig sei, der Haushalt sei mit Ablehnung einer Position in zweiter Lesung bereits insgesamt unrettbar gescheitert. Das mag so sein oder auch nicht. Jedenfalls wäre ein entsprechender Irrtum der Parlamentarier unbeachtlich. Art. 35 der Landesverfassung bestimmt ganz schlicht und ohne jedes wenn und aber: "Der Landtag kann sich durch eigenen Beschluß auflösen. Hierzu bedarf es der Zustimmung der Mehrheit der gesetzlichen Mitgliederzahl." Für die Motive der Parlamentarier interessiert sich die Verfassung nicht. Da die Verfassung außerdem bestimmt: "Nach der Auflösung des Landtags muß die Neuwahl binnen sechzig Tagen stattfinden", ließe sich der Auflösungsbeschluss auch nicht einfach rückgängig machen, sondern allenfalls durch ein verfassungsänderndes Gesetz. Von Verfassungs wegen ist also alles in Ordnung. Selbst wenn die Auflösung des Landtags nicht zwingend geboten war, sind die Wahlentscheidungen in den daraufhin stattfindenden Neuwahlen selbstverständlich wirksam und zur Legitimation des neuen Landtags ausreichend. --Vsop (Diskussion) 02:25, 21. Mär. 2012 (CET)
Du hast in der Sache zweifellos Recht, Vsop, aber der Artikel wurde keineswegs falsch verstanden: Im Artikel wurde der Sachverhalt genau so dargestellt, wie Duschgeldrache ihn hier wiedergegeben hat. Ich weiß nicht, welcher Partei Morlok nahe steht - aber sicher ist es nicht die SPD. --Snevern 06:42, 21. Mär. 2012 (CET)
Hier vertritt er jedenfalls SPD und Grüne, so strenger Gegner kann er nicht sein. --Eike (Diskussion) 08:52, 21. Mär. 2012 (CET)
Dann entschuldige ich mich; vielleicht ist er ja wirklich mehr am juristischen Procedere interessiert als am Erfolg einer konkreten Partei. --Snevern 09:06, 21. Mär. 2012 (CET)

Rechtschreibung

Wie schreibt mans richtig? (1) Rock’n’Roller, (2) Rock ’n’ Roller, (3) Rock-’n’-Roller oder (4) gar nicht weils das Wort nicht gibt? Danke --stfn (Diskussion) 10:50, 21. Mär. 2012 (CET)

WP schreibt es so... GEEZERnil nisi bene 10:52, 21. Mär. 2012 (CET)
Duden scheint es nicht zu kennen ... Gruß Thogru Sprich zu mir! 10:54, 21. Mär. 2012 (CET)
Wenn ich davon ausgehe, dass der Aphostroph einen Buchstaben ersetzt, dann wird aus Rock and Roller ein Rock 'n' Roller. --Shaun72 (Diskussion) 11:01, 21. Mär. 2012 (CET)
Dann gibts noch das hier: Rock-n-Roller. Gruß Thogru Sprich zu mir! 11:06, 21. Mär. 2012 (CET)
So wie auch Jump ’n’ Run -- 208.48.242.106 11:11, 21. Mär. 2012 (CET)
@Geezer: WP schreibt halt so allerlei, daher meine Frage.
Ja, und in Büchern auch.... Eins ist sicher: Ein echter Rock 'n' Roller würde sich mit Stiefeln und Gel gegen Binde(!)striche wehren. GEEZERnil nisi bene 13:16, 21. Mär. 2012 (CET)
@Shaun: Das war auch meine Überlegung.
@Thogru: Ja klar, gibt aber auch Rock N Rolla :)
@208.48.242.106: Klar (analog zu Rock ’n’ Roll), aber demnach dann auch Jump ’n’ Runner?
@alle: Ich tendiere dann mal zu (2). Eine passende Regel wär mir trotzdem genehm. Danke und schöne Grüße --stfn (Diskussion) 11:58, 21. Mär. 2012 (CET)
Tippe ich richtig, wenn ich sage, dass das -er ein Nominalisierungssuffix ist? (Obwohl Rock 'n' Roll ja auch schon ein Nomen ist) Auf jeden Fall ein Suffix. Und durch anhängen des selbigen sollte sich doch am Grundwort nichts verändern, oder? -- 208.48.242.106 12:05, 21. Mär. 2012 (CET) Theoretisch müsste es sowieso Rocker und Roller heißen. Wenn ich mal von Triathlon ausgehe, dann sind die Disziplinen ja Lauf, Radfahrt und Schwimmen. Die Ausführende Person ist ja dann auch Läufer, Radfaher und Schwimmer und nicht Lauf, Radfahrt und Schwimmer. Obwohl ... immerhin ist Rock 'n' Roll ja schon fast sowas wie ein eigenständiges Wort.
Nehme an, dass es ein Suffix ist (aus dem Artikel werde ich nicht recht schlau), analog zu Rocker, Punker oder Blueser. --stfn (Diskussion) 12:50, 21. Mär. 2012 (CET)

Fn-Taste dauerhaft aktiviert

Auf meinem Samsung-Notebook ist die FN-Taste dauerhaft aktiviert. Eine NumLock-Taste gibt es auf der Tastatur gar nicht. Was muss ich tun, um das umzuschalten? Ich kann dies nur tippen, solang ich mit dem linken Zeigefinger die Fn-Taste gedrückt halte. Grüße --Mautpreller (Diskussion) 09:35, 22. Mär. 2012 (CET)

Fn dauerhaft aktiviert? Bist du dir sicher? das würde ja bedeuten, dass du bei allen Tasten mit Fn-Zweit- oder -Drittbelegung du nur noch auf die Fn-Belegung zugreifen könntest, und nicht mehr auf die eigentliche Belegung mit Buchstaben, F-Tasten, Ziffern, etc. -- 208.48.242.106 09:54, 22. Mär. 2012 (CET)

a, gena4 das 5st der Fa33, (normal getippt). Ja, genau das ist der Fall (mit gehaltener Fn-Taste getippt).--Mautpreller (Diskussion) 10:20, 22. Mär. 2012 (CET)

Neuen Tastaturtreiber drüberbügeln? wäre das erste was mir einfällt. Oder: Es sit eine Einrastfunktion, wie bei der Umschalttaste, wo nach 8 Sekunden die anschlagverzögerung aktiviert wird. Wenn du Fn längere Zeit einzeln drückst, geht es dann weg?
Hier heißt es: „lösung fn+f11“. Also doch eine Feststellfunktion. -- 208.48.242.106 10:29, 22. Mär. 2012 (CET)
Ich kenns von meinem Dell Notebook. Da gehts aber nur um die F1-F12 Tasten. Dell ist da der Meinung, dass man letztere seltener braucht und hat das ganze umgedreht. Ich müsste also FN drücken um die F1-F12 Tasten zu verwenden. Da gabs aber eine Einstellung im Bios und beim Tippen hat das auch nicht gestört. Mir wäre sowieso neu, dass eine gedrückte FN Taste beim Tippen der Buchstaben stört... --Maxkhl (Diskussion) 10:34, 22. Mär. 2012 (CET)
Quetsch: Ja, tut sie, wenn auf dem Buchstabenblock der Fn-Ziffernblock liegt. Und warum soll man die F1-F12 tasten selten benutzen? Ich drück die Dinger rund um die Uhr, liegen viel zu viel nützliche Funktionen darauf um es zu vernachlässigen-- 208.48.242.106 10:40, 22. Mär. 2012 (CET)
So haben die bei Samsung also das Platzproblem mit dem Ziffernblock gelöst. Bei meinem Dell hab ich einfach keinen... Mit den F Tasten gehts mir auch so. Aber komischerweise ist Dell da anderer Meinung. Keine Ahnung wie es jetzt ist aber vor 1-2 Jahren haben die das so gemacht. --Maxkhl (Diskussion) 15:24, 22. Mär. 2012 (CET)

Fn+F11 hat geklappt. Danke!--Mautpreller (Diskussion) 10:36, 22. Mär. 2012 (CET)

Archivierung dieses Abschnittes wurde gewünscht von: --Eike (Diskussion) 10:51, 22. Mär. 2012 (CET)

YIT

Wie spricht man YIT aus? Englisch? (Es geht um die YIT Germany, also bezogen auf den deutschen Sprachraum) --тнояsтеn 09:55, 22. Mär. 2012 (CET)

Yleinen insinöörtoimisto

Also, englisch ( wei-ai-ti ) würde heutzutage sicher akzeptiert werden. Ich würde einfach eine Mail schreiben und fragen, hab ich ihn einen ähnlichen Fall schon gemacht, da es ja keine blöden Fragen, nur Antworten gibt. --RobTorgel (Diskussion) 11:13, 22. Mär. 2012 (CET)

+1 Die Leute, die dort arbeiten sprechen es "wei ei ti" aus (mit eigenen Ohren gehört). Nur in Finnland (Hauptfirmensitz) sagt man wohl "Jitt". --Optimum (Diskussion) 12:16, 22. Mär. 2012 (CET)

finnisch y entspricht lautlich aber deutsch "ü", nicht "j", also eher üü-ii-tee. --Janneman (Diskussion) 13:15, 22. Mär. 2012 (CET)
Danke euch. Mit der Bestätigung von Optimum gebe ich mich voll und ganz zufrieden. --тнояsтеn 19:47, 22. Mär. 2012 (CET)
Archivierung dieses Abschnittes wurde gewünscht von: тнояsтеn 19:47, 22. Mär. 2012 (CET)

Eurojackpot - Ziehung schauen

Heute wird das erste Mal der Eurojackpot in Helsinki gezogen. Gibt es eine Möglichkeit, dass im Internet oder Fernsehen Live zu sehen? Ich hab bisher keine Möglichkeit gefunden. --94.134.211.95 14:13, 23. Mär. 2012 (CET)

Gleich der erste Treffer bei Google sagt: Nein. Gruss --Nightflyer (Diskussion) 14:33, 23. Mär. 2012 (CET)
Danke, es gibt einfach zu viele Seiten dazu. --94.134.211.95 (14:58, 23. Mär. 2012 (CET), Datum/Uhrzeit nachträglich eingefügt, siehe Hilfe:Signatur)
Archivierung dieses Abschnittes wurde gewünscht von: тнояsтеn 15:01, 23. Mär. 2012 (CET)

Ruhrkinderverschickung ca. 1920

Ab wann wurden Ruhrkinder wegen der schlechten Versorgungslage in Pflegefamilien anderer Länder geschickt (Mein Vater war in einer Thüringer Familie ca. ab 1920)?

--94.222.47.27 16:41, 19. Mär. 2012 (CET)

Suchwort ist Kinderlandverschickung - ab 1916. GEEZERnil nisi bene 17:48, 19. Mär. 2012 (CET)
Nach einer schlecht mit Lupe (aber kostenlos) lesbaren "Preview" Springerlink offiziell seit der Ruhrbesetzung (also Anfang 1923} einzelne caritative Einrichtungen aber schon seit 1916 , also mitten im Krieg. --G-Michel-Hürth (Diskussion) 18:01, 19. Mär. 2012 (CET)
Einfach auf die Vorschauseite klicken, dann brauchst du auch keine Lupe. --тнояsтеn 21:11, 21. Mär. 2012 (CET)

Beleidigt sein, als Geschäftsmodell

Nicht das ich dies vor hätte, aber wir diskutieren gerade zum Spaß ob dies möglich wäre. Beleidigungen sind strafbar und werden meist mit Geldstrafen geahndet. Auf Youtube zum Beispiel braucht man sich zu bestimmten Themen nur vage kritisch zu äußern und man wird garantiert mit übelsten Beleidigungen überschüttet. Jetzt die Frage. Bekommt man als Beleidigter die Summe die der Beleidiger als Strafe zahlen muss? Oder bekommt man überhaupt nur dann "Schmerzensgeld" wenn man zivil klagt? --81.200.198.20 12:11, 20. Mär. 2012 (CET)

Probier es doch einfach aus, Du dummer Troll. --91.56.187.87 12:32, 20. Mär. 2012 (CET)
Eine etwaige Geldstrafe eines Strafverfahrens geht nicht an den Beleidigten, und ein Schmerzensgeld gibt's wohl eher gar nicht. --Eike (Diskussion) 12:38, 20. Mär. 2012 (CET)
Do you speak English and are you willing to travel? Buckle up! Big buck ahead! Call 1-800-.... Nothing new under my sun. GEEZERnil nisi bene 12:50, 20. Mär. 2012 (CET)
Also um die Frage unter Beachtung von WP:AGF zu beantworten: Nein, die Geldstrafe geht nicht an den Geschädigten (das Opfer der Straftat), sondern an die Staatskasse oder an gemeinnützige Einrichtungen. Und ein Schmerzensgeld bzw. ein Schadensersatz könnte theoretisch auf dem Zivilrechtswege eingeklagt werden, aber wer die Reaktionen selbst provoziert hat, wie du das da oben beschrieben hast, der wird nur wenig Chancen haben - selbst wenn es ihm gelingen sollte, die Täter überhaupt namhaft zu machen. --Snevern 13:24, 20. Mär. 2012 (CET)
Ich hab beim Überfliegen der Artikel Geldstrafe (Deutschland), Geldbuße, Geldauflage, ... wenig darüber gefunden, an wen das Geld geht. Wovon hängt das ab? --Eike (Diskussion) 13:28, 20. Mär. 2012 (CET)
Es kommt darauf an, wer wem und wie auf die Zehen tritt ... aber 6000 € sind doch peanuts, oder? GEEZERnil nisi bene 13:44, 20. Mär. 2012 (CET)
Das ist eine zivilrechtliche Verurteilung, erkennbar zum Beispiel am Aktenzeichen und an der Bezeichnung der Beteiligten. Da fließt das Geld natürlich weder dem Staat noch einer gemeinnützigen Einrichtung zu, sondern dem Kläger. --Snevern 19:37, 20. Mär. 2012 (CET)
Es fragte doch nach Zivilklage. Aber das TV-Beispiel zeigt, dass in D selbst unter "optimalen" Bedingungen nicht viel zu holen ist. Deshalb mein Vorschlag, im Alles-möglichen-Ausland nach Fleischtöpfen zu suchen - oder in D Bundespräsident zu werden ... GEEZERnil nisi bene 07:18, 21. Mär. 2012 (CET):
Geldstrafen und Bußgelder fließen grundsätzlich der Staatskasse zu. Eine Verfahrenseinstellung kann dagegen von Weisungen und Auflagen abhängig gemacht werden, und da ist im Gesetz (u.a.) ausdrücklich die Möglichkeit genannt, einen Geldbetrag zugunsten einer gemeinnützigen Einrichtung zu zahlen. Die meisten Richter entscheiden hier nach eigenem Gutdünken, wem das Geld zufließt - zum Beispiel an eine Einrichtung, die sich um Opfer einschlägiger Straftaten kümmert. --Snevern 19:33, 20. Mär. 2012 (CET)
Denkbar wäre, (zivilrechtliche) Ansprüche im Adhäsionsverfahren geltend zu machen. Abgesehen davon, dass Strafrichter auch heute noch mit einem solchen Ansinnen oft überfordert sind, fällt das zugesprochene Schmerzensgeld nach meinen Beobachtungen oft niedriger aus, als vor dem Zivilgericht (was womöglich daran liegt, dass Strafrichtern die Abgründe menschlichen Handelns weniger fremd sind). -- Ian Dury Hit me  20:20, 21. Mär. 2012 (CET)
Ja, denkbar und möglich - aber nach meiner Einschätzung eine Ausnahmeerscheinung. Und zwar keineswegs nur deshalb, weil das Gericht eine Entscheidung ablehnen könnte, wie es in unserem Artikel dargestellt ist. Wenn deine Beobachtung zutreffen sollte, wäre das noch ein zusätzlicher Grund: Richter, Staatsanwälte und Anwälte mögen das Adhäsionsverfahren schon nicht - das wäre auch noch ein Grund für den Verletzten, es nicht zu mögen. --Snevern 21:52, 21. Mär. 2012 (CET)

Termineinladung mit Thunderbird

Hallo, da ich im Netz nichts gefunden habe, versuche ich es mal hier. Ich verwende Mozilla Thunderbird mit der Lightning-Erweiterung. Dort kann ich bei Terminen weitere Teilnehmer einladen und diese erhalten die Termineinladung per Mail. Soweit so gut. Aber wo finde ich die Termineinladungen? Im Order "Gesendet" sind sie nicht.
Es ist nämlich schon vorgekommen, dass die Einladung nicht verschickt werden konnte (Netzwerkverbindung nicht aktiv o.ä.), dann bringt Thunderbird zwar eine Fehlermeldung, aber ich weiß nicht, wie ich die Einladung nochmal verschicken kann (denn sie ist auch nicht im Ordner "Postausgang"). --89.204.154.169 17:17, 20. Mär. 2012 (CET)

Ohne diese Funktion zu kennen: Wenn Thunderbird die Mails nicht im Postausgang listet, sind die auch nicht rausgegangen. --Hepha! ± ion? 21:06, 20. Mär. 2012 (CET)
Eben nicht. Auch erfolgreich verschickte Termineinladungen tauchen nirgends auf. --82.113.99.8 21:11, 20. Mär. 2012 (CET)
Termin öffnen, den freundlichen "Teilnehmer einladen"-Knopf drücken. --88.130.132.26 22:49, 20. Mär. 2012 (CET)
Das mache ich ja. Meine Fragen bleiben trotzdem.
Nochmal: solange der SMTP-Server OK ist und die Netzwerkverbindung auch, geht die Termineinladung raus. Ich kann das aber nicht nachvollziehen (keine Termineinladung im Ordner "Gesendet"). Schlägt irgendwas beim Senden der Einladung fehl, dann bekommt man eine Fehlermeldung, sucht aber die Termineinladung vergeblich im Ordner "Postausgang".
Man kann nun natürlich den Termin nochmal aufmachen und auf den freundlichen Knopf drücken. Aber solange man am Termin selbst nichts ändert, wird doch auch keine neue Einladung verschickt? (Kann man ja auch nicht nachvollziehen mangels Einladung im Gesendet-Ordner) --89.204.153.151 23:07, 20. Mär. 2012 (CET)
Also bei mir fragt er jedes Mal nach ob ich Einladungen verschicken will, wenn ich auf Speichern und Schließen klicke und dann schickt er jedes Mal Emails wenn ich ok sage. --88.130.132.26 00:19, 21. Mär. 2012 (CET)
Macht mein Thunderbird ja auch... normalerweise. Aber wenn es schiefgelaufen ist? Ich suche nach einer Möglichkeit, Termineinladungen erneut zu verschicken ohne den Termin erneut anlegen zu müssen. Darüber hinaus wäre ein späteres Auffinden der Einladung in Postausgang oder Gesendet-Ordner zur Kontrolle doch außerordentlich schön (hier hat mal Outlook die Nase vorne). --89.204.154.140 17:39, 21. Mär. 2012 (CET)

Fachbegriff für Situation

  • Es gibt zwei Tassen Tee. Der Tee in einer Tasse hat die optimale, heiße Trinktemperatur, der andere ist bereits abgekühlt, aber gerade noch zum Trinken annehmbar. Man hat die Wahl, entweder erst den ersten Tee mit optimaler Temperatur zu trinken und dann eine Tasse mit inzwischen auf unakzeptabel niedriger Temperatur abgekühlten Tee zu haben, oder aber erst den mittelwarmen Tee zu trinken und dann die zweite, inzwischen ebenfalls abgekühlte Tasse zu nehmen, sodass man zwei gleichermaßen subomptimal, aber noch akzeptable temperierte Getränke zu haben.

Wie heißt der Fachbegriff für solche Entscheidungssituationen, zum Beispiel in der Spieltheorie oder der Ökonomie? Danke! --78.52.152.53 19:57, 20. Mär. 2012 (CET)

Ich würd's zeitdiskretes Entscheiden/Handeln nennen, aber ob das ein wirtschaftlicher Fachbegriff ist? Jedenfalls kommt es dort ständig vor, weil man auf bestimmte Vorgänge eben nur zu bestimmten Zeitpunkten Einfluss hat und die optimale Reihenfolge davon abhängt, ob wie zwei Aktionen innerhalb ihrer jeweiligen Perioden zueinander stehen. Totzeit ist auch noch so ein Begriff, der passt. -- Janka (Diskussion) 20:19, 20. Mär. 2012 (CET)
Ganz klar: ein echt tragisches Dilemma! --93.209.87.177 20:48, 20. Mär. 2012 (CET)
+1 Dilemma.
Aber: Frager, du hast eine nicht uninteressante Denkweise! Du siehst dich selber im personellen Widerstreit mit dir selber (Spieltheorie betrifft mehrere Personen). Also: Entweder wirst du hier in 30 Jahren als Antwortonkel in der Auskunft landen (Tee neben dem Keybord, Katze auf'm Schoss), oder wir kommen dich alle in der Anstalt besuchen. Your choice! ;-) GEEZERnil nisi bene 21:53, 20. Mär. 2012 (CET) Ein analoges Problem ist übrigens, immer das gekaufte Brot aufzuessen, obwohl es schon trocken ist, ehe man das frische, das schon gekauft auf dem Tisch liegt, anbricht. Würde man gleich das Frische essen, müsste man aber das schon bezahlte alte wegschmeissen... Probleme über Probleme... ;-)))))
Es gibt Leute die finden das frische Brot widerlich, die hauen dass dann erstmal in den Tiefkühlschrank (und behaupten hinterher es wäre noch genauso frisch wie als es gekauft wurde). Wen man das Brot-Dilemma hat, hat man falsch eingekauft. --22:53, 20. Mär. 2012 (CET) (ohne Benutzername signierter Beitrag von 88.130.132.26 (Diskussion))

Um ehrlich zu sein: Ich selbst stelle mir die Frage immer nur beim Brot, da ich aber auch Leute kenne, die nur altes Brot mögen, wollte ich lieber eine universellere Frage stellen. Deswegen Tee im Beispiel.--78.52.152.53 23:22, 20. Mär. 2012 (CET)

Man kann sich diese Dilemmasituationen vereinfachen - und du bist schon auf dem Weg dazu, weil (a) du das Dilemma erkannt hast und (b) das Dilemma formuliert hast.
Man nimmt sich einen Nachmittag (andere Leute schaffen es schneller) an dem man sich bewusst und willkürlich in diese nervenzerfetzende Situation begibt (alle anderen Parameter - Verdauung, Musik, Wetter, etc. etc. - sollten "im Normalbereich" sein. Man zwingt sich also in diese Situation und löst sie ein für alle Mal, indem man feststellt "DAS ist jetzt die LÖSUNG, die mich auch in Zukunft glücklich machen wird" - denn man ahnt, dass das Leben noch ganz andere Herausforderungen für einen bereit hält...
Mit Milch oder ohne Milch ...? GEEZERnil nisi bene 09:04, 21. Mär. 2012 (CET)
Beides, bitte! --Zerolevel (Diskussion) 12:33, 21. Mär. 2012 (CET)
No Problemo ... das löst man quantenmechanisch: Schrödingers Milch für Schrödingers Katze GEEZERnil nisi bene 17:39, 21. Mär. 2012 (CET)
Für Tee oder Kaffee? -- Ian Dury Hit me  16:45, 21. Mär. 2012 (CET)
Man trinkt natürlich die optimalwarme Tasse und stellt die andere kurz in die Mikrowelle. Dazu ißt man dann großen Hans, den man aus dem alten Brot gemacht hat. --Optimum (Diskussion) 19:50, 21. Mär. 2012 (CET)

Film gesucht

Ich suche einen Stummfilm oder einen Film ohne Sprache, wahrscheinlich französischer Herkunft. Dabei erlebt eine Person nacheinander zahlreiche komische Situationen.

Einige Szenen spielen an einem Strand und in einem Haus und bei Umkleidekabinen. Es kommt ein länglicher Zuckerteig vor, der an einem Süßigkeits-Stand an einem Haken hängt und dabei langsam runter Läuft. Außerdem wird vom Bösewicht an einem alten Auto eine hintere Klappe zugeklappt, in der sich ein Mann sitzend befindet, so dass er eingeschlossen wird. Dabei fliegt der Hut in die Höhe. Es handelt sich um eine solche Klappe, die bei Autos sehr alter Bauart eine weitere Sitzmöglichkeit schaffen und sich dort befinden, wo der Kofferraum ist. Erkennt jemand diesen Film nach zugegeben sehr mageren, allerdings markanten Hinweisen? Wie heißt der? 77.235.178.4 20:04, 20. Mär. 2012 (CET)

Jaques Tati, Die Ferien des Monsieur Hulot? --Incognito.ergo.possum (Diskussion) 20:23, 20. Mär. 2012 (CET)
Jepp, das isses. Nicht stumm, aber Tati steht ganz klar in der Tradition der großen Stummfilmkomiker.
Ja, das ist er. Ihr seid wirklich gut!
Vielleicht könnte das filmhistorisch der Ausgangspunkt dieser Französischen Komödien sein, die bis in die achziger Jahre produziert wurden? Ich meine damit diese Der-Blonde-mit-dem-Schuh-Filme oder auch Louis Defunes-Filme, die dann später oft so "französisch-alberne" Elemente enthielten, also als man versuchte, diese Art Komik immer mehr zu übertreiben. 46.115.17.62 22:26, 20. Mär. 2012 (CET)
Ich glaube, Tatis Humor war um vieles subtiler und enthielt philopsophische / soziologische Aspekte einer Zeitkritik der anderen Art. Was dann später kam war eher eindimensionaler Slapstick für die Einfaltspinsel. Natürlich ist das die subjektive Bewertung eines Mehrfaltspinsels. --Incognito.ergo.possum (Diskussion) 22:47, 20. Mär. 2012 (CET)
Ich halte es ehrlich gesagt für gewagt, Tati als Ausgangspunkt der Französischen Komödie zu sehen. Dafür waren seine Filme zu außergewöhnlich, und Tati wirkt auf mich immer wie ein Außenseiter im französischen Film. Sein romantische Blick auf die Vergangenheit in Mein Onkel und die albern-beißende Kritik an der Moderne in Tatis herrliche Zeiten waren völlig aus der Mode, als die Filme herauskamen, und vielleicht deshalb auch noch heute so brilliant und wunderschön anzusehen (beide Filme gibts in DACH auf DVD).
Französische Komödien gab es schon vor Tati, und genauso wie der deutsche Humor nicht nur Didi Hallervorden, sondern eben auch Loriot oder Gerhard Polt ist, gibt es viele verschiedene Spielarten französischen Humors. Wenn es einen Urvater der (nicht nur französischen) Komödie gibt, dann muss man noch weiter in die Filmegschichte zurückgehen, und kommt dann zu Max Linder, der Anfang der 1910er Jahre einer der ersten Weltstars des Kinos war und einen deutlich erkennbaren Einfluss auf Charles Chaplin und Buster Keaton hatte... --Andibrunt 23:47, 20. Mär. 2012 (CET)
Bei dem Versuch, ein Bild gerade zu rücken, hinterlässt er ein Zimmer mit einigen demolierten und beschädigten Gegenständen. Heisst das, wir sind einem grossen Deutschen erneut als Plagiator aufgesessen...? Haben sich andere Herren - oder auch Damen - dieses Themas angenommen ? GEEZERnil nisi bene 09:36, 21. Mär. 2012 (CET)
Dein Vertrauen in Wikipedia-Artikel in allen Ehren, urteile doch selbst. --84.191.143.195 17:14, 21. Mär. 2012 (CET) p.s.: Hier noch eine Einordnung des Filmausschnitts. --84.191.143.195 17:18, 21. Mär. 2012 (CET) p.p.s.: Und zum Vergleich der angesprochene Sketch von Loriot. --84.191.143.195 17:23, 21. Mär. 2012 (CET) p.p.p.s: Vielleicht, weil einige hier nur Klartext verstehen, etwas deutlicher: Der ursprüngliche Satz findet schon bei der Neuanlage des Artikels Eingang durch Tobe man am 1. Juli 2006: "Bei dem Versuch, ein Bild gerade zu rücken, hinterläßt er ein Zimmer voller zerstörter Gegenstände, glaubt aber, ein schlafender Alter sei verantwortlich." Am 12. Oktober 2007 wird durch Dontworry die falsche Angabe durch eine etwas weniger falsche Angabe "korrigiert". Der "schlafende Alte" fliegt aus der freimütigen Interpretation raus und Hulot hinterläßt nur noch "ein Zimmer mit einigen demolierten und beschädigten Gegenständen". Im Gegensatz zu dem Sketch von Loriot ist die Frage der Zerstörung bei Tati aber nur von geringem Belang. Sie derart hervorzuheben lenkt davon ab, daß es in der Szene um die Gesamtheit der Situationskomik (inklusive des "schlafenden Alten") geht, die bei weitem nicht so drastisch ausfällt wie bei Loriot, dessen Sketch ja konsequent auf die völlige Zerstörung von großen Teilen der Zimmereinrichtung zusteuert. Die falsche, schlampige und subjektiv geprägte Darstellung der Inhalte ist im wp-Filmbereich leider kein Einzelfall. Ich bin da völlig abgegessen und lese das mittlerweile garnicht mehr und hole mir da nur noch die Links und die Faktenangaben... --84.191.143.195 17:58, 21. Mär. 2012 (CET)

Was ist das Gegenstück zu einem Amateurfunker?

ein Profifunker? Und wenn ja, wer darf sich so nennen? --95.112.245.194 21:57, 20. Mär. 2012 (CET)

siehe Funker. --Vsop (Diskussion) 22:09, 20. Mär. 2012 (CET)
Siehe auch Allgemeines Betriebszeugnis für Funker und Beschränkt gültiges Betriebszeugnis für Funker. --Taratonga (Diskussion) 18:34, 21. Mär. 2012 (CET)

Paternoster bei Charlie Chaplin

Aus dem Artikel Paternoster hierher gebracht und dort seit einem Jahr unbeantwortet: In welchem Film von Charlie Chaplin kommt ein Paternoster vor? --Shaun72 (Diskussion) 09:46, 21. Mär. 2012 (CET)

Also in Modern Times hatte das Kaufhaus Rolltreppen, in den anderen Gebäuden erinnere ich mich auch nicht an einen Paternoster. Mensch du ... Paternoster bin ich bestimmt auch 15 bis 20 jahre nicht mehr gefahren. -- 208.48.242.106 10:05, 21. Mär. 2012 (CET)
GoogleBooks => Chaplinesk Paternoster <= Das Booch mal einsehen; legt nahe, DASS Paternoster vorkamen... GEEZERnil nisi bene 10:07, 21. Mär. 2012 (CET)
Ich erinnere mich an eine solche Filmszene, es könnte aber auch Laurel und Hardy gewesen sein. --тнояsтеn 10:15, 21. Mär. 2012 (CET)
Sonst durch den Paternoster] in Metropolis (Not funny!) ersetzen. Klassiker! GEEZERnil nisi bene 10:21, 21. Mär. 2012 (CET)
Daran musste ich auch erst denken. Aber dann kam ich ins Grübeln. Das Große in Metropolis waren ja keine Paternoster, sondern normale Kabinenaufzüge. Wo sonst im Film tauchten da Paternoster auf, in welchem Gebäude? -- 208.48.242.106 11:10, 21. Mär. 2012 (CET)
Die Online-Enzyklopädie Wikipedia, gemeinhin für ihre gut bequellten Informationen bekannt, erwähnt im Artikel zum Paternoster einen Chaplin-Film, in dem ein Paternoster verkehrtherum wieder herunterfährt, leider ohne den Titel des Films zu nennen - und in diesem Fall auch ohne eine Quelle. In seinen späteren (und damit den bekannteren) Filmen kommt die Szene sicher nicht vor. --Snevern 11:31, 21. Mär. 2012 (CET)
<quetsch>Genau darauf bezog ich meine Aussage oben. Solch eine Szene gibt es definitiv, ich kann aber nicht schwören, dass es Chaplin war. --тнояsтеn 12:33, 21. Mär. 2012 (CET)
und da schließt sich der Kreis wieder--Shaun72 (Diskussion) 11:45, 21. Mär. 2012 (CET)
Die Gugel Suche nach „paternoster charlie chaplin upside down“ macht einen aber auch nicht gerade schlauer. -- 208.48.242.106 12:17, 21. Mär. 2012 (CET)
Hmm, vielleicht war Chaplin auch da unterwegs: [9] --тнояsтеn 18:43, 21. Mär. 2012 (CET)
Ich wollte soeben das Ausschlussprinzip vorschlagen, aber die Liste scheint mir doch ein wenig lang dafür. Die wenigsten davon kenne ich, und die die ich kenne kann ich zum Teil keinem titel zuordnen oder nicht ausschließen dass ein Paternoster drin vorgekommen wäre.
Ich weiß nicht mal, ob ich die Szene kenne, oder ob es sich einfach so schön vorstellen lässt wie es wäre, dass man glaubt, es schon gesehen zu haben. -- 208.48.242.106 13:02, 21. Mär. 2012 (CET)
Hier ist er auch nicht dabei. Dafür aber M - Eine Stadt sucht einen Mörder. -- 208.48.242.106 13:55, 21. Mär. 2012 (CET)

Youtube-Aufrufe

Angenommen, man kennt den Nick eines Youtube-Users: Kann man dann sehen, welche Videos er aufgerufen hat? (siehe auch WP:AU: Youtube beobachten

--77.4.95.214 09:47, 21. Mär. 2012 (CET)

Du kannst das nicht, das wär ja ärgstes Nachspionieren. Die Betreiber von Youtube (die Firma Google Inc.) können das aber sehr wohl. --El bes (Diskussion) 10:05, 21. Mär. 2012 (CET)
machen Die regelmäßig, sonst würdest Du nicht mit passender Werbung bepflastert. Andreas König (Diskussion) 17:40, 21. Mär. 2012 (CET)
Danke sagt 77.4.95.214 23:50, 21. Mär. 2012 (CET)

Arbeitslosenversicherung

Warum muß ich in der passiven Phase der Altersteilzeit noch den vollen Anteil in die Arbeitslosenversicherung einzahlen? --87.245.34.169 11:50, 21. Mär. 2012 (CET)

Weil es so festgelegt ist, um die Kassen immer gut gefüllt zu halten? Recht und Gerechtigkeit sind zweierlei Schuhe. -- 208.48.242.106 12:00, 21. Mär. 2012 (CET)
Weil du auch noch Entgelt bekommst? --Eingangskontrolle (Diskussion) 12:27, 21. Mär. 2012 (CET)
Hut ab, wirklich sorgfältig gelesen und gedacht, lieber Kollege: Vollzeit = voller Anteil, Teilzeit = du bekommst noch Entgelt = voller Anteil? --84.191.143.195 14:20, 21. Mär. 2012 (CET)
Nur mal so ein Gedanke, kann es sein, daß bei Wegfall der Altersteilzeit, aus irgendwelchen Gründen vor Renteneintritt auch ein Anspruch auf volles Arbeitslosengeld besteht? Ansonsten verstehe ich als Laie die Frage nicht, was ist der "volle" Anteil? Normalerweise zahlen doch Arbeitnehmer und Arbeitgeber die Beitrag hälftig.Oliver S.Y. (Diskussion) 17:09, 21. Mär. 2012 (CET)
Die Beiträge werden auf das Regelarbeitsentgelt berechnet. Das ist das, was der Altersteilzeitgeldbeziehende aktuell bekommt. Er ist nicht arbeitslos, er ist in Altersteilzeit. Wenn auch ruhend, so doch nicht arbeitslos. Warum sollten also keine Beiträge abgeführt werden? Für die RV allerdings sieht die Berechnung anders aus, da zahlt der Arbeitgeber nochmal drauf. Siehe auch Artikel Altersteilzeit -- Ian Dury Hit me  17:25, 21. Mär. 2012 (CET)

Was genau ist der Unterschied zw. obigen 3 Bauelementen.

  1. Gibt es einen Unterschied zwischen Drossel und Spule? Wenn ja, dann müssen beide Artikel besser von einander abgegrenzt werden, wenn nein, dann sollte Drossel auf Spule redirected werden.
  2. Ist eine Festinduktivität auch einfach gewickelter Draht? Wenn ja, warum heißen die dann Festinduktivität, was soll "fest" bedeuten? Eine Spule hat auch immer die gleiche, feste, Induktivität. Wenn nein, wie sind Festinduktivitäten aufgebaut?
  3. Warum gibt es keinen Artikel Festinduktivität?

Danke!!!--svebert (Diskussion) 12:41, 21. Mär. 2012 (CET)

Es gibt auch veränderliche Spulen, bei denen der Ferritkern raus- und reingeschraubt werden kann. Diese Dinger waren früher in Radios und Fernsehern verbaut, weil man sie auf den genauen Wert abgleichen konnte. Bei Autoradios und auch Fernsehern gab es vor Erfindung der Kapazitätsdiode auch Tuner, bei denen die Stationstasten über einen Mechanismus mit veränderlichen Spulen realisiert wurden. Diese sind wesentlich robuster und vibrationsfester als veränderliche Kondensatoren. --Rôtkæppchen68 13:03, 21. Mär. 2012 (CET)
„spule“ scheint mir n oberbegriff zu sein (also alles was hauptsächlich induktive eigenschaften hat)... „drossel“ ist spezieller (nach meinem gefühl eher was für energiespeicherung/leistungselektronik)... und „festinduktivität“ ist wohl mehr was für signalverarbeitung (fernseher, schwingkreis)... --Heimschützenzentrum (?) 14:08, 21. Mär. 2012 (CET)
wobei „drossel“n wohl meist auch „festinduktivität“en sind... *am kopf kratz* --Heimschützenzentrum (?) 18:27, 21. Mär. 2012 (CET)
Eine Drossel ist eine Spule, die nicht in einem Schwingkreis sitzt, sondern gerade zur Verhinderung von Stromänderung eingesetzt wird. Der wesentliche Unterschied im Aufbau ist, dass die Schwingkreisspule auf möglichst hohe Güte und Linearität optimiert ist, während bei der Drossel eine möglichst hohe Induktivität im Vordergrund steht. -- Janka (Diskussion) 17:17, 21. Mär. 2012 (CET)
Ein weiterer Punkt ist, dass bei Drosseln auch minderwertigere Kernwerkstoffe zur Anwendung kommen, also nicht unbedingt die verlustarmsten Ferrite, sondern teilweise auch nur Elektroblech, wie z.B. bei Leuchtstofflampendrosseln. --Rôtkæppchen68 00:36, 22. Mär. 2012 (CET)

Das geht mir aufs Schwein

Ich habe eben eine Diskussion über den Spruch "Das geht mir aufs Schwein". Ich bin der Meinung das ist vollkommen unbekannt. Behauptet wird das sei ein üblicher Spruch mit überregionaler Verbreitung und eventuell sogar ein klassisches Zitat (Göthe) ? --81.200.198.20 16:10, 21. Mär. 2012 (CET)

Unbekannt und sicherlich nicht "klassisch". "Das geht mir aufs Gemüt", "Das schlägt mir aufs Gemüt" schon eher. GEEZERnil nisi bene 16:20, 21. Mär. 2012 (CET)

Das geht mit aufs/ auf den/ auf die ... ist ein ziemlich verbreiteter Spruch.
Folgende Varianten sind mir geläufig:

  • auf den Keks
  • auf den Wecker
  • auf den Senkel
  • auf den Sack
  • aufs Schwein
  • auf den Zeiger

Den Goehte halte ich hiernach jedoch für äußerst fragwürdig -- 208.48.242.106 16:21, 21. Mär. 2012 (CET)

Ich kenne auch alle genannten, biete außerdem Nerven und Eier. Aber Schwein? Nie gehört. Und klassisch glaub ich nicht bis zum Beweis des Gegenteils. --Joyborg (Diskussion) 16:25, 21. Mär. 2012 (CET)
Mit Google kann man ja viel falsch machen, aber zu Beantwortung der Frage, ob eine Redewendung üblich ist, taugt es doch sehr. [10] --Eike (Diskussion) 16:26, 21. Mär. 2012 (CET)
Ich kannte aufs Schwein gehen noch nicht, aber dafür auf den Geist gehen. --Rôtkæppchen68 16:29, 21. Mär. 2012 (CET)
"Auf die Nerven" und "an die Nieren"... Aber auf’s Schwein? Nö. -- Ian Dury Hit me  16:36, 21. Mär. 2012 (CET)
„Das geht mir aufs Schwein“ ist aus meiner Sicht ein bekannter Spruch, fällt hier im Büro öfters mal. :) Siehe zudem Eikes Google-Link. Unstrittig verbreitet. --NiTen (Discworld) 16:36, 21. Mär. 2012 (CET)
Ich hatte es auch noch nie gehört, aber man darf halt nicht von "Kenn ich nicht." auf "Gibt es nicht!" schließen. (Auch nicht in Dialekten...) --Eike (Diskussion) 16:40, 21. Mär. 2012 (CET)
Dieser Spruch muss sehr neu sein. Der älteste Treffer, den ich bei Google finde ist von 2001. --Rôtkæppchen68 16:42, 21. Mär. 2012 (CET)
Definiere "neu". Aus der Berliner Ecke kenne ich den schon seit den frühen 1990er Jahren. --NiTen (Discworld) 16:45, 21. Mär. 2012 (CET)
Ditte jeht mer voll uffet Schwein, wah! -- 208.48.242.106 16:52, 21. Mär. 2012 (CET)
Ich glaube, das ist schon eher ein Punkt, wo Google nicht heftig geeignet ist. Wieviele Webseiten (mit umgangssprahchlichen Texten) stehen seit dem letzten Jahrtausend unverändert im Netz...? --Eike (Diskussion) 16:59, 21. Mär. 2012 (CET)
Bis auf "Das geht mir auf die Nerven" sind alle oben genannten Beispiele Gossensprache. --El bes (Diskussion) 17:17, 21. Mär. 2012 (CET)
Naja, und "auf den Geist". --stfn (Diskussion) 17:47, 21. Mär. 2012 (CET)
Ich empfinde eigentlich nur den Sack als fragwürdig. Alle anderen klingen für mich flapsig, aber im normalen Umgang durchaus akzeptabel. --89.246.169.139 18:38, 21. Mär. 2012 (CET)
Ich würde das auch "Umgangssprache" nennen und von der "Gossensprache" unterscheiden. --Eike (Diskussion) 19:28, 21. Mär. 2012 (CET)

Humanologie

Geezer hat zu Recht in der Suchhilfe darauf hingewiesen. Im Artikel Kulturerdteil steht "Kulturerdteil ist ein Begriff aus der Humanologie." Bitte was ist die Humanologie? (Das da ist wohl nicht gemeint.) Kann das jemand beheben? --Neitram 17:59, 21. Mär. 2012 (CET)

Anthropologie, teillatinisiert? Kirschschorle (Diskussion) 18:03, 21. Mär. 2012 (CET)
ja, entweder das, oder man könnte auch Humangeographie schreiben. --El bes (Diskussion) 18:10, 21. Mär. 2012 (CET)
Dieser Humanologiedefinition nach ist da ein bisschen mehr als nur Geographie dabei. --Rôtkæppchen68 18:17, 21. Mär. 2012 (CET)
französische Wissenschaftsterminologie muss nicht ident mit deutscher sein. --El bes (Diskussion) 18:22, 21. Mär. 2012 (CET)
Äusserst kritisch beiden Begriffen gegenüber... Mal "Humanologie" in GoogleBooks eingeben <schauder> Ausser Konkurrenz: Definition Humanologie GEEZERnil nisi bene 19:04, 21. Mär. 2012 (CET)
Habe mir gerade diese Definition (Kolb) durchgelesen. Bitte bei den Soziologen etc. nachfragen, das ist extrem fadenscheiniger Krempel (ganz Afrika ist EIN Kulturerdteil. YOOH! Das würde erklären, warum sich alle da so lieb haben und untereinander verstehen...). GEEZERnil nisi bene 20:37, 21. Mär. 2012 (CET)

Errorcode U8201 Westerunion

Ich versende recht häufig geschäftlich Geld über Westernunion. Ab und an kommt es vor das auf der Westernunion Seite der Errorcode U8201 angezeigt wird. Wenn das passier kann man nicht nur kein Geld überweisen sondern ist für Onlineüberweisungen für einen Monat geblockt. Ruft man deswegen die Hotline an und nennt den Code werden die (so hab ich das Gefühl) recht unwirsch sagen das Sie dazu keine Auskunft geben können, der Computer das so entschieden hat das jetzt einen Monat Prüfroutinen?! laufen und das man da nichts machen kann. Den Code findet man in massig Foren zu dem Thema, aber nie mit Antworten. Weiß jemand was das für ein geheimnissvoller Errorcode ist? Ich vermute schon fast das ist die Fehlermeldung für durch das System angenommener Geldwäsche Verdacht oder ähnliches. --85.180.217.52 19:54, 21. Mär. 2012 (CET)

Wenn Du schon die diversen Foren zum Thema gelesen hast, dann kennst Du ja auch schon die Antwort auf diese Frage. Es ist Betriebsgeheimnis von Western Union, wie diese Meldung zustande kommt. WU rät, die Transaktion bar am Schalter vorzunehmen. --79.224.241.9 23:21, 21. Mär. 2012 (CET)

Rätsel um Datenanalyse in QtiPlot

Ich muss in QtiPlot den linearen Zusammenhang zwischen zwei Datenreihen ermitteln. Rätselhafterweise ist dieser lineare Zusammenhang aber größer als alle Einzelwerte, wenn man das Verhältnis der jeweiligen Daten ausrechnet. Konkret errechnet er sich als 3,79 +/- 0,12, während die Einzelwerte 3,44 bis 3,78 betragen, abgesehen von einem Ausreißer nach unten mit 2,64. Weiß jemand, wie das sein kann? --KnightMove (Diskussion) 09:37, 19. Mär. 2012 (CET)

Jetzt wollt ich mal kucken, was war noch mal ein linearer Zusammenhang, darf der vielleicht einfach größer sein? Nach der Lektüre von Linearer Zusammenhang war mir dann schlagartig alles klar. Ähem. Manchmal fühlt man sich in der Wikipedia wie eine OMA.--Eike (Diskussion) 10:19, 19. Mär. 2012 (CET)

Verstehe die Frage nicht, meinst du mit "lineare[r] Zusammenhang", dass du versuchst eine Gerade in die Daten zu fitten und 3,79 +/- 0,12 die Steigung der Geraden ist? Natürlich kann die Steigung größer als die Datenpunkte sein, z.B. die hat die Gerade durch die Punkte (0/0) und (0.1,1) die Steigung 10. Oder willst du den Korrelationskoeffizienten ausrechnen, der sollte aber zwischen -1 und 1 sein. --132.230.80.150 10:34, 19. Mär. 2012 (CET)

Ich habe es inzwischen herausgefunden... danke jedenfalls für die Antworten. --KnightMove (Diskussion) 11:48, 19. Mär. 2012 (CET)

Leider muss ich meine vorige Aussage doch widerrufen. Was ich meinte, war das folgende: Ich habe zwei Datenreihen x(i) und y(i), zwischen denen eine Ausgleichsgerade ermittelt werden soll. Die Steigung dieser Ausgleichsgerade beträgt 3,79 (siehe oben), während die Einzelwerte für y(i)/x(i) zwischen 3,44 und 3,78 liegen. Intuitiv wäre eher zu erwarten, dass die Steigung irgendwo in der Mitte der Werte liegt. Die Lösung, so dachte ich vor 12 Stunden, liegt darin, dass die Gerade ja nicht durch den Nullpunkt geht - wenn der extrapolierte Wert bei x=0 irgendwo liegt, kann auch die Geradensteigung andere Werte annehmen. Aber den Nullpunkt als Datenpunkt hinzuzunehmen, verändert die Gerade nur minimal und löst den offenbaren Widerspruch nicht. Hat jemand eine Idee? --KnightMove (Diskussion) 00:16, 20. Mär. 2012 (CET)

Ich versteh den Widerspruch noch nicht. (Aber vielleicht versteh ich auch die Frage nicht.) Wenn du einen Punkt bei 0,1/0,1 hast und einen bei 0,2/0,2, ist die Steigung ja auch 1, also weit außerhalb der Werte... --Eike (Diskussion) 10:46, 20. Mär. 2012 (CET)
Der Quotient der Werte ist aber genau 1, so wie eben die Steigung. Wie kann ich nun eine Steigung herauskriegen, die größer ist als der Quotient aller Werte? --KnightMove (Diskussion) 19:13, 20. Mär. 2012 (CET)
Ok, ich hatte die Frage wirklich falsch/nicht verstanden. Macht aber nichts. Nimm die Werte 0,1/1 und 0,2/1,1. Die Steigung ist immer noch 1, aber die Quotienten sind klein. Der Quotient der Werte hat etwas mit der Steigung einer Geraden durch den Nullpunkt und den gegebenen Punkt zu tun, aber nichts mit der Steigung zwischen den Punkten oder durch eine Punktwolke. --Eike (Diskussion) 19:30, 20. Mär. 2012 (CET)
Aber wenn man davon ausgeht, dass die Gerade auch durch den Nullpunkt gehen sollte, dann ist die Annahme richtig, oder? --KnightMove (Diskussion) 20:24, 20. Mär. 2012 (CET)
Natürlich würde man intuitiv einen Wert zwischen x(i) und y(i) erwarten, aber nur, wenn man den neuen Wert mit z(i) = (x +y)/2 ausrechnet. Wenn aber zur Berechnung von z(i) alle Ausgangswerte verwendet werden, kann natürlich ein ganz anderer Wert rauskommen, weil ja die Methode der kleinsten Quadrate angewendet wird, die nicht linear ist! Ausreißer von x(i) können bei y(i) genau reinpassen und umgekehrt. Insofern ist die intuitiv-Methode keinesfalls zu empfehlen. 89.247.171.166 23:01, 20. Mär. 2012 (CET)
Wenn du den Nullpunkt als einen weiteren Datenpunkt nimmst, dann geht die Ausgleichsgerade noch lange nicht durch den Nullpunkt (genausowenig wie durch jeden anderen Datenpunkt). Vermutlich fittest du das Modell  . Wenn das geschaetzte b ungleich Null ist, dann hat das Verhaeltnis y/x keine Bedeutung mehr. Wenn du stattdessen das Modell   fittest (sollte sich irgendwie machen lassen - ich kenne das Programm nicht), dann sollte die geschaetzte Steigung a gleich dem Mittelwert der y(i)/x(i) sein. Vom statistischen Standpunkt ist es uebrigens selten empfehlenswert, den Achsenabschnitt b explizit auf Null zu setzen. --Wrongfilter ... 23:12, 20. Mär. 2012 (CET)
Du hast natürlich recht, danke. Aber beim letzten Satz: Warum das? Wenn nach der Theorie die Gerade durch den Nullpunkt gehen sollte, was ist falsch daran? --KnightMove (Diskussion) 13:00, 21. Mär. 2012 (CET)
Ich hab noch nie eine Funktion "geftittet", aber ich probier's mal: Wenn die Theorie sich irrt (wie's bei dir ja anscheinend der Fall war), dann tust du der Funktion Gewalt an, wenn du das b weglässt. Wenn die Theorie sich nicht irrt, solltest du eh nah bei b=0 landen. Ich weiß aber nicht, was dagegen spricht, dann nochmal ohne b zu "fitten". Man muss sich dann aber klar machen, dass das Ergebnis nicht mehr nur den Daten entspringt, sondern "berichtigt" wurde. Andererseits ist das ja auch schon dadurch der Fall, dass man mit y = ax + b eine Gerade erzwingt... --Eike (Diskussion) 10:08, 23. Mär. 2012 (CET)

Wochenorientierter Kalender

Suche einen Kalender der primär wochenorientiert und nicht monatsorientiert ist, auerdem in einer Jahresansicht (und nicht "eine Woche auf einer Seite" oder so). Könnte mir vorstellen, dass einfach vier Wochen in einer Zeile stehen.--Michael Scheffenacker (Diskussion) 13:31, 21. Mär. 2012 (CET)

Kalender als was? Als Programm? Als Druckvorlage? Als Wandkalender? Als Buchkalender? -- 208.48.242.106 13:45, 21. Mär. 2012 (CET)
Eine Druckvorlage wäre optimal. Mir würde eventuell schon ein Name dafür reichen, damit ich mir selber was im Interwebs suchen kann. Wenn ich hier nichts finde, werde ich mir selber was coden, aber ich hätte halt gerne was schön formatiertes. --Michael Scheffenacker (Diskussion) 13:55, 21. Mär. 2012 (CET)
Wie wärs mit Wochenkalender - ich glaube meine Sparkasse verschenkte sowas mal als Kalender zum auf den Schreibtisch legen. --Eingangskontrolle (Diskussion) 14:08, 21. Mär. 2012 (CET)
Lightning, welches auch in SeaMonkey drin ist und mittlerweile glaube auch in Thunderbird, kann das und ist druckbar. Zumindest in der Wochenansicht. --94.134.210.185 14:28, 21. Mär. 2012 (CET)

Ich dachte da mehr an sowas wie das hier

So Mo Di Mi Do Fr Sa So Mo Di Mi Do Fr Sa So Mo Di Mi Do Fr Sa So Mo Di Mi Do Fr Sa So Mo Di Mi Do Fr Sa So
Jänner 1 2 3 4 5 6 7 8 9 10 11 12 13 14 15 16 17 18 19 20 21 22 23 24 25 26 27 28 29
Februar 30 31 1 2 3 4 5 6 7 8 9 10 11 12 13 14 15 16 17 18 19 20 21 22 23 24 25 26
März 27 28 29 1 2 3 4 5 6 7 8 9 10 11 12 13 14 15 16 17 18 19 20 21 22 23 24 25 26 27 28 29 30 31 1
April 2 3 4 5 6 7 8 9 10 11 12 13 14 15 16 17 18 19 20 21 22 23 24 25 26 27 28 29
Mai 30 1 2 3 4 5 6 7 8 9 10 11 12 13 14 15 16 17 18 19 20 21 22 23 24 25 26 27 28 29 30 31 1 2 3
Juni 4 5 6 7 8 9 10 11 12 13 14 15 16 17 18 19 20 21 22 23 24 25 26 27 28 29 30 1
Juli 2 3 4 5 6 7 8 9 10 11 12 13 14 15 16 17 18 19 20 21 22 23 24 25 26 27 28 29
August 30 31 1 2 3 4 5 6 7 8 9 10 11 12 13 14 15 16 17 18 19 20 21 22 23 24 25 26 27 28 29 30 31 1 2
September 3 4 5 6 7 8 9 10 11 12 13 14 15 16 17 18 19 20 21 22 23 24 25 26 27 28 29 30
Oktober 1 2 3 4 5 6 7 8 9 10 11 12 13 14 15 16 17 18 19 20 21 22 23 24 25 26 27 28
November 29 30 31 1 2 3 4 5 6 7 8 9 10 11 12 13 14 15 16 17 18 19 20 21 22 23 24 25 26 27 28 29 30 1 2
Dezember 3 4 5 6 7 8 9 10 11 12 13 14 15 16 17 18 19 20 21 22 23 24 25 26 27 28 29 30 31


--Michael Scheffenacker (Diskussion) 16:00, 21. Mär. 2012 (CET)

Du suchst einen Kalender, in dem zB der Jänner nur 29 Tage hat und im März der 27. mal als Montag und mal als Dienstag geführt wird? -- Ian Dury Hit me  16:43, 21. Mär. 2012 (CET)
Ich glaube, der "Rest-Januar" steht schon im Februar und der erste "27. März" ist noch vom Feb. --Bremond (Diskussion) 20:47, 21. Mär. 2012 (CET)
Früher war es (zumindest bei uns) üblich, den Lohn für Arbeiter nicht monatlich, sondern wöchentlich abzurechnen, wodurch sich bei monatlicher Auszahlung 4-/5-wöchige Abrechnungsperioden ergaben. Und da hieß es, daß ein Monat mit dem Montag der Woche beginnt, in die der Monatserste fällt. War also der 1. Februar ein Donnerstag, begann der Februar mit dem 29. Jänner. - Ich nehme mal an, es wird ein Kalender nach diesem System gesucht. --Haeferl (Diskussion) 20:05, 21. Mär. 2012 (CET)
Das System kenn ich auch heute noch von einigen nordamerikanischen Unternehmen. Vgl. en:4–4–5 calendar -- TZorn 21:43, 21. Mär. 2012 (CET)
Jetzt wo Ihr es schreibt, erinnere ich mich wieder. Danke! -- Ian Dury Hit me  13:19, 22. Mär. 2012 (CET)

Gruppe&Spar, auch gültig im Nahverkehr?

Wir fahren morgen als Gruppe mit einem Bahnticket (Gruppe&Spar) nach Berlin. Ziel ist Berlin Hbf, wo wir mit einem IC ankommen. Wir möchten aber noch S-Bahnen zur Weiterfahrt zum Zielort nehmen. Ist dies vom Ticket noch gedeckt oder benötigt man dafür zusätzliche S-Bahn-Tickets? --79.195.216.61 14:09, 21. Mär. 2012 (CET)

Mit Berlin/Brandenburticket, Schönes Wochenende Ticket oder ähnlichem war es eigentlich immer so, dass man zwar S-Bahn fahren darf in Berlin, jedoch nicht U-Bahn. -- 208.48.242.106 14:16, 21. Mär. 2012 (CET)
Der Unterschied ist ja, dass die von Dir aufgezählten Tickets den ganzen Tag gültig sind, während das Gruppe&Spar-Ticket nur für eine bestimmte Strecke gilt. In den Fernverkehrszügen gilt Zugbindung, bei den Nahverkehrszügen kann man sich einen beliebigen Zug aussuchen. Also ziemlich vergleichbar mit einer normalen Fahrkarte. Ziel ist "Berlin Hbf", und für weitere Fahrten gilt die Fahrkarte ja eigentlich nicht. Oder kann man noch die S-Bahnen innerhalb Berlins als "Anschlusszüge" nutzen? 79.195.216.61 14:22, 21. Mär. 2012 (CET)
Aber vorher vergewissern, ob die gewünschte S-Bahn überhaupt fährt. --Komischn (Diskussion) 14:32, 21. Mär. 2012 (CET)
Diese Seite lässt darauf schließen, dass es nromalerweise nicht dabei ist, wenn es hier als Sonderleistung angepriesen wird. -- 208.48.242.106 15:18, 21. Mär. 2012 (CET)
Ich weiß nicht, wie es mit dem Tarifzonensystem ist. Weil eigentlich zahlst du ja nicht bis zum Bahnhof X sondern bis zur Tarifzone, in der Bahnhof X liegt. Wenn dein Zielbahnhof nun 3 Haltepunkte hinter Bahnhof X liegt, du dafür allerdings vom Fern in den Nahverkahr umsteigen musst, ändert sich ja auch der Fahrpreis normalerweise nicht. -- 208.48.242.106 15:22, 21. Mär. 2012 (CET)
Kurz und knackig: Es gilt das auf dem Ticket aufgedruckte Fahrziel. Steht dort "Berlin Hbf", dann gilt es nur bis dort, auch wenn es einen Verkehrsverbund gibt und "Berlin Hbf" in einer Zone mit dem gewünschten S-Bahn-Fahrziel liegt. -- 188.105.142.133 15:30, 21. Mär. 2012 (CET)
Es wäre mir neu, dass in DE Bahnhöfe als Ziel in der Fahrkarte eingetragen werden. Schon seit einigen Jahren wird nur noch die Ziel-Stadt angegeben und du kannst dort im Anschluss an deine IC/ICE-Fahrt auch noch den Bahn-Nahverkehr nutzen, sprich z. B. S-Bahn, Regionalbahn, Regionalexpress, aber nicht ohne weiteres U-Bahn, Straßenbahn oder Bus. Ich fänd es ungewöhnlich, wenn das beim Gruppenticket anders wäre. Das Fahrkarten-Ziel darf nicht mit dem Ziel in der Reiseverbindung verwechselt werden, die ihr evtl auch bekommen habt. Dort steht natürlich der konkrete Bahnhof mit Gleis drin. -- TZorn 15:47, 21. Mär. 2012 (CET)
Die Deutsche Bahn betreibt meines Wissens nirgendwo U- oder Straßenbahnen. In welcher Stadt kann man mit einem DB-Fahrschein eine U- oder Straßenbahn nutzen? --79.224.241.9 15:57, 21. Mär. 2012 (CET)
Ohne City-Ticket? In keiner. Nur S-Bahnen und ähnliche Nahverkehrszüge. -- 208.48.242.106 16:08, 21. Mär. 2012 (CET)
Wenn ich mir ein Ticket Leipzig-Chemnitz kaufe, fahr ich da mit einem RE und komme in Chemnitz Hbf an. Wenn ich aber nach Chemnitz Süd will, kann ich ohne ein neues Ticket zu kaufen in die RB richtung Zwickau steigen und bis Chemnitz Süd fahren. Zumindest war das vor ein/ zwei Jahren so. -- 208.48.242.106 16:12, 21. Mär. 2012 (CET)
Vergiss die Bahnbusse nicht, oder was davon unter 1000 verschiedenen Namen übriggeblieben ist, weil die damalige Deutsche Bundesbahn diese Defizitbringer zwar verkaufen wollte, das aber bis auf wenige Ausnahmen nicht geschafft hat. --79.224.241.9 16:33, 21. Mär. 2012 (CET)
Mit dem passenden Bahn-Ticket in vielen: http://www.bahn.de/p/view/bahncard/vorteile/cityticket.shtml --Eike (Diskussion) 16:36, 21. Mär. 2012 (CET)
Auf die Gefahr hin, mich zu wiederholen: Es gilt das aufgedruckte Fahrziel. Punkt. Solche Spässekens wie City-Ticket lassen wir mal aussen vor, wer das hat, weiß auch, was er da in der Hand hat, spätestens nach Eikes Verlinkung da oben. Und ja, Bahnhofsnamen werden aufgedruckt. Beispiele z.B. Plauen (es gibt oberer und unterer Bahnhof), Gera (Außer Gera selbst gibt es noch Gera Süd, Gera Ost). Kein Zusatz, dann Hauptbahnhof. Nur weil Du jahrelang (unwissentlich) schwarz gefahren bist, lieber TZorn, heißt das nicht, dass es jeder machen sollte. Auch wenn meine Kollegen dann wieder eine Quoten-Sorge weniger haben, wenn noch mehr auf Deine Idee kommen. -- 188.105.142.133 20:58, 21. Mär. 2012 (CET)
(Partieller) Einspruch - "Kein Zusatz, dann Hauptbahnhof." muss nicht immer stimmen. Auf Grund der "Tariflichen Gleichstellung" kann es (im Fall von ein paar Dutzend Städten bei Fahrten > 100 km!) in der Tat sein, dass S-Bahn-Fahrten zu bestimmten (!) Fahrtzielen durch die Fahrkarte abgedeckt sind. Siehe bahn.de und die dort verlinkt PDF-Datei. --78.48.192.186 21:16, 21. Mär. 2012 (CET)
Verdammt, da weiß bei uns wieder die eine Hand nicht, was die andere tut. Ich hatte definitiv schon ein Fernverkehrs-Ticket in der Hand mit Ziel-Aufdruck Gera Süd, was es laut dieser Tabelle ja gar nicht geben dürfte. WTF. -- 188.105.142.133 21:36, 21. Mär. 2012 (CET)
Doch ist möglich. Denn wenn es weniger als 100 Kilometer sind, oder ein Fahrschein der Produkteklasse C ist. Dann ist bei Gera der genaue Endbahnhof erforderlich. Ja ich weiss, solche Tabellen muss man auch lesen können --Bobo11 (Diskussion) 21:47, 21. Mär. 2012 (CET)
Das habe ich gelesen, der Abgangsbahnhof lag im Dreieck Stuttgart-Augsburg-München, wenn das weniger als 100km bis Gera Süd sind, dann geht Google Maps falsch. Und es war ein normales Hin- und Rückfahrticket, nicht mal ein BC-Rabatt oder eine Zugbindung. Also auch nix mit Produktklasse C. Und noch dazu war es ein Automatenticket, also nix mit vom Schalterpersonal falsch ausgestellt oder so. Mysteriös. -- 88.67.153.22 10:43, 22. Mär. 2012 (CET)
Und dann gibts noch den NRW-Tarif (ok, im Nahverkehr …) --Stefan »Στέφανος«  21:44, 21. Mär. 2012 (CET)
NRW ist zwar groß, aber so groß, dass Berlin drinliegt dann aber auch wieder nicht. --Rôtkæppchen68 00:33, 22. Mär. 2012 (CET)

Artikel des Bereichs Mathematik

Meine Anfrage betrifft -nicht scharf abgrenzbar- eine ganze Reihe von Artikeln des Fachbereichs Mathematik: Mir ist aufgefallen, dass ich den Ausführungen (Herleitungen u.Ä.) in den Artikeln häufig nicht oder nur bruckstückhaft folgen konnte, obwohl ich zumindest eine solide mathematische Grundbildung habe (Abi mit Leistungsfach Physik, gelegentliche Berührungen im Kontext des Biologie- und Sozialwiss.-Studiums). Da eine Enzyklopädie wie Wikipedia ja theoretisch (soviel ich weiß) den Anspruch der allgemeinen Verständlichkeit auch für Nicht-Spezialist_innen verfolgt, frage ich mich, ob die didaktische Konzeption der Mathematik-Artikel (und ggf. auch anderer Bereiche) generell weiter entwickelt werden könnte?-oder ob dieses Problem aufgrund einer (noch?) zu großen Trennung von Allgemeinbildung und Profi-Mathematik unvermeidlich ist? --Bramkamp (Diskussion) 16:16, 21. Mär. 2012 (CET)

Also, ich hab vergleichbare Voraussetzungen und dasselbe Problem. Ich vermute, dass es Bereiche gibt, die den Laien einfach kaum zu erklären sind, und dass es andere Bereiche gibt, die verbessert werden könnten. Ein Ansprechpartner wäre Portal:Mathematik. Das Prinzip, auf das du dich berufen kannst, heißt übrigens WP:OMA. --Eike (Diskussion) 16:23, 21. Mär. 2012 (CET)
(BK) Die richtige Anlaufstelle fuer dein Anliegen ist Portal:Mathematik/Qualitätssicherung. Konkrete Beispiele sind sicher hilfreich, um dort die Ohren zu öffnen. --Wrongfilter ... 16:27, 21. Mär. 2012 (CET)
[BK2] Da bin ich froh, dass es dir auch so geht. Mir geht es ähnlich, ich schrieb es aber meinem mathematischen Unverständnis zu. Ein Billigbuch von einem bekannten Lebensmitteldiscounter ist allerdings immer in der Lage gewesen, mir die (zu Schulzeiten noch unverständlichsten) Sachverhalte schnell klar zu machen, warum also nicht auch hier? Ich habe deine Frage mal im entsprechenden Portal bei deren QS platziert. -- Ian Dury Hit me  16:32, 21. Mär. 2012 (CET)
Also in der höheren Mathematik ist es meiner Ansicht nach kaum möglich dem Laien noch zu erklären worum es hier geht. Den Artikel Atiyah-Singer-Indexsatz allgemeinverständlich zu schreiben, erscheint mir unmöglich, da meines Wissens nicht mal eine besondere Anschauung dahintersteht. Aber um solche Artikel geht es hier wahrscheinlich nicht mal, oder? Welche Artikel haben Euch denn zu dieser Diskussion bewogen? Bestimmt gibt es bei vielen weniger abstrakten Themen (beziehungsweise Artikeln) die Möglichkeit sie verständlicher zu formulieren, aber ich denke das bedarf einer Einzelprüfung. --Christian1985 (Diskussion) 16:46, 21. Mär. 2012 (CET)
Keine Sorge, das geht mir als promoviertem Ingenieur auch so ;) --89.204.154.140 17:48, 21. Mär. 2012 (CET)
Ein Verwandter von mir hat Mathematik studiert und gibt zu, dass er (jetzt, einige Jahre später) seine eigene Diplomarbeit nicht versteht. Natürlich könnte er sich wieder einarbeiten, aber das wäre auch erforderlich. So ist sie halt, die höhere Mathematik... jedenfalls nichts für Omas. --Snevern 18:30, 21. Mär. 2012 (CET)
Auch komplizierte Sachverhalte kann man so erklären, dass Oma sie verstehen kann, bis zu einem gewissen Grad jedenfalls. Artikel enthalten nach dem Stichwort erst eine Definition und dann eine allgemeine Erläuterung. Im Bereich Mathematik und Physik wird diese Erläuterung aber oftmals weggelassen. Dafür gibt es bestimmt verschiedene Gründe, von denen die meisten aber nicht besonders ehrenwert sind (vergl. z.B. Diskussion:Impulserhaltungssatz, wo dem Leser nahegelegt wird, einige Semester Physik zu studieren, um den Artikel zu verstehen. Der Satz "Es ist üblich, dass bei manchen Dingen gesagt wird, um was es sich handelt - aber bestimmt nicht bei allem." zeugt ebenfalls von so einer Einstellung.) --Optimum (Diskussion) 19:32, 21. Mär. 2012 (CET)
Das Problem besteht darin, dass WP kein Lehrbuch ist und daher nicht alle Begrifflichkeiten, auf denen ein Mathe/Physik-Artikel aufbaut in der Einleitung erklärt werden können. Falls wir das täten würden a) Sehr viele Artikel massive Redundanzen aufweisen, b) 10 mal länger sein, c) keine Enzyklopädie sein.
Ich bin dafür jeden Einzelfall zu prüfen, falls Beschwerden kommen, dass etwas nicht unverständlich ist und dann versuchen zu verbessern. Leider habe ich bislang nur Kommentare wie „TOTAL UNVERSTÄNDLICH“ gelesen. Wenn man dann nachfragte, was denn genau unverständlich ist, dann kam keine Antwort. Leider kann ich dann auch nichts verbessern/vereinfachen. Also bitte bei solchen Vorwürfen nicht pauschal sage "alles zu schwer", denn damit kann kein Autor was anfangen. Bitte immer explizit sagen, was man nicht versteht. Dann kann der/die Autor(en) nämlich nachbessern.--svebert (Diskussion) 21:03, 21. Mär. 2012 (CET)
Sagen wir mal so. Man kann auch einen Matemathik-Artikel so schreiben, dass der Leser den Artikel zumindest soweit versteht, dass ihm klar wird, warum er ihn nicht versteht. Da hat Svebert schon recht, bisschen mehr als „TOTAL UNVERSTÄNDLICH“ sollte es schon sein, damit man ihn umarbeiten kann. --Bobo11 (Diskussion) 21:29, 21. Mär. 2012 (CET)
Meiner Beobachtung nach hat es auch mit Prioritäten zu tun, die man sich setzt. Und die scheint in der Mathematik (im Gegensatz zu etwas "bodenständigeren" Fächern wie der Physik) eindeutig auf Exaktheit zu liegen. Das führt dazu, dass ein Mathematiker einfach nicht anders kann, als schon in der Einleitung eine vollständige, alle Details und Nebenbedingungen erfassende Definition des Begriffs abzuliefern, ohne die leiseste Rücksicht auf Verständlichkeit. Wir hatten da schon diverse Diskussionen. In Physik und Technik überwindet man sich schon eher, in den ersten Sätzen auf Kosten der Präzision etwas allgemeinverständlicher zu formulieren, aber irgendwann später (oder auch schon im direkt folgenden Satz) muss man dann halt auch endgültige Exaktheit und Komplettheit zu erreichen versuchen, nur eben nicht gleich im allerersten Atemzug. --PeterFrankfurt (Diskussion) 02:53, 22. Mär. 2012 (CET)
Ja, da muss ich PeterFrankfurt zustimmen. Genau da liegt in der Regel das Problem bei den als unverständlich herüber kommenden Mathematikartikel. Da erschlägt man den Leser schon im ersten Satz mit Fachwörtern und Formulierungen die er bis heute noch nie gehört hat, und lässt ihn dann so im Regen stehen. Man versucht es schon gar nicht allgemein Verständlich zu erklären, um welches Gebiet der Mathematik es sich eigentlich handelt. DAS wäre meist schon ausreichend. Da man als normal Sterblicher mit der Experimentelle Mathematik o.Ä. nicht klar kommt, leuchtet einem in der Regel von selber ein. Nur sollte eben die Einleitung so geschrieben sein, dass man als Normalsterblicher begreift, dass es sich um ein "Nischen"-Thema für Fortgeschritene handelt. --Bobo11 (Diskussion) 09:42, 22. Mär. 2012 (CET)

Die ursprüngliche Kritik ist ja wenig konkret. Aber man könnte in der Tat bei jedem mathematischen Thema wenigstens die groben Zeitvoraussetzungen (etwa : "2 Jahre Mathe-Grundstundium, 1 Jahr Funktionalanalysis sind zum echten Verständnis notwendig") und dann eine etwas saloppe (ob das immer geht, weiß ich allerdings nicht; oft sind solche "Erklärungen" irreführend!) Erklärung anfügen.--Geometretos (Diskussion) 14:19, 22. Mär. 2012 (CET)

Davon würde ich dringend abraten. Bei einer guten Verlinkung der verwendeten Begriffe kann sich der Leser auch ohne Studium die Zusammenhänge anlesen. Mathematik hat oftmals keinen direkten Bezug zur stofflichen Welt, ist also nicht unbedingt "anschaulich". Bei euklidischer Geometrie kann man sich viele Sachen noch aufmalen, bei nichteuklidischer geht das schon nicht mehr. Zahlen mögen uns vom Zählen als intuitiv verständlich erscheinen (2 Äpfel plus 3 Äpfel sind 5 Äpfel), aber schon bei den negativen Zahlen oder der Null hörts auf mit den Äpfeln (von den komplexen Zahlen ganz zu schweigen).
Betrachten wir doch mal die Einleitung des oben erwähnten Artikels Atiyah-Singer-Indexsatz bezüglich seiner Verständlichkeit. Zuächst heißt es:
"Der Atiyah-Singer-Indexsatz ist die zentrale Aussage aus der globalen Analysis, einem mathematischen Teilgebiet der Differentialgeometrie."
Durch die Links kommt man jeweils zu Differentialgeometrie, von wo aus man wiederum zu Analysis und Geometrie weitergeleitet wird. Im Analysis-Artikel wird in der Einleitung auf Grenzwert (Folge), Folge (Mathematik), Reihe (Mathematik) und Funktion (Mathematik) verwiesen, also einigen wichtigen Basiswerkzeugen der Analysis. Bei Geometrie ist in der Einleitung von "Punkten, Geraden, Ebenen, Abständen, Winkeln etc." die Rede, man bekommt so einen Eindruck.
Übersetzt besagt der erste Satz der Einleitung also etwa, dass der Atiyah-Singer-Indexsatz eine zentrale Aussage eines Teilgebiets desjenigen mathematischen Fachgebiets ist, welches eine Synthese aus zwei mathematischen Teilgebieten darstellt, einem, welches sich mit Folgen, Reihen, Funktionen usw. beschäftigt und einem, welches sich mit Punkten, Geraden, Ebenen, Winkeln, Abständen usw. beschäftigt.
Nächster Satz:
"Er besagt, dass für einen elliptischen Differentialoperator auf einer kompakten Mannigfaltigkeit der analytische Index (eng verbunden mit der Dimension des Lösungsraums) gleich dem topologischen Index (über topologische Invarianten definiert) ist."
Uff. Der verlinkte Artikel verweist in seiner Einleitung auf Partielle Differentialgleichung, dort wird in der Einleitung wiederum auf Differentialgleichung und Partielle Ableitung verwiesen und erwähnt, dass partielle Differentialgleichung "der mathematischen Modellierung vieler physikalischer Vorgänge" dienen. Aber was soll "kompakte Mannigfaltigkeit" sein? Wäre Mannigfaltigkeit verlinkt, könnte man erfahren, dass es sich hierbei um einen "topologischen Raum, der lokal dem euklidischen Raum   gleicht", handelt. Wäre Kompakter Raum verlinkt, könnte man sich dort die Kompaktheits-Eigenschaften anlesen. Ähnliches gilt für "Dimension des Lösungsraums" und "topologische Invarianten": Bei Verlinkung auf Lösungsmenge#Lösungsraum, Dimension (Mathematik) und Topologische Invariante könnte man sich einlesen.
Soweit erst einmal. Mathematik besteht in einem streng strukturierten Begriffsgebilde, alle Begriffe sind durch weitere Begriffe definiert, bis hin zu ein paar elementaren Axiomen. Das kann man nicht vereinfachen, nur den Zugang ermöglichen. Gruß --stfn (Diskussion) 15:38, 22. Mär. 2012 (CET)
Habs mal dementsprechend verlinkt ... ist doch gleich viel verständlicher. ;) --stfn (Diskussion) 11:16, 23. Mär. 2012 (CET)

Küchenholz abdichten

Kann mir jemand sagen wie ich die Arbeitsplatte aus Holz in der Küche abdichten kann (zumindest für 3 Monate)? Denn wir haben schon einen Riss in der Arbeitsplatte neben dem Spülbecken und es tropft ausserdem runter.--Sanandros (Diskussion) 17:12, 21. Mär. 2012 (CET)

Nur der Rat, lass einen Tischler kommen. Wenn vieleicht Feuchtigkeit von der Spüle der Grund war, nützt kein "Flicken" des einen Risses etwas, denn wenn Spannung im Holz ist, reißt es dann nur an anderer Stelle erneut. Oliver S.Y. (Diskussion) 17:20, 21. Mär. 2012 (CET)
Wenn es wirklich nur vorrübergehend ist: evtl silikon oder diese zwei komponenten knetmasse, die es in gut sortierten baumärkten oder internetshops für die verschiedensten materialien (auch hol) gibt.--79.195.45.120 19:38, 21. Mär. 2012 (CET)
mit Silikon verschmieren war auch mein erster Gedanke, auch kann man es mit Epoxidharz (2-Komponentenkleber) vergießen (unten mit Klebeband abkleben,damit es nicht rausläuft). Knetmasse/Kitt wird nicht tief in einen schmalen Spalt eindringen. - Andreas König (Diskussion) 19:42, 21. Mär. 2012 (CET)
Wenn die Feuchtigkeit schon in das Holz reingezogen ist, nutzen Silikon und Epoxidharz wenig. Hier ist feuchtigkeitsbeständiger Holzleim gefragt. --Rôtkæppchen68 03:45, 22. Mär. 2012 (CET)
Arbeitsplatte trocknen lassen, mit einem Fön erwärmen, mit 2-Komponenten-Epoxidharz (zB. Uhu Endfest 300) bestreichen und den Riss füllen, dann wieder mit Fön erwärmen und anschliessend trocknenlassen und schleifen. Durch den Fön wird der Leim flüssig wie Wasser und zieht sehr gut ein. Alternativ kann man wie oben den Riss mit Epoxy und Sägespänen verfüllen und anschliessend die Platte mit Leinöl(firnis) oder Tungöl behandeln. Sooft, bis die Platte nichts mehr aufnimmt. Echholzplatten sollten immer mal wieder mit Öl behandelt werden, damit kein Wasser eindringen kann, denn sonst bilden sich die Risse. Grüße, --FirestormMD ♫♪♫♪ 08:25, 22. Mär. 2012 (CET)
@Oli: Den Tischler will meine Frau gerade nicht haben, da sie lernen muss.
@alle anderen: Ich hoffe ich finde in Kroatien auch einen "gut sortierten Baumarkt", wenn nicht werde ich mich noch mal melden. Ansonsten vielen Dank für die Tipps--Sanandros (Diskussion) 16:43, 22. Mär. 2012 (CET)

Figuren gruppieren in GeoGebra

Lassen sich in GeoGebra Figuren (Geraden, Kreise, Vielecke...) zu einem einzelnen Objekt gruppieren, so dass man sie gemeinsam verschieben oder kopieren kann? --KnightMove (Diskussion) 17:15, 21. Mär. 2012 (CET)

ich kenne GeoGebra nicht, aber hast du schon die Strg- und die Shift-Taste ausprobiert? --Duckundwech (Diskussion) 17:24, 21. Mär. 2012 (CET)
Ja, hat leider nicht funktioniert. --KnightMove (Diskussion) 17:34, 21. Mär. 2012 (CET)
[11] --89.204.154.140 17:46, 21. Mär. 2012 (CET)
Ok, jetzt hat es geklappt. Folgefrage, zuerst die Problemstellung präzisierend: Ich möchte ein Schachbrett zeichnen samt Schachfiguren. Wenn ich ein großes Quadrat zeichne und mittels Linien in kleine unterteile - wie kann ich die einzelnen Quadrate schwarz und weiß färben? --KnightMove (Diskussion) 17:43, 21. Mär. 2012 (CET)
Denkst du nicht, dass GeoGebra für diesen Zweck das falsche Programm ist? Ein Grafikprogramm (z. B. Inkscape) ist hierfür wohl besser geeignet. --Steef 389 19:33, 21. Mär. 2012 (CET)
Da hast du schon recht, aber wir müssen als Übung etwas zeichnen, und ich habe mich nun einmal für ein Schachbrett entschieden... --KnightMove (Diskussion) 22:00, 21. Mär. 2012 (CET)
Na dann: Schieberegler Füllung unter darstellung auf 100. --Steef 389 22:15, 21. Mär. 2012 (CET)
Danke, aber das färbt leider das gesamte Brett - ist es unmöglich, die einzelnen Quadrate zu färben? --KnightMove (Diskussion) 02:14, 22. Mär. 2012 (CET)
Du hast das Problem ja schon selbst beschrieben: Deine Figur besteht aus einem großen Quadrat und einer Reihe von Linien. Kleine Quadrate entstehen daraus nur in deiner Anschauung, aber nicht für das Programm. Dazu müsstest du auch im Programm kleine Quadrate aneinanderfügen. --Jossi (Diskussion) 09:13, 22. Mär. 2012 (CET)

Frage zu AntiVir

Hi, Leute. Wenn mein AntiVir nen Virus in Quarantäne verschoben hat, ist der dann harmlos? Und wann kann ich ihn bedenkenlos löschen? LG;--Nephiliskos (Diskussion) 18:27, 21. Mär. 2012 (CET)

Vielleicht hilft dir das hier weiter, ansonsten kannst du dort im Forum weitersuchen oder einen neuen Thread aufmachen. Gruß --Jogo.obb Disk / ± 18:36, 21. Mär. 2012 (CET)
Hmmmm... da stand was darüber, was man machen kann, wenn beim Systemcheck etwas gefunden wurde. Aber ich meinte folgendes Ereignis: während des Surfens im Internet kreischt mein AntiVir und das kleine Fund!-Fenster erscheint. Welche Aktion ist nun sinnvoller: "Zugriff verweigern" oder "In Quarantäne verschieben"? LG;--Nephiliskos (Diskussion) 18:46, 21. Mär. 2012 (CET)
Ganz klar: in Quarantäne. Da kommt die Datei in einen speziellen ordner, wird umbenannt und der Zugriff wird ebenfalls verweigert. Dann kannst du in Ruhe das Protokoll lesen, welche Datei es war. Wenn es keine wichtige Datei war: sofort löschen. War es eine wichtige Systemdatei, dann schleunigst ersatz suchen. War es ein wichtiges dokument von dir, dann irgendwo reparieren lassen. Da die meldung aber beim Surfen im Netz kam, ist es mit großer Wahrscheinlichkeit eine unnütze Datei aus dem Temp Ordner deines Browsers, die du getrost löschen kannst. Vollständige Systemprüfung im Anschluss würde ich dir raten! -- 208.48.242.106 09:19, 22. Mär. 2012 (CET)
Schließlich kann es auch sein, dass es eine wichtige eigene oder Systemdatei war, der Alarm jedoch falsch ist. Also ruhig etwas in der Quarantäne lassen, ein wenig googlen und sich umschauen, 1-2 Tage warten und dann (mit frisch upgedatetem Scanner) nochmal testen. Habe gelegentlich erlebt, dass es Falschmeldungen gab, die dann nach kurzer Zeit wieder wegwaren. --Wolli (Diskussion) 13:47, 23. Mär. 2012 (CET)

Kreatives zum Thema Salz

Ich wollte eigentlich nur schnell gugeln, wie ich Salzkristalle am geschicktesten mahle - das weiss ich jetzt auch schon. Dabei bin ich aber über diese Seite gestolpert: [12] Hier wird so frisch von der Leber weg Stuss verzapft, dass ich mich frage (es handelt sich schliesslich um Produktinformationen und Werbung), ab wann solche Falschaussagen eigentlich strafbar sind? --92.202.90.162 18:41, 21. Mär. 2012 (CET)

Ich fürchte, dass das leider nicht strafbar ist. Gibts aber auch in "richtig seriösen Geschäften": War in einem Geschäft einer internationalen Pafümerie-Filialkette und wollte eine Gesichtscreme für meine Freundin kaufen. Was mir die Verkäuferin dort an Esoterik unterschieben wollte, war nicht mehr feierlich. Da fielen so Begriffe wie "energetisierendes Siliziumextrakt", "DNA-Regeneration" und "von der Genforschung inspiriert"... Hee, bei Genforschung bin ich doch im Bilde ;) Auf mein interessiertes Nachfragen kam nur noch mehr Müll. Das hab ich ihr dann auch direkt so gesagt, wobei sie mich anschaute wie "na dann kaufs halt nicht" und ich bin gegangen. Der Laden war gut besucht, scheinbar braucht ein Großteil der Kundschaft so Stuss. --Hareinhardt (Diskussion) 19:38, 21. Mär. 2012 (CET)
„Von Zeit zu Zeit melden sich irgendwelche Pseudosachverständige, die versuchen – offensichtlich aufgrund einer Profilneurose – sich wichtig zu machen, [...]“ – und alles ist gesagt. --85.179.85.211 20:25, 21. Mär. 2012 (CET)
Tschuldigung, aber auf einer Esoterikseite sollte man ernsthafte naturwissenschaftliche Informationen ungefähr so vermissen wie das Lob Allahs im Atheistenverband, den Kampf gegen Autobahnen beim ADAC oder drei Prozent bei der F.D.P. Vor dem Stolpern Kontext anschauen... Kirschschorle (Diskussion) 00:48, 22. Mär. 2012 (CET)
Wenn sowas strafbar wäre, dann würde das Joghurt-Regal im Supermarkt inzwischen recht leer aussehen. -- 208.48.242.106 09:25, 22. Mär. 2012 (CET)
…und im Shampoo- und Conditioner-Regal wäre es auch sehr viel leerer. Am besten finde ich immer noch die im Fernsehen beworbenen Effekte französischer Dekorativkosmetika (mit Mikronanoblablaphytostammzellwasweißich). --Rôtkæppchen68 11:36, 22. Mär. 2012 (CET)
mich wundert, dass sich die ganzen engagierten Atheisten nicht viel mehr auf solchen Schmafu stürzen. Das ist doch in Wirklichkeit der neue religiöse Aberglaube unserer Zeit. Und die Kommerzialisierung dieses Aberglaubens ist der moderne Ablasshandel. Statt dessen liefern sie sich ständig ein Schattenboxen, mit den in Europa doch schon längst ausgezählten institutionellen Kirchen, was eher nervt. --El bes (Diskussion) 03:16, 23. Mär. 2012 (CET)

Webcam geht nicht mehr

Hallo liebe WP-Gemeinde, die eingebaute Webcam (PC-Cam) meines HP-Netbooks geht nicht mehr. Skype findet keine Webcam. Das Ding machte vor ein paar Wochen schon zwei oder drei mal Macken, nach einem Neustart jedoch war das Problem behoben. Heute wieder Neustart, dann leuchtete bei einem eingeleiteten Skype-Video-Call das Cam-LED für zwei oder drei Sekunden, und ebensolang konnte ich meine eigene Visage auf dem Monitor bewundern. Danach wieder nix mehr. Was ist los? Hardware failure, Treiberproblem, oder sonstwas? Wie/was könnte ich noch testen, um ggf. das Problem einzugrenzen? --Ratzer (Diskussion) 22:37, 21. Mär. 2012 (CET)

Erster Kandidat: Wackelkontakt, Abhilfe gerne schwierig. --PeterFrankfurt (Diskussion) 02:44, 22. Mär. 2012 (CET)
Ich würd das Gerät über den Gerätemanager mal ganz raushauen aus dem System und dann mit den mitgelieferten Treibern (oder alternativ mit neu heruntergeladenen) noch mal Schritt für Schritt installieren. Manchmal zerschießt Windoof einfach nur einen Treiber, das hab ich schon bei diverser anderer Hardware erlebt. -- 208.48.242.106 09:01, 22. Mär. 2012 (CET)
Bei meinem Medion-Laptop ging die eingebaute Kamera noch nie. Ich hab das aber erst gemerkt, als ich das Ding schon ein paar Jahre hatte. Ich hab mir dann für 10 € eine externe Kamera gekauft, die einwandfrei funktioniert. -- Gerd (Diskussion) 09:59, 22. Mär. 2012 (CET)
Dann kann es doch sein, dass sie zwei Jahre lang funktionierte (bzw. bei Gebrauch funktioniert hätte, was jedoch mangels desselben nie bemerkt wurde).--Ratzer (Diskussion) 10:15, 22. Mär. 2012 (CET)
Ja, kann auch sein. -- Gerd (Diskussion) 10:51, 22. Mär. 2012 (CET)

Videos am PC aufnehmen

Hallo Ihr Nachteulen,

ich bin technisch unversiert und benötige ein absolut idiotensicher zu bedienendes Programm zum Aufnehmen von audio- und audiovisuellen Medien, am besten freie Ware :-) für Windows XP.

Schönen Dank, --77.4.95.214 23:49, 21. Mär. 2012 (CET)

Für Audiodateien empfehle ich hier den Windows Sound Recorder, der bei Windows XP mit dabei ist. Für Videodateien mit oder ohne Ton gibt es Vidcap32, was seinerzeit bei Video for Windows mit dabei war. Dieses Programm ist bei vielen VfW-kompatiblen Webcams und TV-Karten mit dabei, gibt es aber von diversen Quellen auch zum runterladen. Sowohl Sound Recorder als auch Vidcap32 erlauben nur aufnahmen, keinen Schnitt. Bei Windows XP ist eine brauchbare Version vom Windows Movie Maker mit dabei, mit dem Videos auch geschnitten und mit Effekten versehen werden können. Zum Schneiden von Audiodateien empfehle ich Audacity. --Rôtkæppchen68 00:21, 22. Mär. 2012 (CET)
Wenn ich etwas dazu anmerken darf: IMHO passt "technisch unversiert" und "Videos aufnehmen" nicht zusammen. Je mehr Grundkenntnisse über Formate, Codecs, Komprimierungen etc. man hat, desto weniger Frust hat man beim Aufnehmen, Bearbeiten und Anschauen. --Plenz (Diskussion) 00:43, 22. Mär. 2012 (CET)
Eine etwas ausführlichere Frage wäre schön. Was willst Du denn aufnehmen? Bzw. und vor allem: Von wo (Mediathek, YouTube, eigene Kamera, eigene E-Guitarre...?) --Joyborg (Diskussion) 14:20, 22. Mär. 2012 (CET)
…und wie soll die Aufnahme verbreitet bzw gespeichert werden? DVD-Video, Bluray, Videoportal, Handyvideo, DV-Camcorder, VCD, VHS, … --Rôtkæppchen68 14:28, 22. Mär. 2012 (CET)
Also meistens von Youtube zur Festplatte oder Flash-Speicher --77.4.95.214 14:45, 22. Mär. 2012 (CET)
Ach so. Freie "Downloader" für youtube gibt es wie Sand am Meer. Die meisten wollen dir beim Installieren eine Browserleiste unterschieben. 84.177.31.47 14:57, 22. Mär. 2012 (CET)
Vergiss alles, was oben gesagt wurde und google nach flash ripper, flash ripping software, flv ripper, flv ripping software, youtube ripper oder youtube ripping software. Und hüte Dich vor Viren. --Rôtkæppchen68 15:04, 22. Mär. 2012 (CET)
Danke Euch allen! Ich probier mal YouRipper über chip.de. --77.4.95.214 15:40, 22. Mär. 2012 (CET)

Klangdatei gesucht

Gesucht wird eine qualitativ hochwertige Klangdatei vom Geräusch des Zuschlagens einer Frontlader-Waschmaschine, die vom Dateiformat her dazu geeignet ist, auf ein tragbares Mobiltelefon aufgespielt zu werden. Wo ist sowas zu finden? --91.56.173.101 07:35, 22. Mär. 2012 (CET)

Hier vielleicht? --Pilettes (Diskussion) 08:00, 22. Mär. 2012 (CET)
Oder eine allgemeinere Sound-Suchmaschine hier? --Asdert (Diskussion) 11:22, 23. Mär. 2012 (CET)

Wer war schonmal in Name? oder kennt die Orte?

In der BKL Johannisthal sind unter Polen 3 Orte aufgeführt, die früher einmal Johannisthal hießen, jetzt aber mit Name? betitelt sind. Wer kennt die Orte, wer kann es nachtragen? -- 208.48.242.106 10:19, 22. Mär. 2012 (CET)

Einen Ort konnte ich noch nachtragen, allerdings keinen der drei gesuchten. Wo kann man sowas recherchieren? -- 208.48.242.106 13:59, 22. Mär. 2012 (CET)
Wenn, dann mussen es neue polnische Namen sein (falls diese Orte nicht in anderen Gemeinden aufgegangen sind). Zu der Zeit, als die Orte noch auf deutschem Gebiet lagen (Ende 19. Jahrh.), wurden sie auch in polnischen Ortslexika nur mit dem deutschen Namen angegeben. Habe nachgesehen. GEEZERnil nisi bene 15:24, 22. Mär. 2012 (CET)

Habe jetzt mal über drei Ecken zwei weitere Johannisthals identifiziert: Im GenWiki kann man die entsprechenden Orte aufrufen, dort gibt es dann eine Verlinkung zu GoogleEarth und GoogleMaps und wenn man sich das anzeigen lässt, sieht man dort, wie der Ort heute heißt. Im Kreis Tuchel zeigt GenWiki allerdings kein Johannisthal an, das wäre also entweder ein Irrtum oder es ist lediglich ein Ortsteil und keine eigene Gemeinde gewesen. --88.73.159.93 16:04, 22. Mär. 2012 (CET)

Deine Vermutung ist nicht schlecht!
ich gebe dir in Letzt'rem recht.
Denn schaut man nach, mein lieber Sohn
steht im Gemeindelexikon:
  • Johannisthal, Wohnpl. in Deutschland, Kgrch Preussen, Prov. Westpreussen, Reg.-Bz. Marienwerder, Kr. Tuchel, AGer Tuchel, Ld-Gem. Iwitz, ev. Kspl Iwitz, kath. Kspl Grossbislaw, SdsAmt Grossbislaw, Amt Grossbislaw, (1895) 12 Gebäude, 76 Ew.
Quellennachweis:
Gem.-Lexikon Preussen, Bd. II (Prov. Westpreussen), Volkszähl. 2. Dez. 1895, Verlag der Königl. Statist. Bureaus, Berlin (1898), Seite 145
Da findet man - guckt man in allen -
Zu "Johannisdorf" ganz andre Zahlen.
So 20-30 waren vorhanden
in ehemalig doitschen Landen. GEEZERnil nisi bene 17:54, 22. Mär. 2012 (CET)
Iwiec (Iwitz) ist heute selber eingemeindet (polnische WP), also nicht wahrscheinlich, dass das frühere Johannisthal heut ein eigener Ort ist. GEEZERnil nisi bene 18:45, 22. Mär. 2012 (CET)
Na das sieht doch schonmal gut aus. für das letzte verbleibende habe ich auch keine idee mehr, außer denjenigen zu fragen, der es eingetregen hat um herauszufinden was seine Quelle war oder die genaue Lage einzugrenzen. -- Lord van Tasm «₪» 08:21, 23. Mär. 2012 (CET) P.S.: hab mal angefragt
Was man noch machen könnte: Alte Karte mit Iwitz suchen, Position bestimmen (das "Johannisthal" hatte etwa 1/5 der Grösse von Iwitz). So. Jetzt mit der Position in GoogleDings gehen und nachsehen, wie das heute aussieht. Vermutung: In der Obergemeinde von Iwiec aufgegangen. ("Heute Teil von ...").
Also, die alte karte hätte ich: [13], da sieht man die "Colonie Johannisthal" als einen Haufen verstreuter Häuser klnapp südlich von Wissoka. Auf wikimapia sehe ich da heute noch zwei oder drei Gutshäuser stehen. Kann daher gut sein, dass das heute gar keinen eigenen (Ortsteil-)Namen mehr besitzt. Diese Quelle, die sonst für solche Fragen recht ergiebig ist, kennt jedenfalls keinen. --88.73.26.2 11:34, 23. Mär. 2012 (CET)
Mehr fehlende Ortsnahmen ? ;-) GEEZERnil nisi bene 09:01, 23. Mär. 2012 (CET)
Hab ich gleich mal eingearbeitet. -- Lord van Tasm «₪» 09:52, 23. Mär. 2012 (CET)
Hier gabs noch eins -- Lord van Tasm «₪» 12:19, 23. Mär. 2012 (CET)
Hah! habs! [14], [15] heutiger Name aber weiterhin unbekannt. Alle anderen Daten stimmen aber. -- Lord van Tasm «₪» 12:22, 23. Mär. 2012 (CET)
Der Vergleich zwischen [16] und [17] zeigt mir, dass es die verstreuten Häuser auf diesem Feld sein müssen. eigener Ortsname unwahrscheinlich. -- Lord van Tasm «₪» 13:52, 23. Mär. 2012 (CET)

Für polnische Ortsnamen empfehle ich z.B. http://maps.geoportal.gov.pl/webclient/ (einfach mal mit den verschiedenen Kartentypen rechts oben und den Ebenen experimentieren). --тнояsтеn 14:30, 23. Mär. 2012 (CET)

Damit käme ich auf Mukrz oder Kochelno wobei das keine Orte sind. -- Lord van Tasm «₪» 14:50, 23. Mär. 2012 (CET)

Wie bringe ich CUPS Benehmen bei?

Situation: CUPS druckt auf Tintendrucker. Linux-Treiber dafür kam vom Hersteller und läuft ganz roll. Bunte Farben, Duplex, Seitenränder ok. Auf der CUPS-Testseite steht dann "Medienmaße:" gefolgt von den üblichen Zoll- und Millimeter-Maßen für A4, sowie eine weitere Zeile "Mediendruckgrenzen" in Zoll und Millimeter, etwas knapper bemessen, damit der Druck auch aufs Papier passt (weil ein Tintendrucker eben nicht komplett randlos drucken kann).

Drucke ich in den PDF-Drucker von CUPS, kann ich zwar die Medienmaße einstellen, aber nicht die Mediendruckgrenzen. (Beim Tintendrucker kann man sie übrigens auch nicht manuell über die GUI ändern, aber er macht ja auch alles schon per default richtig.)

Ich stehe nun vor der Situation, dass ich auch für den PDF-Druck Mediendruckgrenzen vorgeben können sollte, da das PDF direkt an den Drucker geschubst werden soll (das Gerät spricht nativ PS2, PS3, PCL5, PCL6 und PDF, was ganz schickes) , ohne Umweg über einen Adobe Reader oder anderes Anzeigeprogramm. Problem ist, dass der Drucker die Einstellung "Anpassen" bzw. "Übergroße Seiten verkleinern" nicht kennt. Ich muss also das PDF schon kleiner kriegen. Wie mache ich das? Kann ich dort auch irgendwie Mediendruckgrenzen vorgeben?

Ich habe schon versucht, die Seitengröße auf "Custom" zu setzen und den Millimeterwert abzuziehen, der den jeweils addierten Druckrändern entspricht, aber das scheint ihn nicht wirklich zu interessieren. CUPS meint nach wie vor, auf A4 zu drucken/pdfisieren (Erkennbar im Text der Testseite).

--88.67.153.22 11:00, 22. Mär. 2012 (CET)

hummelnest ist das verstellbar

 
Wie sieht die kleine Freundin aus? So? Dann wäre es eine Holzbiene, die auch im Holz "arbeitet". Wird der Stuhl noch gebraucht? Sonst einfach in eine Ecke stellen - und beobachten - und lernen. GEEZERnil nisi bene 13:02, 22. Mär. 2012 (CET)

kann ich das hummel nest verstellen und sie findet es wieder da ich den balkon machen wollte hab ich festgestellt das sie sich in einem alten holzstuhl eingenistet hat

--91.4.117.180 12:52, 22. Mär. 2012 (CET)

siehe bitte auch da. --gp (Diskussion) 13:08, 22. Mär. 2012 (CET)
siehe bitte auch da. grüße, --inuit - institut 22:15, 22. Mär. 2012 (CET)

Findet mit Sicherheit Ihr Nest wieder.Wir hatten einmal in Korsika einen Holzstock mit mehreren dieser Holzbienen um etwa 300 m versetzt, um ihn als Camping Sitz zu verwenden. Am nächsten Morgen waren wir "umschwärmt"von den nicht ganz zufriedenen Wohnungsinhabern. Um den Frieden wieder herzustellen, haben wir den Stock an seinen alten Platz zurück getragen. (nicht signierter Beitrag von 92.105.54.154 (Diskussion) 11:04, 23. Mär. 2012 (CET))

Suche biogr. Infos. --77.183.117.47 16:30, 22. Mär. 2012 (CET)

Hast du uns mal ein paar Stichworte mit denen wir uns beschäftigen können?--Sanandros (Diskussion) 16:46, 22. Mär. 2012 (CET)
Geburtsjahr und Schulort bietet der DNB-Eintrag. Ralf G. Diskussion 19:39, 22. Mär. 2012 (CET) Erg.: In seiner Dissertation gibts auf der letzten Seite einen auf Latein geschriebenen Lebenslauf (bis 1903). Ralf G. Diskussion 19:54, 22. Mär. 2012 (CET)
Oder ist dieser Benutzer der englischsprachigen Wikipedia gemeint? Ralf G. Diskussion 19:43, 22. Mär. 2012 (CET)

Speichererweiterung am Netbook selbst vornehmen

Es gibt Anweisungen (bspw. computerbild), wie man den Arbeitsspeicher bei Netboks selbst erweitern kann (mit Sicherheitshinweisen: Akku ab, sich selbst erden usw.). Trotzdem warnt der Hersteller (bspw. MEDION) vor einer eigenhändigen Durchführung. Richtet sich diese Warnung an alle, oder nur an solche, die auch vorhatten, ihre Katze in der Mikrowelle zu trocknen, oder den Fön beim Tauchen zu benutzen? Andersrum, kann jemand davon abraten?. Danke --Wikiseidank (Diskussion) 13:44, 22. Mär. 2012 (CET)

Zwei Gründe: (a) Sie würden dieses Business gerne selber haben und (b) sie können bei "Verletzung der Integrität" des Rechners - und evtl. erforderlichen Reparaturkosten für Karl-Heinz nach erfolglosem Rumbasteln - darauf hinweisen, dass sie jetzt bezahlt werden müssen, da die Garantievereinbarungen verletzt wurden (s.a. Sheldon in einer der frühen Big-Bang-Episoden). GEEZERnil nisi bene 13:53, 22. Mär. 2012 (CET) P.S. wer Katzen badet verdient keine Mikrowelle...
Es könnte auch noch einen dritten Grund geben: bei manchen Netbooks sind keine normalen SO-DIMMs verbaut sondern der RAM ist direkt auf dem Mainboard verlötet. Daher sollte Wikiseidank am besten nach [Modellnummer des Netbooks] arbeitsspeicher erweitern (o.Ä.) googeln. --MrBurns (Diskussion) 18:05, 22. Mär. 2012 (CET)

Switch mit Port Trunking gesucht

Hi, ich suche gerade einen 8, 10 oder 12-Port GBit Switch der Port Trunking kann... geeignet wäre der Dell PowerConnect 2808, aber mit 220€ ist mir der n bisschen zu teuer... kennt jemand eine günstigere Lösung?

--188.174.44.221 15:46, 22. Mär. 2012 (CET)

Translation Movement - Arabistik

Wie lautet der deutsche Ausdruck für: Translation Movement - siehe: http://en.wikipedia.org/wiki/Translation_Movement?

Habe die Doku "Science and Islam: Part 1: The Language of Science" (http://www.youtube.com/watch?v=TPlaS_wGzx8& ) gesehen und würde mir gerne deutschsprachige Literatur besorgen. Falls jemand etwas vorschlagen kann... bitte gerne. --80.136.48.173 18:45, 22. Mär. 2012 (CET)

Man kann auch in der Wikipedia anfangen: Haus der Weisheit, Graeco-Arabica, Abbasiden, Blütezeit des Islam, Christliche Wissenschaftler im Orient, Al-Kindi (Philosoph), al-Farabi, Almagest, Avicenna, al-Ghazali, usw. usw. Was übrigens in diesen Fernsehdokus oft vergessen wird zu sagen ist, dass diese kulturelle Hochblüte des Islam sowohl in Mesopotamien, als auch in Syrien, Ägypten und Spanien fast ausschließlich unter schiitischen Herrschern existiert hat. Die verschiedenen Spielarten der Schia dominierten den Islam auch bis ins 12. Jahrhundert. Ab dem 12. Jahrhundert wurden dann viel antikes Wissen, das in arabischen Schriften tradiert worden ist, in Spanien, aber auch in Italien und in Byzanz wieder ins Lateinische bzw. Griechische rückübersetzt und so dem Okzident zugänglich gemacht, siehe Übersetzerschule von Toledo. Das war dann in Europa der Beginn der Scholastik, die interessanterweise heute gar nicht mehr so gelobt wird, im Gegensatz zur islamischen Hochblüte, etwa bei den Mauren. Dabei war die Philosophie und die Wissenschaftstheorie in der christlichen Scholastik und in der maurischen Blütephase ziemlich genau die selbe. --El bes (Diskussion) 20:12, 22. Mär. 2012 (CET)
Du meinst sicher Iberische Halbinsel und nicht Spanien ;-) Für den Hinweis mit den "schiitischen Herrschern" bin ich dankbar. Und selbstverständlich hatte auch ich diese WP-Stichworte bereits aufgerufen... Aber bitte, wie lautet der deutschsprachige Begriff für "Translation Movement"? Hat denn keiner eine Idee?!--80.136.30.67 23:08, 22. Mär. 2012 (CET)
Ich meinte zeitgenössisch lateinisch Hispania bzw. arabisch Al-Andalus. Und "Translation Movement" dürfte gar nicht so ein stehender Begriff sein, wie es im Artikel in der en.Wikipedia den Anschein macht. In dem Sinn kann man sagen, dass auch das wörtliche deutsche Pendant "Übersetzungsbewegung" teilweise in der Literatur vorkommt, zb hier bei Rüdiger Lohlker: Islam. Eine Ideengeschichte. --El bes (Diskussion) 00:02, 23. Mär. 2012 (CET)

Um welches Theaterstück handelt es sich?

Unsere 82-jährige Mutter beschrieb uns vor einiger Zeit ein Theaterstück, in dem eine Szene vorkam, die sie sehr lustig fand: Der Kaiser führte ein Gespräch mit einem Soldaten. Der Soldat erzählte unentwegt von einer Wolldecke. Der Kaiser wollte das Gespräch in andere Bahnen lenken, aber der Soldat kam immer wieder auf die Wolldecke zurück.

Zugegeben, eine etwas dürftige Beschreibung. Aber vielleicht kann sich jemand erinnern, um welches Stück es sich handelt. Die Eingabe der Stichworte in Suchmaschinen brachte uns leider keine Hinweise. --Sentoku (Diskussion) 18:56, 22. Mär. 2012 (CET)

Wann hat sie es gesehen? Komödie oder ernstes Theaterstück? Das wird es nicht gewesen sein, aber statt "Kaiser" liegt natürlich Der Alte Fritz nahe ... GEEZERnil nisi bene 08:56, 23. Mär. 2012 (CET)
Erinnert mich an den alten Witz über den Biologiestudenten, der zur mündlichen Prüfung alles, wirklich alles über den Regenwurm gelernt hat, weil der Prof. Regenwurmexperte war. Dieser jedoch war am Prüfungstage erkrankt, sein Vertreter forderte den Studenten auf, über den Elefanten zu referieren. Der Student: "Die Elefanten bilden eine Familie der Rüsseltiere. Der Rüssel sieht einem Regenwurm in gewisser Weise ähnlich. Regenwürmer, lat. Lumbricidae, sind im Erdboden lebende, gegliederte Würmer aus der Ordnung..." -- Ian Dury Hit me  10:45, 23. Mär. 2012 (CET)
Oder es war der Napoleonwurm ... und die klassische Variante ist der Floh ! GEEZERnil nisi bene 11:24, 23. Mär. 2012 (CET)

Nähere Information erbeten. War dies der Komponist des Triumphmarsch's der NSDAP? --77.24.77.99 22:44, 22. Mär. 2012 (CET)

Selber nicht gefunden... ? GEEZERnil nisi bene 23:13, 22. Mär. 2012 (CET)
Da steht aber auch nnnichts von einem Triumphmarsch der NSDAP...was soll das eigentlich sein?--Pilettes (Diskussion) 23:17, 22. Mär. 2012 (CET)
dieses Heil Hitler Dings => Google => Bruno Schestak heil hitler <= => => hier. GEEZERnil nisi bene 23:19, 22. Mär. 2012 (CET)
Aber was hat das hiermit zu tun? --Pilettes (Diskussion) 23:25, 22. Mär. 2012 (CET)
Hast recht, scheint etwas anderes zu sein. Komponist nicht gefunden. GEEZERnil nisi bene 23:35, 22. Mär. 2012 (CET)

Grafik

Kann Otto Normalverbraucher so eine Grafik am PC selbst erstellen und mit welchem Programm z.B.? --93.133.196.176 08:23, 23. Mär. 2012 (CET)

Mit jedem Grafikprogramm. --AndreasPraefcke (Diskussion) 08:30, 23. Mär. 2012 (CET)

Für ein Logo (das z.B. als Druckvorlage dienen soll) empfiehlt sich ein Vektorgrafikprogramm, z.B. das kostenlose Inkscape. --Neitram 09:33, 23. Mär. 2012 (CET)

Fehlermeldung - Email

Ich lasse Emails von einer GMX- auf ein WindosLive-Adresse weiterleiten. Jeder der mir an die GMX-Adresse eine Nachricht schickt bekommt folgende Fehlermeldung: Hi. This is the qmail-send program at mx0.gmx.net. I'm afraid I wasn't able to deliver your message to the following addresses. This is a permanent error; I've given up. Sorry it didn't work out. <xxxxxx@live.de>: 65.55.92.136_does_not_like_recipient./Remote_host_said:_550_Requested_action_not_taken:_mailbox_unavailable/Giving_up_on_65.55.92.136./

Die Emails kommen trotzdem auf dem Live-Account an. Die Absender sind dennoch reglmäßig irritiert und rufen jedesmal an um nachzufragen. Ich will das Konstrukt GMX->Live jetzt auflösen und mich für einen der beiden Dienste entscheiden. Könnt ihr mir sagen, wer da Mist baut? Ein (kostenpflichtiger) Anruf beim GMX-Support hat nicht viel gebracht. Einerseits schien ihnen das Problem bekannt und sie sagten zu sich darum zu kümmern. Andererseits besteht das Problem auch 4 Wochen nach dem Anruf noch. --141.15.31.1 09:01, 23. Mär. 2012 (CET)

Was mir da spontan einfällt, ist die Option, deine GMX-Emails von Hotmail aus per POP3 abzuholen. Wenn du in Hotmail oben Rechts auf Optionen -> Weitere Optionen -> "E-Mails von anderen E-Mail-Konten aus senden/empfangen" -> "E-Mail-Konto hinzufügen" und dann den Anweisungen folgen. So hebelst du das Weiterleiten in GMX komplett aus und Hotmail macht die ganze Arbeit. Dabei sollten auch keine Fehlermeldungen gesendet werden. MfG --Maxkhl (Diskussion) 11:03, 23. Mär. 2012 (CET)

wieviel klamotten darf ich aus china einführen

--195.74.44.29 09:50, 23. Mär. 2012 (CET)

Am besten mal beim Zoll nachfragen! Thogru Sprich zu mir! 09:56, 23. Mär. 2012 (CET)

Es gibt eine Wert-Obergrenze für zollfrei einführbare waren. Ich vermute mal dieser wird zur Anwendung kommen. --Bobo11 (Diskussion) 09:58, 23. Mär. 2012 (CET)
Die Frage wird Dir auch beim Zoll so niemand beantworten können. Oder es werden mit großer Sicherheit diverse Fragen gestellt: willst Du die privat bei der Rückkehr von einer Reise mitbringen, oder gewerblich importieren. Um was für Textilien handelt es sich, kennst Du ggf. schon den Warencode? Handelt es sich um Produktpiraterie oder "normale" Textilien? Nur damit wirst Du eine seriöse Antwort bekommen (übrigens geben auch Speditionen diese Auskünfte). Der andere, wesentliche Punkt, bei dieser Art von Fragen ist doch wohl eher, "Wieviel Klamotten" man zoll- bzw. steuerfrei aus China einführen darf. Und da ist wie gesagt, der Weg und der private Charakter bedeutsam, für gewerbliche Einfuhren ohne Kontingente kannst eigentlich immer Steuern einkalkulieren.Oliver S.Y. (Diskussion) 10:03, 23. Mär. 2012 (CET)
Soweit ich weiß - alle Angaben ohne Gewähr - können einem gefälschte Produkte ja auch abgenommen werden, auch wenn man sie legal erworben hat, oder? --Eike (Diskussion) 10:11, 23. Mär. 2012 (CET)
Richtig. --Bobo11 (Diskussion) 10:11, 23. Mär. 2012 (CET)
Wer sagte denn etwas von gefälscht? :) Einfach bei dem Zoll anrufen. Damit habe ich mehrfach sehr gute Erfahrung gemacht. Die beraten nämlich lieber, als hinterher dem Papierkram am Hals zu haben wenn es mal wieder einer verbockt. --85.181.219.233 10:19, 23. Mär. 2012 (CET)
Am besten mal beim Zoll nachfragen! GEEZERnil nisi bene 11:00, 23. Mär. 2012 (CET)

Lederimprägnierung auf Parkett

Hi zusammen,

meine (ansonsten wirklich!) bessere Hälfte hat versehentlich den guten Parkettboden mit einer Wolke ihres silikon- und wachshaltigen Leder-Imprägniersprays getroffen. Seitdem ist der Boden dort fies rutschig, und es quietscht albern beim Drüberlaufen (sogar mit Socken). Dreimaliges Wischen mit Neutralreiniger hat nicht geholfen. Weiß jemand Rat (à la Oma hats gewusst), wie man den Siff-Film vom Holz kriegt? Klassischer Silikonlöser ist übrigens nicht im Hause.

Vielen Dank und beste Grüße —[ˈjøːˌmaˑ] 19:29, 20. Mär. 2012 (CET)

bin kein chemiker, aber: aceton? isopropanol? kann aber auch sein, dass es noch schlimmer wird... chemie kann auch nach hinten losgehen... besonders wenn man die chemikalien aus dem hörsaal meiner schule nimmt (irgendwie stimmten wohl die etiketten nicht wirklich...)... :-) --Heimschützenzentrum (?) 20:14, 20. Mär. 2012 (CET)
<quetsch>Während meiner Schulzeit hat es an einer anderen Schule mal ein Explosionsunglück gegeben, weil jemand Benzol aus der Chemikalienflasche geklaut hatte und zur Tarnung mit Methanol aufgefüllt hatte. Bei der Nitrobenzolsynthese ist dann natürlich auch Methylnitrat entstanden und prompt explodiert. --Rôtkæppchen68 21:20, 20. Mär. 2012 (CET)
Ich würde es mit einem unpolaren Lösungsmittel versuchen, also Waschbenzin oder Verdünnung. Die Versiegelung des Parketts ist wahrscheinlich danach hin. --Rôtkæppchen68 20:25, 20. Mär. 2012 (CET)
Wenn Silikonöl erstmal in ein Material eingesickert ist, kriegt man es praktisch gar nicht mehr raus. Vermutlich hilft auf einer intakten Versiegelung aber Scheuern mit einer holzspänebasierten, sandlosen Handwaschpaste, z.B. von Holste. -- Janka (Diskussion) 20:46, 20. Mär. 2012 (CET)
Da hast du echt die A..karte gezogen. Der Vorschlag des Vorredners läuft auf eine mechanische Enfernung hinaus, Anlagerung des Öls an die Putzkörper. Falls es klappt Glückwunsch. Der Artikel Silikonöle weist auf die Lösungsmittel hin, darunter zyklische Kohlenwasserstoffe (z.B. in Verdünner), aliphatische Kohlenwasserstoffe (in Reinigungsbenzin enthalten) und chlorierte Kohlenwasserstoffe z.B. Trichlorethen. Das mildeste davon ist Reinigungsbenzin, das könntest du mit einem Wattestäbchen an unauffälliger Stelle auf Lackverträglichkeit testen. Die anderen Stoffe sind entweder krebserzeugend oder giftig und sind auf jeden Fall so aggressiv, dass sie die Versiegelung des Parketts angreifen. Verdünner und Tri sind oder waren gängige Mittel zur Enfernung von Farb- und Lackschichten. Als Ultima Ratio bleibt nur noch Boden abschleifen und neu versiegeln. --Giftzwerg 88 (Diskussion) 21:05, 20. Mär. 2012 (CET)

Ruf mal da an, die kennen sich mit Siliconölen und -wachsen aus. --82.113.99.8 21:14, 20. Mär. 2012 (CET)

Troll-free Nummer, is klar. --46.115.22.178 21:49, 20. Mär. 2012 (CET)
Auch wenn du es lustig findest, mein Vorschlag war durchaus ernst gemeint. --89.204.153.151 22:11, 20. Mär. 2012 (CET)
Glaubst Du ernsthaft, ein Silikonhersteller macht Telefonsupport für ein Fremderzeugnis? Wenn schon, muss sich Benutzer:JøMa an den Originalhersteller des Lederimprägniersprays halten. --Rôtkæppchen68 23:30, 20. Mär. 2012 (CET)
Glaubst du ernsthaft, ein Lederimprägniersprayabfüller stellt die Komponenten selbst her? ;) --89.204.154.140 17:37, 21. Mär. 2012 (CET)
Nicht wirklich. Wenn überhaupt, dann macht der Silikonhersteller nur Second-Level-Support für das Produkt, das der Lederimprägniersprayoriginalhersteller herstellt. Du wirst als Lederimprägniersprayendkunde also um den Support des Lederimprägniersprayoriginalherstellers nicht herumkommen. --Rôtkæppchen68 18:15, 21. Mär. 2012 (CET)
Ich würds lassen. Lauft viel drüber, aber vorsichtig ;) . Chemische Behandlung machts nur schlimmer. Silikonöle können sich verflüchtigen, wahrscheinlich haben die das Abwaschen schon nicht überstanden. Das Wachs läuft sich ab, warscheinlich schneller als die Parkettversiegelung. Gegen die Glätte hilft vielleicht schon scheuern mit der rauen Seite vom Abwaschschwamm. Oder halt mit der oben schon erwähnten Handwaschpaste. --Hareinhardt (Diskussion) 01:58, 21. Mär. 2012 (CET)
Wenn man mit Silikon arbeitet (z.B. Verfugt) nimmt man normalerweise Spüli als Trenner für die Finger, um zu verhindern das Silikon an den Fingern haftet. Ich würde es mit einfachen nur leicht verdünntem Spühlmittel, einem rauen Lappen und starker mechanischer Einwirkung versuchen. Damit wird zumindest das Parkett nicht angegriffen. Das Spüli wird nicht beim Lösen helfen. Aber möglicherweise verhindert es, daß sich das durch die rauen Fasern des Lappens gelöste Silikonöl, erneut anlagert bei dem Drüberwischen. --81.200.198.20 13:10, 21. Mär. 2012 (CET)

Silikonentferner - steht im Baumarkt bei den Silkionkartuschen - versuchen.... -- Andreas König (Diskussion) 17:44, 21. Mär. 2012 (CET)

Vielen Dank an alle, die mir Ratschläge dazu gegeben haben! Für den heutigen Wocheneinkauf hätte Silikonentferner auf dem Einkaufszettel gestanden, aber in der Tat „verflüchtigt“ sich das Malheur zusehends. Noch eine Woche, und der Boden fühlt sich durch simple Weiterbenutzung wieder an wie vor dem Sprühunfall – was mich sehr erleichtert.
Nochmal vielen Dank und beste Grüße —[ˈjøːˌmaˑ] 13:58, 24. Mär. 2012 (CET)
Archivierung dieses Abschnittes wurde gewünscht von: тнояsтеn 20:32, 24. Mär. 2012 (CET)

Einkommenssteuer

Hallo, ich hab gehört, dass zwecks Lohnsteuerjahresausgleich einige Veränderungen mit 2012 eingetreten sind. Muss ich noch eine Einkommenssteuer abgeben? Vielen Dank, LG Uschi

--77.20.198.149 10:58, 23. Mär. 2012 (CET)

Einkommenssteuererkärung

Hallo, muss ich eine o.g. Erklärung machen? Danke sehr-Uschi

--77.20.198.149 11:02, 23. Mär. 2012 (CET)

Kommt drauf an. --Eike (Diskussion) 11:30, 23. Mär. 2012 (CET)
Stimmt, kommt auf deine Einkommenssituation und weitere Faktoren drauf an. Siehe auch hier -- Lord van Tasm «₪» 11:32, 23. Mär. 2012 (CET)

Vielen Dank, worauf kommt es an? Ich bin Rentnerin und mein Mann, 58, arbeitet auf Lohnst.kl.3. (nicht signierter Beitrag von 77.20.198.149 (Diskussion) 12:18, 23. Mär. 2012 (CET))

Für Rentner habe ich das hier gefunden: http://www.forium.de/redaktion/steuererklaerung-rentner-pflicht-rentenfreibetrag-gesetzliche-rente-freibetraege/ --Eike (Diskussion) 12:48, 23. Mär. 2012 (CET)

Geh zum Finanzamt und frage dort nach. Dort wirst du fachmännisch beraten. --тнояsтеn 14:05, 23. Mär. 2012 (CET)

Ok, vielen Dank! (nicht signierter Beitrag von 77.20.198.149 (Diskussion) 14:21, 23. Mär. 2012 (CET))

Archivierung dieses Abschnittes wurde gewünscht von: тнояsтеn 20:34, 24. Mär. 2012 (CET)

American Football

Warum ist der Spielball im American Football eiförmig? --E-W (Diskussion) 16:06, 19. Mär. 2012 (CET)

Ohne das überprüft zu haben, vermute ich mal, zusammen mit der Rotation, die die Spieler dem Ball verpassen, ist die eiförmige Form schlicht besser geeignet, große Weiten beim Wurf zu erzielen, als die Kugelform -- Vinceres (Diskussion) 16:10, 19. Mär. 2012 (CET)
Laut unserem Artikel Football (Sportgerät) stammt der Ball vom Rugbyball ab un der hat laut im Abschnitt Rugbyball#Weblinks verlinkter Website [18] seine Form von der Schweinsblase. --Rôtkæppchen68 16:37, 19. Mär. 2012 (CET)
+1 Blase "plum-shaped" Gib mich die Pflaume Kirsche ... !. GEEZERnil nisi bene 18:22, 19. Mär. 2012 (CET) P.S. Das Link erklärt auch, warum man heute Ballpumpen verwendet...
Es gibt aber auch die Theorie, dass die Bälle durch Verformung der Gummiblasen, die ab ca. 1862 verwendet wurden, entstanden ist. Siehe Weblink von Rôtkæppchen68 (Abschnitt Richard Lindon, weiter oben wird auch die Theorie mit der Form der Blase erwähnt). --MrBurns (Diskussion) 21:13, 19. Mär. 2012 (CET)
PS: hier werden auch die Gummiblasen als Verursacher der Form bezeichnet: [19] --MrBurns (Diskussion) 21:19, 19. Mär. 2012 (CET)
<Klugscheiß/> Die Form dieser Bälle ist keineswegs "eiförmig" oder oval, egal was der eine oder andere Sporthistoriker behaupten mag, sondern elliptoid bis spindelförmig.</Klugscheiß> Ugha-ugha (Diskussion) 21:39, 19. Mär. 2012 (CET)
@MrBurns, diese Theorie halte ich für unglaubwürdig. Gummiblasen sind elastisch, nehmen also jede Form an und der Ball kann getreten werden, ohne dass die Blase platzt. Bei unregelmäßig geformten Schweinsblasen, die zudem nicht bis wenig elastisch sind, kann es eher passieren, dass die Blase durch ungleichmäßge mechanische Spannung platzt, wenn der Ball außenrum nicht zu der Form passt. --Rôtkæppchen68 23:47, 19. Mär. 2012 (CET)
Du musst auch bedenken, dass Gummi wie er Mitte des 19. Jahrhunderts eingesetzt wurde heute kaum noch eingesetzt wird, damals hat man Naturkautschuk-Arten verwendet, heute werden eher Kunststoffe verwendet. Weißt du auch, wie die Eigenschaften von Naturlatex sind? Jedenfalls steht unter Naturkautschuk#Physikalische_Eigenschaften Unvulkanisierter Kautschuk ist viskoelastisch, unter einer länger anliegenden Kraft verformt er sich dauerhaft und geht nach dem Ende der Krafteinwirkung nicht vollständig in seine ursprüngliche Form zurück. Das macht die Theorie ebenso plausibel. Vulkanisation wurde schon 1839 erfunden, aber das heißt nicht unbedingt, dass sie 1862 in Rugbybällen verwendet wurde, es ist durchaus möglich, dass diese Methode den Herstellern von Rugby-Bällen damals noch unbekannt war. Wissen verbreitete sich im 19. Jahrhundert noch recht langsam. Außerdem ist vulkanisierte Kautschuk wegen dem Energiebedarf wohl teurer als unvulkanisierter. Ich denke man kann es dabei belassen, dass es zwei Theorien gibt und für beide Quellen. --MrBurns (Diskussion) 15:20, 22. Mär. 2012 (CET)
Man kann ihn einfach viel besser fangen als wenn er ganz rund wäre.--79.251.107.152 12:48, 24. Mär. 2012 (CET)
Unfundierte Idee: Man könnte auch durchaus wollen, dass das Spiel durch eine ungünstige Form erschwert wird. Damit nicht jeder Depp jeden Ball fangen kann, sozusagen. --Eike (Diskussion) 12:56, 24. Mär. 2012 (CET)

Beweise beim KPD-Verbot

Wegen des Anstehende NPD-Verbotsverfahren wollte ich mich mal beim KPD-Verbot wissen welche Beweise denn zum Verbot geführt haben, nur leider habe ich da nichts gefunden. Kann mir sonst jemand sagen was für Beweise in dem Verfahren verwendet wurden und wie die beschafft wurden, ausser durch die Hausdurchsuchungen?--Sanandros (Diskussion) 16:36, 22. Mär. 2012 (CET)

Da nichts gefunden? --Eike (Diskussion) 16:46, 22. Mär. 2012 (CET)
(BK) Die gewichtigsten Beweise hat die KPD in ihrem Parteiprogramm, ihrem Aufruf zum Sturz der Adenauer-Regierung und ihrer engen Zusammenarbeit mit der SED selbst geliefert. --79.224.231.220 16:47, 22. Mär. 2012 (CET)
OK hab das was die IP gesagt hat in dem Dokument gefunden.--Sanandros (Diskussion) 18:59, 23. Mär. 2012 (CET)

Was war 1834 die Selbeck

Ich hab in einem Heiratseintrag von 1834 in Saarn (Stadtteil von Mülheim a.d. Ruhr) als Berufsbezeichnung Pförtner in der Selbeck gefunden. Nun wüßte ich gern, was damit gemeint ist. Es gibt zwar noch einen Ort Selbeck, der heute Ortsteil von Saarn ist, aber wenn hier nur der Ort bezeichnet werden sollte, würde das Pförtner in Selbeck lauten. So aber klingt es für mich, als wäre "die Selbeck" ein damals bekannter Begriff für eine bestimmte Einrichtung gewesen (ggf eine Fabrik, eine Schule, eine Verwaltung, oder etwas derartiges). Heute beispielsweise wird ein Einkaufszentrum so bezeichnet. Zuerst hatte ich vermutet, dass es sich um eine Zeche handeln könnte - aber ich hab schnell rausgefunden, dass der organisierte Bergbau in der Gegend erst nach 1850 begann (und da war der betreffende schon tot). Weiss hier jemand, worum es sich handelt? Chiron McAnndra (Diskussion) 13:54, 23. Mär. 2012 (CET)

Sicher, dass sich das nicht auf einen Ort beziehen kann? Hier verspricht einer, er wolle "den Wohlstand in der Lübeck fördern". Und vom zitierten Heiratsantrag sind wir sprachlich über 150 Jahre entfernt... --Eike (Diskussion) 13:58, 23. Mär. 2012 (CET)
+1 Auch sollte man nachsehen, wie weit der Beruf "Pförtner" in dieser Zeit ging. Das war sicherlich mehr als jemand, der in einem (1) Gebäude abschliesst. Pförtner in der [Ortschaft]] Selbeck GEEZERnil nisi bene 14:04, 23. Mär. 2012 (CET)
Selbeck hatte historsich den weiblichen Artikel, siehe hier: Der Kreutzberger, der Hof Anger, die Selbeck, die Orte Mündelheim, Reinheim, Serm und Angermund werden als zu diesem Gaue gehörig, in den bezogenen Urkunden benannt. (Die alte und neue Erzdiözese Köln in Dekanate eingetheilt. (1828) Kommt ja auch von Bek; auch der Ort Bäk heisst im regionalen Sprachgebrauch bis heute die Bäk. --Concord (Diskussion) 14:54, 23. Mär. 2012 (CET)
Hermann Jellinghaus erwähnt den Ortsteil "Selbecke" bei Obehundem/Olpe als "die Selbecke". GEEZERnil nisi bene 15:12, 23. Mär. 2012 (CET)
Also bei Lübeck bin ich mir ziemlich sicher, dass da nur jemand beim Texteditieren das Wort "Hansestadt" gestrichen und den Artikel vergessen hat. Aber ansonsten Zustimmung, mundartlich ist es sehr üblich, Ortsnamen mit Artikeln oder Präpositionen zu benutzen, die in der Standardsprache unüblich sind. Und bei über 150 Jahre alten Texten muss man immer damit rechnen, dass solche mundartlichen Erscheinungen mal in der Schriftsprache durchschlagen. --::Slomox:: >< 14:57, 23. Mär. 2012 (CET)
Ok, offenbar kann es also doch lediglich eine Ortsbezeichnung sein - schade, denn ich hatte gehofft, da einen Ansatz dafür zu finden, wo er Pförtner war. Laut Angabe war er damals 25 Jahre alt und war zudem ein Zugereister, der noch nicht sehr lange dort lebte - ich kann mir nicht so recht vorstellen, was das gewesen sein soll - meistens waren Pförtner ältere Leute, für die andere Arbeiten zu schwer waren, die aber eine gewisse Erfahrung und auch Verantwortungsbewusstsein hatten. Hat jemand ne Idee, wie man sich da weiter rantasten könnte, um die Stelle/Arbeitgeber zu identifizieren? Chiron McAnndra (Diskussion) 15:11, 23. Mär. 2012 (CET)
Vermute, du hast ein Kirchenbuch konsultiert. Mal im Personenstandsarchiv (NRW) Brühl nachfragen, die Zivilstandsregister wurden rechtsrheinisch ab 1810 oder 1815 geführt, könnte also passen. --Aalfons (Diskussion) 15:21, 23. Mär. 2012 (CET)
Wo mehr erfahren? Standardantwort: Hier anmelden und freundlich fragen. GEEZERnil nisi bene 15:28, 23. Mär. 2012 (CET)
@Aalfons - ich hatte schon beim Stadtarchiv nachgefragt - von dort kam die Auskunft, dass die Personenstandsregister vor 1835 nicht alphabetisch erfasst seien und man hatte mir empfohlen, deshalb Kirchenbücher zu konsultieren. Da ich von der Arbeit in solchen Archiven recht wenig weiss, kann ich diese Antwort nicht einschätzen - entweder heisst das, die Leute hatten einfach keine Lust, zu suchen, oder aber es ist aussichtslos, das gesuchte zu finden. Wenn sie nicht alphabetisch erfasst sind - wie sind sie denn sonst sortiert? Chiron McAnndra (Diskussion) 18:18, 23. Mär. 2012 (CET)
Chronologisch? --89.204.153.144 18:45, 23. Mär. 2012 (CET)
...und man muss lernen, die Handschrift des Herrn Pfarrers zu lesen. Deshalb mein Hinweis auf die Mailing-Liste. Da hocken Leute, die bereits KB verkartet haben. Wenn du dann gefunden hast, was du suchst, weisst du ganz genau, welche Seite du für die Unterlagen kopieren musst (wenn du überhaupt ans Kirchenbuch 'rankommst). GEEZERnil nisi bene 23:04, 23. Mär. 2012 (CET)

Spülmittel

Ist es gefährlich, Spülmittel zu sich zu nehmen? (nicht signierter Beitrag von 85.180.147.109 (Diskussion) 14:58, 23. Mär. 2012 (CET))

Das kommt (wie immer) auf die Menge an. Spuren von Spülmittel an gespültem Geschirr sind völlig ungefährlich; eine Pulle voll Spülmittel dagegen würde ich nicht trinken wollen. Warum fragst du? --Snevern 15:03, 23. Mär. 2012 (CET)
Ich hab gerade nochmal auf der Palmoliveflasche nachgeschaut, kein einziger Gefahrenhinweis. Lediglich bei Augenkontakt soll man mit Wasser spülen, damit sie rein werden^^. Aus eigener Erfahrung mit mehrere Sorten, nein, es ist nicht gefährlicher, Spülmittel zu sich zu nehmen, als andere Chemikalien. Es kommt immer auf die Menge, die Umstände und den an, der es zu sich genommen hat. Jemand mit einer Vorerkrankung oder ein Kind kann vieleicht das Spülmittel nichts anhaben, aber das damit meist verbundene Erbrechen oder der Durchfall sind gefährlich, wenn sie nicht beobachtet und bekämpft werden.Oliver S.Y. (Diskussion) 15:14, 23. Mär. 2012 (CET) Achso, das ist natürlich ein völlig laienhafter Gedanke, weder verbindlich, noch soll er zum Verzehr von Spülmittel anregen.Oliver S.Y. (Diskussion) 15:15, 23. Mär. 2012 (CET)
So ganz harmlos ist das wohl nicht. Jedenfalls finden sich problemlos Artikel wie dieser.--Optimum (Diskussion) 15:28, 23. Mär. 2012 (CET)
Selbst wenn das Spühlmittel nicht direkt tödlich wäre, kann es zur Schaumbildung kommen die eine Erstickung zur Folge haben kann. Wenn man daran nicht erstickt, kann trotzdem die Lunge geschädigt werden wenn der Schaum eingeatmet wird. --FNORD (Diskussion) 15:38, 23. Mär. 2012 (CET)
Entschuldigung, aber das ist haltloses Gewäsch einer Pseudoexpertin, die sich auf eine einzelne Tierstudie beruft. Nicht zu vergessen die Frage, von was man als Spülmittel spricht. Ich hab jetzt darunter normalen Geschirrspüler von menschlichem Geschirr, und nicht Spezialreiniger in der Tiermast verstanden. Was die Rückstände angeht, so nimmt man die als Mensch auch permanent auf, denn es bleibt immer etwas auch an sauberen, bzw. reinem Geschirr hängen, wenn man es wie allgemein üblich im Spülwasser wäscht, abtrocknet, und in den Schrank stellt. Denn "Klarspülen", wie früher üblich, betrachten viele doch als überflüssig, oder machens nur bei Gläsern, eben wegen der dort besser sichtbaren Rückstände. Anders gefragt, wer hat noch nie nen Schluck Lauge beim Seifenblasenpusten geschluckt, oder wie oft geschieht es doch im Haushalt, das Spülmittel in einem Geschirrteil oder Flasche verbleibt, und dann getrunken wird. Wer natürlich ein Schnapsglas Pril auf Ex wegen ner Kneipenwette trinkt, hat selbst Schuld, aber bekommt keinen Reizdarm von diesem einen Mal.Oliver S.Y. (Diskussion) 15:37, 23. Mär. 2012 (CET)
Handspülmittel ist mit Sicherheit unbedenklich. Bei Maschinengeschirrspülmittel wäre ich mir da nicht so sicher. Da ist nicht umsonst eine Pinzette mit dabei. Gewerbliches Maschinengeschirrspülmittel enthält Natriumhydroxid und Natriumhypochlorit und ist damit ätzend und umweltgefährlich. Natriumhydroxid ist unter der Nummer E 524 Lebensmittelzusatzstoff. Ich möchte es aber trotzdem nur dann zu mir nehmen, wenn auf einer Brezel klebt, die nach dem Eintunken gebacken wurde. --Rôtkæppchen68 15:50, 23. Mär. 2012 (CET)
Unbestätigt, ich meine aus dem "Handbuch des unnützen Wissens" von Haefs: Jeder Mensch nimmt im Laufe seines Lebens ca. zwei Flaschen Spülmittel zu sich. Vermutlich aus der Zeit vor der massenhaften Verbreitung von Geschirrspülern. Gr., redNoise (Diskussion) 16:10, 23. Mär. 2012 (CET)
„Dosis sola venenum facit“ (deutsch: „Allein die Menge macht das Gift“) sagte schon Paracelsus vor Erfindung des Spülmittels ;-) --79.199.96.123 00:32, 24. Mär. 2012 (CET)

Was ist eine Edelweiß Pizza

--77.118.182.134 15:42, 23. Mär. 2012 (CET) Ich wollte gesteern eine Salami Pizza mit Mais bestellen. Als die Pizza fertig war, kam der Kellner und brachte sie mir. Dabei sagte er "Bitte sehr einmal Edelweiß Pizza" Jetzt möchte ich wissen was das beudeutet. In der Speise Karte steht auch PPizza Salami. DANKE

Vielleicht hat er ja gesagt: "Bitte sehr, einmal Ihre Mais-Pizza" -- ein Verhörer? --Neitram 15:56, 23. Mär. 2012 (CET)
Oder hat er eine Anspielung auf den weißen Edelschimmel gemacht, der eine echte Salami umgibt? -- 91.42.50.35 16:07, 23. Mär. 2012 (CET)

Schau dir ein Edelweiss an, von oben. Schau dir besagte Pizza an, von oben. Vergleiche die gelben Kügelchen. --178.196.103.86 16:12, 23. Mär. 2012 (CET)

Die Salami nennt man auch Edelwurst und -weiß klingt sehr ähnlich wie Mais.--77.116.160.2 17:35, 23. Mär. 2012 (CET)
Früher hätte man in so einer Situation einfach mal den Kellner gefragt, was es mit der Bezeichnung auf sich hat. Vielleicht hat er sie auch nur benutzt, um danach gefragt zu werden. Jetzt frage ich mich gerade, warum der Fragesteller nicht den Mund aufbekommen hat. Das lernten wir doch schon beim Sesamstraßenlied: "...wer nicht fragt bleibt dumm." --91.56.184.42 07:21, 24. Mär. 2012 (CET)
Vielleicht hatte der Kellner keine URL... --77.185.162.151 13:28, 24. Mär. 2012 (CET)

Probleme mit einem DVR Gerät

Ich habe (beim Chinesen in Honkong) eine Unterwasserkamera gekauft. (Kommt mit Koffer und Bildschirm, Batterie und 50 m Kabel). Da alles prima funktioniert, habe ich noch ein Aufzeichnungs-Gerät dafür bestellt. Das lässt sich aber nicht auf dem Laptop installieren. Auf Anfrage wurde ich daraufhin belehrt, dass das Gerät nur auf einer Maschine mit 32 bits läuft. Also XP oder Vista, ich Hab aber schon Windows 7. (64 bit) Die Firma hat keinen Driver für 64 bits. Kennt Jemand eine Möglichkeit,das „Hindernis“ zu umgehen? Wenn nicht, ist nicht schlimm, das Gerät kostete nur 37 USD. --92.105.54.154 18:29, 23. Mär. 2012 (CET)

5. und 7. Frage hier --89.204.153.144 18:52, 23. Mär. 2012 (CET)

Also, war nichts, schade, trotzdem Danke.Gruss aus CH (nicht signierter Beitrag von 92.105.54.154 (Diskussion) 19:11, 23. Mär. 2012 (CET))

Für Windows 7 Professional gibt es den Windows XP Mode, vielleicht funktioniert es ja damit. Wenn Du kein Windows 7 Professional hast, könntest Du es mit Windows Virtual PC versuchen, wenn Du noch irgendwo eine 32-Bit-Windows-Lizenz (XP, Vista, 7) rumliegen hast. --Rôtkæppchen68 20:45, 23. Mär. 2012 (CET)

Den Virtual PC hab ich heruntergeladen, er reagiert aber nicht bw er erkennt den 32 bit Treiber nicht. Auf meinem Desktop habe ich Vista, weiss aber wirklich nicht wie man das auf den Laptop rüberladen könnte, bin schon zulange aus dem Geschäft. (nicht signierter Beitrag von 92.105.54.154 (Diskussion) 12:47, 24. Mär. 2012 (CET)) Nachtrag, auch auf dem 32 bit System (Vista) lässt es sich nicht installieren, da muss noch was anderes sein. Ich nehme nochmal mit den Chinesen Kontakt auf. Danke schon mal für die Bemühungen. (nicht signierter Beitrag von 92.105.54.154 (Diskussion) 13:57, 24. Mär. 2012 (CET))

Schlüsselband

Warum sind Schlüsselbänder eigentlich immer verdreht? Im Artikel Lanyard heißt es: „Für einen besseren Tragekomfort wird das Band oft, damit es flach am Körper liegt, um eine halbe Umdrehung versetzt zusammengenäht.“Q, aber ich kann mir nicht vorstellen, was bei einem ungedrehten Band problematisch sein sollte. --92.229.207.205 23:57, 23. Mär. 2012 (CET)

Ich hasse die Verdrehungen wie die Pest, das sieht immer so falsch und hässlich aus. --94.134.211.95 00:11, 24. Mär. 2012 (CET)
Man stelle sich das einmal im Detail vor dem inneren Auge vor (geht so vielleicht sogar besser als mit einer Abbildung): Man nehme ein langes, flaches Band. Das lege man sich hinten horizontal auf den Nacken, dann führt man beide Stränge links und rechts vom Hals nach vorn und legt sie wie Hosenträger auf den Oberkörper. Man lässt sie etwas schräg zur Mitte laufen, damit sie sich etwa am Bauchnabel treffen und dort den Schlüssel tragen. Wenn man nichts verdreht, müsste man sie an dieser Treffstelle so umbiegen, dass der Schlüsselring praktisch senkrecht von der Körperoberfläche absteht. Das wäre noch unbequemer. Wenn man aber die beiden Bandenden schräg zusammenlaufen lässt und am Ende per Umklappen in der Ebene der Körperberfläche knickt, so dass der Schlüsselring auch flach und bequem aufliegt, dann hat das Band genau diese eine halbe Verdrehung zusätzlich. Geometrische Topologie lässt grüßen. --PeterFrankfurt (Diskussion) 02:52, 24. Mär. 2012 (CET)

Hm, das kann sein. Allerdings trage ich Schlüsselbänder so gut wie nie um den Hals, außer, wenn ein Konferenznamensschild dran ist, und die wiederum sind meistens quer zum Bandende befestigt, sodass man sich wünscht, das Band stände senkrecht zur Brust, weil dann das Schild glatt läge. So richtig sehe ich den Sinn also immer noch nicht…--92.229.207.205 10:43, 24. Mär. 2012 (CET)

Tragen wirklich Leute ihren Schlüsselbund um den Hals?--92.229.207.205 10:44, 24. Mär. 2012 (CET)

Schlüsselkind. Kirschschorle (Diskussion) 12:35, 24. Mär. 2012 (CET)

Windows 7 problem mit defrag

auf einem Windows 7 Rechner wurde aus versehn die defrag Datei gelöscht, eigentlich sollte aufn dem Desktop nur die Verknüpfung gelöscht werden, stattdessen wurde die Verknüpfung und die Programm Datei gelöscht. das komische ist das allem anscheint nach die defrag Datei noch da ist im system32 Ordner, wenn man versucht das Programm zu starten, kommt das eingabefenster sehr kurz und läst was durch laufen (geht so schnell das man es nicht mal anhalten kann)und ist wieder weg, das eigentliche defrag wird nicht ausgeführt. habe schon im netzt nach gesehen, konnte nichts Brauchbares finden um das Problem zu Lößen, hoffe man kann mir hier besser helfen ^^ --Diamant001 (Diskussion) 23:59, 23. Mär. 2012 (CET)

defrag.exe in System32 ist das Kommandozeilenprogramm. Du suchst dfrgui.exe im selben Ordner. Wenn du die wirklich gelöscht haben solltest (ich bezweifle es), hilft dir eine Windows-CD/DVD, sie wiederherzustellen. --Steef 389 01:03, 24. Mär. 2012 (CET)
(BK) Bei meinem Windows 7 Professional existiert die Datei defrag.exe zweimal. Einmal ist die unter C:\Windows\System32 zu finden, das andere mal unter C:\Windows\winsxs\x86_microsoft-windows-defrag-cmdline_31bf3856ad364e35_6.1.7600.16385_none_c75225df27a90f8d . Kopiere die Datei einfach aus dem Ordner, in dem sie noch existiert, in denjenigen Ordner, aus dem Du sie versehentlich gelöscht hast. Defrag.exe ist wohl die Kommandozeilenversion. Es gibt nämlich noch eine Datei dfrgui.exe, auf die der Startmenüeintrag Defragmentierung verweist. Die Datei dfrgui.exe findest Du in den Ordnern C:\Windows\System32 und C:\Windows\winsxs\x86_microsoft-windows-defrag-adminui_31bf3856ad364e35_6.1.7601.17514_none_9b1d78a9ee870c74 . Rest s.o. Je nach Windows-Version (Home, Professional, 32 bit, 64 bit), Sprache und Installationspfad können die Verzeichnisse abweichen. --Rôtkæppchen68 01:07, 24. Mär. 2012 (CET)
ich danke euch, werde die tage das bei der betreffenden person nach schauen, das es wieder geht mit euren hinweisen :-) melde mich dann wieder --Diamant001 (Diskussion) 01:15, 24. Mär. 2012 (CET)

Lohn-/Einkommensteuer bei der Ausbildung

Hallo! Ich wollte mal fragen, wie viel Steuer ich als 20-jähriger Single bei einer Ausbildung in Westdeutschland ohne Kirchensteuer zu zahlen habe, wenn ich _Brutto_ 520 € im ersten Jahr, 550 € im zweiten Jahr und 600 € im letzten Ausbildungsjahr verdienen würde.

--DasPortal (Diskussion) 10:34, 24. Mär. 2012 (CET)

Lohnsteuer und Kirchensteuer 0,00 €, allerdings Sozialbeiträge. Lohnsteuertabellen gibt es übrigens online in großer Auswahl. Grüße 85.180.196.206 10:45, 24. Mär. 2012 (CET)

Melodie aus Terra X von letztem Sonntag gesucht

Hallo zusammen, hier fragt jemand in Facebook, wie die Melodie heißt, die man in dieser Sendung ab 3:30 hören kann. Ich bin der Meinung, dass es relativ bekannt ist. Wisst ihr es? Gruß, --Flominator 11:02, 24. Mär. 2012 (CET)

Das ist von Dimitri Schostakowitsch, angeblich Suite für Jazzorchester Nr. 2, in Wirklichkeit aber Suite für Varieté-Orchester, Nr. VI, Walzer Nr. 2, bekanntgeworden auch bei Nicht-"Hardcore-Klassikfans" durch die Bearbeitung von André Rieu und den Soundtrack zu dem Film Eyes Wide Shut (1999) von Stanley Kubrick (so unser Artikel). Eine ungewohnte Aufnahme bei Youtube hier, mit Orchester gibt dort etliche. - Die Fassung bei Terra X scheint mir von Rieu zu sein. War übrigens eine klasse Sendung. --Bremond (Diskussion) 11:33, 24. Mär. 2012 (CET)
-> Liste der in der Dokureihe "Schneller als das Auge" verwendeten Musiktitel: http://superzeitlupe.zdf.de/ZDFde/download/0,6753,7026749,00.pdf --gp (Diskussion) 11:38, 24. Mär. 2012 (CET)
Korrektur: Liste der vewendeten Musiktitel <= Schneller als das vewendete proof-reading-Programm ... GEEZERnil nisi bene 11:52, 24. Mär. 2012 (CET)

Hundeausbildung

Guten Abend! Wie kann ich meinen 6 Monate alten Bullterrier am besten scharf machen ohne daß ich dem Tier Gewalt antue? DANKE 178.114.31.10 00:55, 20. Mär. 2012 (CET)

Mit einer Bullterrierin?! --77.188.159.169 01:04, 20. Mär. 2012 (CET)
Laut Schärfe (Kynologie) #Zucht und Abrichten auf Schärfe ist dazu eine bestimmte Ausbildung für Hund und Hundeführer erforderlich. Die Hundegesetze verbieten zunächst einmal, "Hunde auf Schärfe zu züchten und auszubilden. [...] Ausnahmen von diesem Verbot gelten für Diensthunde der Behörden. Die gewerbliche Ausbildung von Schutzhunden für Objekt- und Personenschutz und der Einsatz solcher Hunde sind genehmigungspflichtig." Sofern du also befugt sein solltest, müsstest du auch über das "Wie" informiert sein, falls nicht, ist das, was du vorhast, strafbar. Meine Meinung: Ein Haustier ist weder ein geeignetes Spielzeug noch eine geeignete Waffe. --213.39.224.170 01:13, 20. Mär. 2012 (CET)

Ich dachte dabei eher für Zwecke der Selbstverteidigung. 178.114.31.10 01:19, 20. Mär. 2012 (CET)

Na, dann musst Du nichts weiter tun. Die funktioniert bei den meisten Hunden instinktmäßig. Wenn er jetzt sechs Monate alt ist, dauert es auch nicht mehr lange, bis er sich im Notfall selbst verteidigt. Kirschschorle (Diskussion) 01:24, 20. Mär. 2012 (CET)
Ich weiß ja nicht, wie das in Niederösterreich ist, aber in Deutschland ist das so, dass viele Bundesländer und Gemeinden für Bullterriers und deren Halter Prüfungen vorschreiben und höhere Hundesteuern bis hin zu Strafsteuern erheben, sodass viele Menschen sich einen derartigen Hund nicht leisten oder halten können. Mit Ju-Jutsu bist Du wahrscheinlich besser bedient was Selbstverteidigung angeht. --Rôtkæppchen68 01:29, 20. Mär. 2012 (CET)
Sicherer ist es allemal. Wie unbedacht ist einmal "Fido, Fass!" befohlen und die Situation gerät völlig außer Kontrolle. Jetzt wird nämlich derjenige, der vom Hund angegriffen wird zur Selbstverteidigung eine Waffe ergreifen und den Hund töten. Und sich danach mit derselben Waffe, die praktischerweise gleich zur Hand ist, den Halter vornehmen. Exitstrategie für den Hundehalter? Keine.
Falls der OP hingegen vorhat, den Hund als Waffe gegen Wehrlose einzusetzen wünsche ich ihm, dass ihn die Töle selbst möglichst oft beißt. -- Janka (Diskussion) 03:25, 20. Mär. 2012 (CET)
Man möge dem Fragesteller bitte zu Gute halten, dass er sich scheut, seinem Tier zu Ausbildungszwecken Gewalt anzutun. Das sollte mal gesagt werden, denn mehr fällt mir in dieser Richtung nicht ein - und in der entgegen gesetzten Richtung wurde von den übrigen Antwortenden bereits alles gesagt. --Snevern 08:14, 20. Mär. 2012 (CET)

@ Fragesteller: Dein Bedürfnis nach Selbstverteidigung in allen Ehren, aber ein "Scharfmachen" dürfte sich erübrigen. Wer einen robusten Hund an der Leine (und dazu vielleicht noch einen kräftigen Wanderstock) mit sich führt, hat bereits ein deutliches psychologisches Abwehrsignal gesetzt. Solange Du Dich nicht im Dunkeln in ausgesprochenen Risikogebieten bewegen musst und auch keine "Opfer-Signale" aussendest, bist Du schon ganz gut geschützt. --Zerolevel (Diskussion) 11:00, 20. Mär. 2012 (CET)

Im Wald muss man sich eher selten selbstverteitigen und die Zeiten in denen man in der Stadt einen Wanderstock mit sich führt sind doch schon seit einiger Zeit vorbei. Der Hund reicht vollkommen aus, auch ohne Ihn scharf zu machen. Jeder weiß das man ein Problem mit dem Hund kriegt, wenn man sein Herrchen angreift. Aber man kann mit seinem Hund auch in eine Hundeschule gehen und Ihn dort beibringen auf Befehl Laut zu geben usw. Das dürfte viel effektiver sein, als eigenständig zu versuchen den Hund irgendwie "scharf" zu machen. --81.200.198.20 11:07, 20. Mär. 2012 (CET)
Der Wanderstock war nur eine legale Option, ein zusätzliches optisches Abwehrzeichen zu setzen: Ein normaler Spazierstock signalisiert Schwäche, ein Wanderstock signalisiert Fitness. Mit "die Zeiten sind vorbei" hat das nichts zu tun, Wandern ist auch eine Fitnessbewegung gerade in der 2. Lebenshälfte, und der Fragesteller hat nichts über sein Alter angedeutet - irgendwo zwischen 15 und 75 möglicherweise. ;-) --Zerolevel (Diskussion) 11:20, 20. Mär. 2012 (CET)
Da ist natürlich was dran. Ich hatte Ihn automatisch eher jünger eingeschätzt --81.200.198.20 11:22, 20. Mär. 2012 (CET)
Jünger als 15? Echt? Also ich nicht. --Snevern 12:08, 20. Mär. 2012 (CET)
Ernstgemeinte Zusatzfrage (an Hundeexperten):
Der Hund wird also nicht "scharf gemacht"; es handelt sich um einen ganz normalen Hund, sagen wir mal stadttauglich erzogen, gerne groß und kräftig. Nun geht Herrchen oder Frauchen (oder auch die ganze Familie) mit dem Hund spazieren; der Hund befindet sich also nicht im eigenen Revier (Wohnung, Haus, Garten), sondern im öffentlichen Stadtpark. Nun kommt tatsächlich ein Angreifer und greift (wie und zu welchem Zweck auch immer) einen der den Hund begleitenden Menschen tätlich an. Wie reagiert der Hund tatsächlich (wie es im Roman ausgeht, weiß ich selber)? Verteidigt er in irgendeiner Weise seinen Menschen? Greift er den Angreifer an? Kirschschorle (Diskussion) 01:10, 22. Mär. 2012 (CET)
Das ist nicht allgemein vorhersehbar: Hund ist nicht gleich Hund. Zum einen gibt es erhebliche rassebedingte Unterschiede, zum anderen gibt es erhebliche individuelle Unterschiede. Da ist von einer selbsttätigen Attacke auf den Angreifer über Schwanzwedeln und Abschlabbern fast alles möglich. Zeige mir vorher den Hund, und ich sage dir, wie er (wahrscheinlich) reagieren wird. Ansonsten ist es in etwa so, als würdest du mich danach fragen, wie ein den Menschen begleitender menschlicher Freund reagieren würde.
Wie meine eigenen Hunde in einer derartigen Situation reagieren, kann ich dir mit so großer Sicherheit vorhersagen, dass ich einen Geldbetrag darauf wetten würde. --Snevern 07:31, 22. Mär. 2012 (CET)
Ich habe gerade keinen Geldbetrag da - sagst Du es mir trotzdem? Kirschschorle (Diskussion) 13:04, 23. Mär. 2012 (CET)
Ich hatte in den letzten rund fünfzehn Jahren vier Hündinnen derselben Jagdhundrasse. Die erste, zweite und vierte stellen und verbellen selbsttätig, wenn sie etwas für verdächtig halten (das ist nicht immer das, was ich für verdächtig halte). Obwohl nicht abgerichtet, sind sie ausreichend wachsam und ausreichend groß, dass ich mich mit ihnen jederzeit vollkommen sicher fühle - meine Frau übrigens auch. Sie haben noch nie jemanden gebissen, aber ich bin mir zu einhundert Prozent sicher, dass sie es tun würden, wenn es drauf ankäme. Die dritte ist ein lammfrommes Wesen ohne jede Aggressivität, die sich nicht einmal ihrer eigenen Tochter gegenüber durchsetzt. Als Wach- oder Schutzhund wäre sie sicher nicht geeignet; im Falle eines Angriffs würde wohl eher ich sie als sie mich beschützen. Da wir allerdings die meiste Zeit zwei oder drei gleichzeitig hatten (haben), verhalten sie sich natürlich anders, wenn sie zusammen sind - und da erfüllt dann auch unser lammfrommes Schaf seine Schutzfunktion. --Snevern 15:18, 23. Mär. 2012 (CET)
Danke, war eine interessante Auskunft (hoffentlich auch für den fragenden Scharfmacherkandidaten). Kirschschorle (Diskussion) 20:38, 24. Mär. 2012 (CET)

Julian Assange: Kandidatur/ Verhaftung

Vor kurzem las ich, dass Julian Assange als Kandidat für den australischen Senat 2013 kandidieren möchte. Momentan befindet er sich meines Wissens nach noch in britischem Hausarrest, die Entscheidung des Gerichts steht aus. Gesetzt den Fall, Assange darf nach Schweden ausgeliefert werden und wird dort verhaftet, kann er dann in absentia für den Senat kandidieren? Und falls er dann gewählt würde, welche Auswirkungen hätte das auf seine Haft? Schließlich bekäme die Geschichte m.E. dann eine diplomatisch recht brisante Komponente. Weiß jemand, ob es schon vergleichbare Fälle gab? --92.231.94.144 20:51, 22. Mär. 2012 (CET)

Mir fällt Bobby Sands ein, der als hungerstreikendes und inhaftiertes Mitglied der IRA für das Unterhaus kandidiert hat und gewählt worden ist. Er starb vier Wochen nach seiner Wahl an den Folgen des Hungerstreiks. Vergleichbare Fälle für das Vereinigte Königreich hat das Unterhaus im gleichen Jahr durch den Representation of the People Act für die Zukunft unmöglich gemacht, indem darin Personen, die eine Haftstrafe von einem Jahr oder länger verbüßen müssen, das passive Wahlrecht entzogen wird. Eine vergleichbare Norm gibt es in Deutschland im § 45 StGB. Wie es in anderen Staaten aussieht, weiß ich nicht. Haftstrafen unter einem Jahr hindern jedenfalls weder in Deutschland noch im Vereinigten Königreich daran, sich als Kandidaten aufstellen zu lassen und gewählt zu werden. --88.73.159.93 23:06, 22. Mär. 2012 (CET)

Seitdem hat sich aber die Rechtslage geändert. In dem berühmten Präzedenzfall en:Hirst v United Kingdom (No 2) hat der Europäische Gerichtshof für Menschenrechte in Straßburg entschieden, dass es der Europäischen Menschenrechtskonvention widerspricht, Häftlinge mit einer rechtskräftigen Verurteilung zu einer Haftstrafe über einem Jahr automatisch vom Wahlrecht auszuschließen. Eine ähnliche Klage ein paar Jahre später gegen die Republik Österreich wurde genau gleich entschieden (siehe hier und hier). Ob damit nur das aktive Wahlrecht (also das Recht seine Stimme abzugeben) gemeint war oder auch das passive (also gewählt zu werden) ist juristisch etwas kompliziert, da in vielen Ländern das aktive und passive Wahlrecht in der Verfassung aneinander gekoppelt ist. Im Falle von Herrn Assange ist das ganze noch komplizierter, da er erstens noch nicht rechtskräftig verurteilt ist. Zweitens ist er Bürger eines EU-Drittstaates, ist aber in einem EU-Mitgliedsstaat inhaftiert, bzw. dort auf Kaution freigelassen worden, jedoch mit einem weiterhin anhängigen Verfahren. Für Australien hat die Judikatur des EGMR keine Relevanz und die Frage, ob eine eventuelle Wahl zum Senator in Australien eine juristische Relevanz für seinen Status im Vereinigten Königreich hat, dürfte ein reines inner-Commonwealth-Problem sein, das in letzter Instanz nur vom Privy Council entschieden werden kann. --El bes (Diskussion) 02:59, 23. Mär. 2012 (CET)
Besten Dank für die umfassende Antwort! (nicht signierter Beitrag von 92.231.93.43 (Diskussion) 22:02, 24. Mär. 2012 (CET))

Anna und die Liebe

Hallo Warum macht ihr Sendungen wie Anna und die Liebe, hört dann einfach auf, finde es einfach unmöglich man ist doch neugierig wie es weiter geht bitte weiter machen sonst seid ihr noch wenig interessant sorry --87.168.231.198 10:08, 23. Mär. 2012 (CET)

Wir machen keine Sendungen, wir schreiben eine Online-Enzyklopädie. --Eike (Diskussion) 10:10, 23. Mär. 2012 (CET)
Wir haben weder damit angefangen, noch damit aufgehört. Bin mir keiner Schuld bewusst. Uninteressant ist vor allem die erwähnte Seifenoper, nicht jedoch die Enzyklopädie. -- Lord van Tasm «₪» 10:11, 23. Mär. 2012 (CET)
Eventuelle Beschwerden gegen die Absetzung der Serie solltest du an Sat1 richten. -- Lord van Tasm «₪» 10:22, 23. Mär. 2012 (CET)
In unserem Artikel steht: "Im Januar 2012 wurde die Absetzung der Serie bekanntgegeben. Seit dem 30. Januar 2012 werden die restlichen Folgen der Telenovela auf dem Frauensender sixx ausgestrahlt und im Internet gezeigt. Die Ausstrahlung bei ORF eins bleibt unverändert.". Hier die Bekanntgabe von Sat1, und hier die Online-Folgen. -- Wiprecht (Diskussion) 10:53, 23. Mär. 2012 (CET)
sixx ist ein Frauensender? OMG!! --stfn (Diskussion) 12:37, 23. Mär. 2012 (CET)
Das Pendant zu DMAX sozusagen -- Lord van Tasm «₪» 13:21, 23. Mär. 2012 (CET)
Soso, die mit dem XX sind also die für die Frauen und die mit dem dicken MAX für die Männer. Interessant! :-) --Duschgeldrache2 (Diskussion) 04:08, 25. Mär. 2012 (CEST)

Vergleichende Linguistik

Welches Wort für dasselbe Objekt wird in den meisten Sprachen gleich oder sehr änlich ausgesprochen? --93.133.196.176 14:44, 23. Mär. 2012 (CET)

Coca-Cola 79.224.237.166 14:48, 23. Mär. 2012 (CET)
[[af:Wodka]] [[ar:فودكا]] [[arz:فودكا]] [[az:Araq]] [[bat-smg:Arielka]] [[be:Гарэлка]] [[be-x-old:Гарэлка]] [[bg:Водка]] [[bn:ভদকা]] [[bo:ཧྥུ་ཐ་ཅ་ཨ་རག]] [[bs:Vodka]] [[ca:Vodka]] [[cs:Vodka]] [[cv:Водка]] [[cy:Fodca]] [[da:Vodka]] [[el:Βότκα]] [[en:Vodka]] [[eo:Vodko]] [[es:Vodka]] [[et:Viin (jook)]] [[eu:Vodka]] [[fa:ودکا]] [[fi:Votka]] [[fo:Vodka]] [[fr:Vodka]] [[ga:Vodca]] [[gl:Vodka]] [[gu:વોડકા]] [[gv:Vodka]] [[he:וודקה]] [[hi:वोदका]] [[hr:Votka]] [[hsb:Wódka]] [[hu:Vodka]] [[id:Vodka]] [[is:Vodka]] [[it:Vodka]] [[ja:ウォッカ]] [[ka:არაყი]] [[ko:보드카]] [[ksh:Wodka]] [[la:Vodca]] [[lb:Wodka]] [[lbe:Аьракьи]] [[lt:Degtinė]] [[lv:Degvīns]] [[mk:Вотка]] [[ms:Vodka]] [[my:ဗော့ဒ်ကာ]] [[nah:Vodka]] [[nl:Wodka]] [[no:Vodka]] [[pl:Wódka]] [[pt:Vodca]] [[ro:Votcă]] [[ru:Водка]] [[sh:Votka]] [[si:වොඩ්කා]] [[simple:Vodka]] [[sk:Vodka]] [[sl:Vodka]] [[sr:Вотка]] [[su:Vodka]] [[sv:Vodka]] [[ta:ஓட்கா]] [[th:วอดก้า]] [[tr:Votka]] [[uk:Водка]] [[vi:Vodka]] [[xal:Əрк]] [[yi:וואדקע]] [[zh:伏特加]] Prosit! GEEZERnil nisi bene 14:53, 23. Mär. 2012 (CET)
[[az:Araq]] [[bat-smg:Arielka]] [[be:Гарэлка]] [[be-x-old:Гарэлка]] [[et:Viin (jook)]] [[lbe:Аьракьи]] [[lt:Degtinė]] [[lv:Degvīns]] [[xal:Əрк]] werden wohl ein bisschen anders ausgesprochen. --Rôtkæppchen68 15:41, 23. Mär. 2012 (CET)
" in den meisten Sprachen" Q.E.D. GEEZERnil nisi bene 16:17, 23. Mär. 2012 (CET)
Quark (Physik) und zigtausende anderer "moderner" Fachbegriffe, die in praktisch allen Sprachen so nah wie möglich am Ursprungswort und dessen Aussprache gehalten (fast immer nur transliteriert) werden -- sogar im Isländischen, wo man ansonsten aus Sprachpuristik-Gründen gern ganz neue Wörter erfindet. Und dann natürlich die ganzen Eigennamen (Coca-Cola, Linux usw.), sofern die nicht außer Konkurrenz laufen sollen. --Neitram 15:43, 23. Mär. 2012 (CET)
Mama? --Wrongfilter ... 16:20, 23. Mär. 2012 (CET)
Big Mac? --Besserimmeralsnie (Diskussion) 18:05, 23. Mär. 2012 (CET)
Nö.
Denn die Frage war ja nach demselben Objekt.
Ein (auch nicht überall gleich, aber doch ähnlich ausgesprochener) Big Mac ist manchenorts pappiges Brötchen mit Rind und mit Schwein drinnen, manchenorts mit Rind und mit ohne Schwein, manchenorts mit ohne Rind und ohne Schwein, dafür mit totem Huhn - und so weiter, schon aus religiösen Gründen. Kirschschorle (Diskussion) 18:12, 23. Mär. 2012 (CET)
Ein Big Mac ist IMMER mit Rinderhack! In Indien z.B. kommt das nicht so gut, da gibt es den Burger mit Geflügelfleisch, er heißt dort aber Maharaja Mac. --89.204.153.144 18:48, 23. Mär. 2012 (CET)
Das Wort Roboter ist auch in fast allen Sprachen sehr ähnlich. Es wurde vor 90 Jahren von einem polnischen Schriftsteller (Josef Čapek) erfunden. Dass Quark (Elementarteilchen) und Markennamen sich in verschiedenen Sprachen nicht unterscheiden, liegt einfach daran, dass dass die Begriffe gar nicht verändert wurden. --Cubefox (Diskussion) 21:35, 23. Mär. 2012 (CET)
Und Automat hätte ich noch. --Cubefox (Diskussion) 21:39, 23. Mär. 2012 (CET)
Ich würde mal sagen, das A für den gleichnamigen Buchstaben wird in so ziemlich allen Sprachen der Welt gleich gesprochen. -- Liliana 18:52, 23. Mär. 2012 (CET)
Das haut schon beim Vergleich deutsch - englisch - französisch nicht hin. Abgesehen von Sprachen ohne lateinisches Alphabet. --89.204.153.144 18:55, 23. Mär. 2012 (CET)
Lassen wir mal die europäischen Sprachen weg, das sind ja nur 100 von so ungefähr 7.000 Sprachen auf der Welt. Und freilich wird man auch in anderen Teilen der Welt, sofern sie nicht zu abgelegen sind, schon mal ein A gesehen haben (die Chance ist freilich höher als bei Coca-Cola oder Wodka!). Von den oben genannten Beispielen funktioniert keins wirklich überall, selbst "Mama" nicht (das Mohawk hat kein M). -- Liliana 18:58, 23. Mär. 2012 (CET)
Das haut schon nicht hin, bevor man auch nur irgendwohin rausgeht aus der deutschen Sprache: A ist ein langes "a" wie in Wahn und A ist ein kurzes "a" wie in Wannsee. Kirschschorle (Diskussion) 20:19, 23. Mär. 2012 (CET)
Wikipedia :-) Mauerquadrant (Diskussion) 22:32, 23. Mär. 2012 (CET)
Wodka ist gut - Kaffe erscheint noch besser (Tee ist schwach...):
[[af:Koffie]] [[am:ቡና]] [[an:Café]] [[ang:Caffiȝ]] [[ar:قهوة]] [[arz:قهوه]] [[ast:Café]] [[az:Qəhvə]] [[bar:Kafää]] [[bat-smg:Kava]] [[be:Кава]] [[be-x-old:Кава]] [[bg:Кафе]] [[bjn:Kupi]] [[bn:কফি]] [[bo:ཁོ་ཧྥེ།]] [[br:Kafe]] [[bs:Kahva]] [[ca:Cafè]] [[chr:ᎧᏫ]] [[cs:Káva]] [[csb:Kawa]] [[cv:Кофе]] [[cy:Coffi]] [[da:Kaffe]] [[el:Καφές]] [[en:Coffee]] [[eo:Kafo]] [[es:Café]] [[et:Kohv]] [[eu:Kafe]] [[fa:قهوه]] [[fi:Kahvi]] [[fiu-vro:Kohv]] [[fr:Café]] [[fur:Cafè]] [[ga:Caife]] [[gan:咖啡]] [[gd:Cofaidh]] [[gl:Café]] [[hak:Kâ-pî]] [[he:קפה]] [[hi:कॉफ़ी]] [[hif:Coffee]] [[hr:Kava]] [[ht:Kafe]] [[hu:Kávé]] [[hy:Սուրճ]] [[ia:Caffe]] [[id:Kopi]] [[io:Kafeo]] [[is:Kaffi]] [[it:Caffè]] [[ja:コーヒー]] [[jbo:ckafi]] [[jv:Kopi]] [[ka:ყავა]] [[kk:Кофе]] [[kn:ಕಾಫಿ]] [[ko:커피]] [[krc:Кофе]] [[ku:Qehwe]] [[la:Coffeum]] [[lad:Kavé]] [[lb:Kaffi]] [[lij:Caffè]] [[lo:ກາເຟ]] [[lt:Kava]] [[lv:Kafija]] [[mk:Кафе]] [[ml:കാപ്പി (പാനീയം)]] [[mn:Кофе]] [[mr:कॉफी]] [[ms:Kopi]] [[nds-nl:Koffie]] [[nl:Koffie]] [[nn:Kaffi]] [[no:Kaffe]] [[nv:Ahwééh]] [[oc:Cafè]] [[om:Buna]] [[os:Къофи]] [[pap:Koffie]] [[pl:Kawa]] [[pnb:کافی]] [[ps:قهوه]] [[pt:Café]] [[qu:Kaphiy]] [[ro:Cafea]] [[ru:Кофе]] [[rue:Кава]] [[rw:Ikawa]] [[sa:काफीपेयम्]] [[sah:Кофе]] [[scn:Café]] [[sco:Coffee]] [[se:Gáffe]] [[sh:Kava]] [[simple:Coffee]] [[sk:Káva]] [[sl:Kava]] [[so:Bun]] [[sq:Kafeja]] [[sr:Кафа]] [[su:Kopi]] [[sv:Kaffe]] [[sw:Kahawa]] [[szl:Kafyj]] [[ta:காப்பி]] [[te:కాఫీ]] [[th:กาแฟ]] [[tl:Kape]] [[tpi:Kopi]] [[tr:Kahve]] [[tt:Каһвә]] [[uk:Кава]] [[ur:کافی]] [[uz:Qahva]] [[vec:Cafè]] [[vi:Cà phê]] [[vls:Kaffie]] [[war:Kape]] [[wuu:咖啡]] [[yi:קאווע]] [[zh:咖啡]] [[zh-min-nan:Ka-pi]] [[zh-yue:咖啡]] GEEZERnil nisi bene 09:34, 24. Mär. 2012 (CET)
Das ICAO-Alphabet ist im Wesentlichen ein Versuch, moeglichst fuer jeden Buchstaben ein "internationales" Wort zu identifizieren. Taxi ist das klassische Beispiel, das geht bis ins Japanische, aber beim Chinesischen ("Chuzuche") ist der Kaffee im Vorteil. -- Arcimboldo (Diskussion) 03:56, 25. Mär. 2012 (CEST)

timescale of life

Hallo, ich suche eine Tabelle wo die Entwicklung der Lebewesen gut nachvollziehbar ist. So ähnlich wie eine geologische Zeittabelle. Nur eine die die Zeitspannen richtig zeigt. Also beispielsweise 20 Bildschirmmeter lang ist weil es ja auch viel zeit war. Gibt es so etwas bei Wikiepedia?

und beschriftet mit den Tieren und Pflanzen die es damals gab


Ich suche das, damit ich es mir besser vorstellen kann wie die Evolution abgelaufen ist.(nicht signierter Beitrag von 46.115.0.167 (Diskussion) 20:39, 23. Mär. 2012 (CET))

Nicht ganz was du suchst, aber die englische Wikipedia hat die Artikel en:Timeline of evolutionary history of life und en:Evolutionary history of life. Leider hat einer der beiden ein deutsches Äquivalent... --Wrongfilter ... 20:44, 23. Mär. 2012 (CET)
+1 (BK) Google => timescale of evolution <= liefert aber nur Kasperle-Effekte. Wie soll man das nebeneinander (und Aussterben) von mehreren Millionen Spezies zeigen? Beschränkt man sich auf Einzeller, oder Pflanzen oder Wirbeltiere oder Pize oder den Menschen ... kriegt man auch nur Kasperle-Effekte. ;-) GEEZERnil nisi bene 20:48, 23. Mär. 2012 (CET)
 
Leider nur englisch und ohne Stammbaum-artige Darstellung. --Cubefox (Diskussion) 22:04, 23. Mär. 2012 (CET)
Ich denke die IP will etwas, wo die Zeit im richtigen Verhältnis dargestellt wird. Bei der "Zeitspirale" rechts (von Wrongfilter genanntes Bild in höherer Auflösung) ist die Zeit wohl richtig umgesetzt, leider sind aber die evolutionären Zusammenhänge darauf nicht abgebildet. --Cubefox (Diskussion) 22:04, 23. Mär. 2012 (CET)
Und in bewegten Bildern (BBC, fängt etwa bei -530 Mill. Jahren an? Mehrere Teile ... (ab Min. 2 wird evolviert). GEEZERnil nisi bene 22:43, 23. Mär. 2012 (CET)
Nein, aber eine Seite vielleicht wo dass jedes Jahr eine Zeile hat in der drin steht was da war. weiter unten kanns dann natürlich auch zusammengefasst sein. 46.115.19.106 08:44, 24. Mär. 2012 (CET)
Jedes Jahr? So wie: Jahr -152.127.204: Stegosaurus ? GEEZERnil nisi bene 09:27, 24. Mär. 2012 (CET)
Wenn es so eine Seite gibt würde ich sie gern sehen. Ist klar dass da dann nichts drin steht für das Jahr 152.127.204 v.Chr.. Aber es gab dieses Jahr früher mal. 46.115.19.106 11:08, 24. Mär. 2012 (CET)
Nö, das gab es erst im Nachhinein. Weil damals waren ja die Punkte zwischen den Zahlenblöcken noch nicht erfunden! Kirschschorle (Diskussion) 13:07, 24. Mär. 2012 (CET)
Aber es gab dieses Jahr früher mal - bist Du sicher? Vielleicht lag das ja auch in einem Schaltmillennium.
Weil damals waren ja die Punkte zwischen den Zahlenblöcken noch nicht erfunden - ebensowenig wie die Blöcke selbst, die man damit voneinander getrennt hat - na und? Wir schreiben ja auch deutsche Artikel über Epochen, in denen es diese Sprache noch nicht gegeben hat. Chiron McAnndra (Diskussion) 15:28, 24. Mär. 2012 (CET)
Wie lang wäre so eine Seite wo alle Jahre drin sind, die es gibt? 46.115.19.106 15:46, 24. Mär. 2012 (CET)
Eine lineare zeitliche Darstellung wäre witzlos und stinkelangweilig. Fange mal vor 3,8 Milliarden Jahren mit den ersten Prokaryoten an. Wenn Du jetzt pro Jahr eine Zeile Text schreibst und ca. 60 Zeilen auf eine Seite bekommst, dann wäre Dein Text über 60 Millionen Seiten stark und auf den meisten Seiten würde nichts außer Jahreszahlen stehen. Nimm einen logarithmischen Zeitmaßstab und die Sache wird wesentlich interessanter und kürzer. Beschränke Dich auch auf die wichtigsten Arten, denn mit 1,5 Millionen bekannten Arten wird Dein Diagramm sonst sehr unübersichtlich. --Rôtkæppchen68 16:33, 24. Mär. 2012 (CET)
Ich glaube, eine logarithmische Skale überfordert unsere Intuition. Die lineare Skala wär schon besser, dann wohl mit der Message: nicht viel los gewesen die ersten paar Milliarden Jahre! --Eike (Diskussion) 16:59, 24. Mär. 2012 (CET)
Die ganze Evolution in 5 Min MIT Jahreszahl-Counter. Gut Ding will Weile haben... GEEZERnil nisi bene 23:36, 24. Mär. 2012 (CET)

Fliegende Fische im Roten Meer

 

Gibt es die da? Irgendwas hatte ich beim Windsurfen vor Safaga im Augenwinkel gesehen. Danke --89.204.154.246 20:32, 24. Mär. 2012 (CET)

Fliegende Fische kommen wohl unter anderem im Indopazifischen-Raum vor, laut der dort eingebundenen Karte zählt dazu auch das Rote Meer
-- Viele Grüße von Jogo.obb Disk 20:56, 24. Mär. 2012 (CET)

Webseite gesucht

Ich suche eine Webseite: In der Mitte war ein Suchfeld wie bei Google, rundherum eine art Schnellwahl. Aber es waren nicht, wie es bbei Opera ist, die Webseiten dargestellt, sondern höchstens kleine "Bildlinks". Und es war eine Webseite, kein Feature oder kein neuer Browser. Ging im Mozilla und im Internet Explorer.

--91.40.120.180 21:10, 24. Mär. 2012 (CET)

du suchst dashier? --Duckundwech (Diskussion) 12:35, 25. Mär. 2012 (CEST)

Anti-Atomkraft-Blume

In den Anfangsjahren der Anti-Atom-Bewegung existierte neben der heute verwendete Sonne auch ein Aufkleber, der eine Hand mit einer Blume zeigte. Weiß jemand, wo ich das Bild finden kann? Danke. --Tripscale (Diskussion) 17:49, 22. Mär. 2012 (CET)

Du meinst aber nicht die Faust mit der Rose? --= (Diskussion) 18:35, 22. Mär. 2012 (CET)
Den blauen Button hab ich, trägt die Aufschrift "stoppt die atomindustrie - kämpft für das leben". Wenn Dir ein Foto genügt, mache ich es morgen. Liebe Grüße, --Haeferl (Diskussion) 18:40, 22. Mär. 2012 (CET)
Mit der Information findet man's dann auch im Netz: [20] --Eike (Diskussion) 18:44, 22. Mär. 2012 (CET)
...als Direktlink dann wohl das da. -- Felix König 18:49, 22. Mär. 2012 (CET)
Weil dieses Symbol ursprünglich aus dem gemeinsamen deutsch-französischen Kampf gegen das AKW Wyhl und das AKW Fessenheim seit Anfang/Mitte der 1970er Jahre stammt und in der BRD von den französischen Atomkraftgegnern übernommen wurde wird man auch fündig mit der Suche nach “halte à l'industrie nucléaire – combat pour la vie.” --84.191.143.217 21:01, 22. Mär. 2012 (CET) Ergänzung (Fessenheim - zur Klarheit) --84.191.143.217 21:23, 22. Mär. 2012 (CET)
Symbolismus ... die "Sonne" ist ein Fusionsreaktor und die Schönheit von "Blumen" hängt von der Funktion dieses Reaktors ab - aber ich vermute, dass ich irgendwie falsch liege... GEEZERnil nisi bene 21:06, 22. Mär. 2012 (CET)
hihi, ja, da liegst du falsch. Nicht mit Symbolismus, aber mit dem Fusionsreaktor. Sooo tief hat damals noch kaum jemand gedacht. So weit ich weiß, steht/stand die Blume eher für den Schutz der Natur - und die geschützte Natur für das (nur so für Menschen in dieser Natur mögliche) Leben. Hab aber keine valide Quelle zur Hand. --84.191.143.217 21:23, 22. Mär. 2012 (CET) p.s.: Die lachende Sonne des ursprünglichen „Atomkraft? Nej tak“ bezieht sich ganz klar auf die Alternativenergien, mitgedacht war da z.B. aber auch schon das später legendäre Windkraftwerk der Freien Schule Tvind (vgl. z.B. hier), das ebenfalls schon ab 1975 gebaut wurde.--84.191.143.217 21:30, 22. Mär. 2012 (CET) Addendum: Bilder zur "Tvindkraft" --84.191.143.217 21:42, 22. Mär. 2012 (CET)
Aufgrund der extremen Dichte im Zentrum und der damit verbundenen Neutrino-Kühlung zündet diese Fusionsreaktion zunächst innerhalb einer heißeren kugelschalenförmigen Zone um das Zentrum. Wenn er weit genug weg ist, ist er lieb, oder? GEEZERnil nisi bene 23:27, 22. Mär. 2012 (CET)
Das ist ein Grundmuster. Je weiter etwas weg ist, desto lieber erscheint es uns. Hunger, Krieg, Tsunami, Bundespräsident, Schwiegermutter, ausgeflippter Fusionsreaktor, was auch immer. Der Scheinriese ist eine bestätigende Ausnahme von der Regel. Und Kraus fügt noch hinzu: „Je näher man ein Wort ansieht, desto ferner sieht es zurück". Diese kluge Sprache – sie hält uns auf Distanz, damit sie uns noch lieben kann, wenn wir sie selbst schon nicht mehr lieben und ihr zu dicht und zu aufdringlich auf den Pelz rücken. :) --84.191.143.217 02:56, 23. Mär. 2012 (CET)
Noch ein schöner Nachtrag zum Thema Symbolsprache (ein Versuch, ich ahne, daß Symbolismus - welcher auch immer - wohl etwas anderes ist): Es ist der Hut... Interessanterweise nach dieser Quelle von einer deutschen Agentur: Serviceplan Hamburg / München, Germany mit Axel Thomsen und Alexander Schill (Creative Directors) und Jonathan Schupp (Art Director); Copywriter: Francisca Maass. --84.191.136.161 16:28, 24. Mär. 2012 (CET)
Jaaaa, die Sonne ist halt für sehr lange Zeit inhärent betriebssicher, da die Kühlleistung ausschließlich durch das fast konstante Verhältnis von Oberfläche zu enthaltener Masse bestimmt wird. Überhaupt ist die Konstruktion denkbar einfach. Wasserstoff, Gravitiation, Abfallprodukte werden hauptsächlich im Kern eingelagert, unschädlich ist das meiste davon auch noch - und vor allem alles völlig kostenlos !. -- Janka (Diskussion) 21:59, 22. Mär. 2012 (CET)
Nein, Janka, kostenlos stimmt m.E. einfach nicht. Das ist eine Angriffsfläche, die man da garnicht bieten muß, denn in der Kostenfrage steckt ein gutes Argument drin, das man nicht freiwillig aus der Hand geben muß. Sogenannte alternative Energien (den Begriff "erneuerbare Energien" halte ich unter dem Aspekt des Energieerhaltungssatzes für eher dümmlich), also auch die Energiegewinnung über die Sonne kosten auch Geld (oder Arbeitskraft, Rohstoffe etc.). Nur ist das Kosten-Nutzen-Verhältnis eindeutig und insgesamt wesentlich günstiger. Vor allem, wenn man die Kosten mitberechnet, die von der Atommafia sozialisiert, also der Gesellschaft aufgehalst wurden und werden - von den Forschungskosten angefangen über die Steuergeschenke bis zu den heute noch radioaktiv verseuchten Wildschweinen in Bayern und der ungelösten sogenannten "Endlagerung". --84.191.143.217 22:13, 22. Mär. 2012 (CET)
Pardon, 84.191.143.217, aber wenn man die Erde als in sich abgeschlossenes System betrachtet (und im Hinblick auf Energieverbrauch und Energiegewinnung ist das durchaus sinnvoll), dann stellt die Sonnenenergie eine Zufuhr von Energie von außen dar; der Energieerhaltungssatz gilt insofern nicht. Mit der Verwendung des Wortes "dümmlich" wäre ich da zurückhaltend.
Ansonsten bin ich aber bekanntermaßen weitgehend deiner Meinung. --Snevern 22:27, 22. Mär. 2012 (CET)
Die Erde ist aber kein geschlossenes System, sondern gibt genausoviel Energie an den Weltraum ab wie sie von der Sonne erhält. -- Wiprecht (Diskussion) 23:32, 22. Mär. 2012 (CET)
Doch, das ist im Wesentlichen schon ein geschlossenes System (nur kein abgeschlossenes), denn Massenaustausch findet kaum statt (am ehesten noch durch Meteoriten, die kosmische "Strahlung" ist erst recht vernachlässigbar), nur Energieaustausch. Und dabei strahlt die Erde etwas weniger ab, als sie empfängt. (Es gibt im gegenwärtigen Sonnensystem natürlich kein Strahlungsgleichgewicht durch Temperaturangleichung, zumal es ja noch viel weniger als die Erde ein abgeschlossenes System ist, sondern der Löwenanteil der Energie einfach in die übrige Milchstraße verschwindet.) Laut unserem Artikel werden z.B. für die Photosynthese (mithin in geologischen Zeiträumen für die Bildung fossiler Energieträger wie Kohle oder Erdöl) gut 0,1 Promille der eingestrahlten Sonnenenergie abgezweigt. Und der gegenwärtige Weltenergiebedarf (der natürlich großteils durch schon vor langer Zeit eingestrahlte Sonnenenergie, die sich in chemische Energie verwandelt hatte, gedeckt wird) liegt in einer ähnlichen Größenordnung. Außerdem wäre selbst dann, wenn diese kleine Differenz gegenwärtig exakt 0 wäre, natürlich die Sonne ein großer potenzieller Energielieferant. Denn sobald durch menschliche Aktivität (z.B. Umwandlung in elektrische oder chemische Energie durch Sonnenkollektoren) die Strahlungsenergie in Energieformen umgewandelt wird, die nicht einfach wieder in den Weltraum zurückreflektiert werden, sinkt die Energieabstrahlung der Erde, ohne dass die Einstrahlung zurückgeht.
D.h. das System Erde lässt die "Quelle" Sonnenenergie bisher weitgehend durch sich hindurchfließen, ohne viel zu entnehmen, aber es ist kein Naturgesetz, dass das in Zukunft so bleiben muss. --Grip99 02:38, 24. Mär. 2012 (CET)
Zahlt ihr der Sonne was dafür, dass sie euch den Arsch auf 300K wärmt? Ich nicht. Zahlt ihr der Sonne was dafür, dass sie für Pflanzenwachstum und damit für was zu Essen sorgt? Ich nicht. Zahlt ihr der Sonne was dafür, dass sie das Wetter antreibt und damit extrem viele ökologische Nischen schafft, in denen sich sowas wie intelligentes Leben überhaupt entwickeln konnte? Ich nicht. Dagegen ist der zivilisatorische Energiebedarf, den wir meinen zu brauchen geradezu bescheiden und dann auch noch weitenteils verzichtbar. -- Janka (Diskussion) 01:13, 23. Mär. 2012 (CET)
Er hat A*** gesagt! Bezahlen? Wer wettet mit mir, ob Atomkraftwerke oder die Sonne mehr Krebs auslösen, Leute verdursten und verhungern lassen? ;-) Der Unterschied ist, dass wir bei der Sonne KEINE ALTERNATIVE haben und uns deshalb daran GEWÖHNT haben. Hehehe! GEEZERnil nisi bene 07:59, 23. Mär. 2012 (CET)
@Janka: Yepp, so verstehe ich das, worum es dir geht, besser (wobei ich persönlich jedoch den zivilisatorische Energiebedarf für unseren "Sieg über die Nacht" nicht missen möchte und diesen Sieg in seiner kulturhistorischen Tragweite auch für unterschätzt und teils unverstanden halte).
Lieber Snevern, der Fehler liegt sicher bei mir, weil ich mich über den Begriff Erneuerbare Energie nur in einem Nebensatz mokiert habe, ohne konkret zu argumentieren. Der Begriff bezieht sich ja im allgemeinen Gebrauch nicht (wie du in deiner vorstehenden kritischen Anmerkung) nur auf Sonnenenergie sondern versucht mit "erneuerbar" auf bestimmte Eigenschaften abzuheben, um diese Formen der Energienutzung von anderen, derzeit vorherrschenden Formen abzugrenzen. Das zentrale Kriterium ist dabei die Erschöpflichkeit der Ressource und der Zeitraum, der benötigt wird, um eine verbrauchte Energieressource wieder "aufzufüllen" oder auszugleichen. Würden Öl, Kohle, Gas und Uranvorkommen nicht absehbar für lange Zeit ausgeschöpft sein, käme wohl niemand auf die Idee, unter genau diesem Aspekt zu unterscheiden. Unser Artikel spricht (bei allen Vorbehalten hinsichtlich der wp-Definitionen) von "Energien aus Quellen, die sich entweder kurzfristig von selbst erneuern oder deren Nutzung nicht zur Erschöpfung der Quelle beiträgt. Es sind nachhaltig zur Verfügung stehende Energieressourcen [...] Eine weitere Quelle erneuerbarer Energien ist das energetische Potenzial [...] der aus nachwachsenden Rohstoffen gewonnenen Biomasse."
Statt um Energie geht es tatsächlich also nur um das Potential der Energieressource und seine Eigenschaften, wie schnell und in welcher Form Energie in dieses Potential umgewandelt wird. Weil sich Energie nicht erneuert sondern nur umwandelt wären ja streng genommen auch Öl oder Gas Energieressourcen, die sich in diesem Sinn "erneuern". Man muß ja nur lange genug warten. Das wiederum ist aber nicht im Sinne des allgemeinen Gebrauchs von "Erneuerbarer Energie". Es dauert zu lang. Mit "Erneuerbarer Energie" ist also eigentlich keine Energie gemeint sondern eine Energieressource, die sich langfristig nicht erschöpft, weil sie in relativ kurzer Zeit nachwächst oder sich nachbildet oder ohnehin mehr oder weniger dauerhaft oder zyklisch zur Verfügung steht. Der Zeitfaktor ist eine wichtige begriffsbestimmende Bedingung.
In einer Welt, in der die wenigsten Menschen detailliert erklären können, was passiert, wenn sie in einer Alltagshandlung einen Lichtschalter betätigen (und solange der Wert von Bildung und Allgemeinbildung derart geringgeschätzt wird, ist das diesen Menschen selbst nur bedingt vorzuwerfen) kann man nicht erwarten, daß der überwiegenden Mehrheit geläufig ist, daß sich Energie eben nicht erneuert sondern nur wandelt. Unter diesen Bedingungen einen Begriff zu setzen, der suggeriert, Energie erneuere sich, trägt zumindest bei mir zu dem begründeten Anfangsverdacht bei, daß es nicht darum ging, über unsere Energieprobleme aufzuklären sondern darum, ein gefälliges werbegriffiges Sedativum zu verabreichen: Macht euch keine Sorgen, es wird genug Energie da sein. Wenn das Öl alle ist fahren und fliegen wir halt mir Rapsöl. Ihr müßt euer Konsum- und Verbrauchsverhalten nicht in Frage stellen.
"Erneuerbare Energie", die weder dem Begriff Energie, noch den Begriffen Erneuerbarkeit und Wandlung gerecht wird, steht für mich damit in einer Reihe mit ähnlichen Euphemismen wie "Entsorgungspark", "Präventivschlag", "Freisetzung" oder "Zu Ihrer Sicherheit wird dieser Laden videoüberwacht".
Dies also als Argument für meine Kritik. "Dümmlich", weil diese Begriffsbildung die eigentlichen Sachverhalte viel zu durchsichtig entweder nicht versteht oder verstehen will oder gar bewußt verschleiert und irritierend umdeutet. "Verdummend" würde ich jetzt noch hinzufügen in dem Sinn, daß man (individuell und gesellschaftlich) dauerhaft nur gut lösen kann, was man auch gut verstanden hat.
Der Kritik des Kollegen Wiprecht kann ich folgen, und im Hinblick auf Energieverbrauch und Energiegewinnung erscheint es mir sinnvoller, die Zufuhr und Abgabe innerhalb des Sonnensystems in eine Systembetrachtung einzubeziehen. Nicht nur, weil wir ohne Sonne, wie der Kollege Janka richtig ausführt, ganz schön blöd im Dunkeln dastehen würden sondern auch, weil es ja die Sonne ist, die in etwa 500 Millionen Jahren das System dauerhaft zu unseren Ungunsten ausgestalten wird. (Licht aus für uns Menschen, aber nicht für das System... Aber vielleicht schaffen wir das aus eigener Kraft und mit Hilfe unserer schöpferischen Intelligenz auch schon vorher ganz alleine. ) --84.191.143.217 02:27, 23. Mär. 2012 (CET)
Wie gesagt: Ich teile im Wesentlichen deine Meinung. ;o) --Snevern 07:39, 23. Mär. 2012 (CET)
Gott bin ich froh, wenn endlich die Fusionskraftwerke erfunden und in Betrieb sind. Die gleichen Leute werden dann zwar genauso wegen der Fusionskraft rumjammern, aber wenigstens gibt es dann das erste mal nach 30 Jahren neue Aufkleber --FNORD (Diskussion) 12:16, 23. Mär. 2012 (CET)
Die Platte wird über die Jahrzehnte langweilig. --Eike (Diskussion) 12:54, 23. Mär. 2012 (CET)
Was oft vergessen wird ist, dass auch die gesellschaftliche Komponente einer Entwicklung entworfen ist, die der natürlichen bzw. technischen Entwicklung aber immer nur *nacheilt*. Und das auch nur lückenhaft: Hand auf's Herz: Wie viele von euch haben eine bewusste Erinnerung, wie es vor dem Smartphone war? Vor dem Internet? Vor dem Handy? Oder als *der Russe* immer vor der Tür stand? Wer von euch hat vor 1985 vor einem Computer gesessen? Wer erinnert sich noch an die Zeit vor der Eroberung des Weltraums? An die Errichtung des ersten kommerziellen Kernreaktors in Deutschland? Wer an die Zeit, als Automobile exotisch und Pferde allgegenwärtig waren?
Wie soll man jemandem, der nur die Gegenwart kennt, erklären, dass es all die Dinge, die in der Gegenwart selbstverständlich sind vor 10, 20, 30, 40, 50, 60 Jahren nicht gab? Also in einer Spanne, die weniger als ein Menschenleben umfasst? Wir sind inzwischen an einem Punkt angekommen, wo eine Generation bereits mehrere technische Sprünge verkraften muss, jeden als das ultimative Konzept für die Zukunft angepriesen und schon wenige Jahre später wieder invalidiert. Früher war es vielleicht eine wesentliche Erfindung pro Generation. Das ist hyperpostmodern.
Erinnert sich noch wer an StudiVZ? -- Janka (Diskussion) 12:19, 23. Mär. 2012 (CET)
@Janka - alles völlig kostenlos ! - sag das nicht so laut - sonst kommt am Ende jemand auf die Idee, Sonnenlicht zu besteuern. Chiron McAnndra (Diskussion) 14:05, 23. Mär. 2012 (CET)
Also, an die Zeit, in der es noch keine Automobile mehr gab, kann ich mich nicht mehr so gut erinnern, aber ich weiß, dass die jüngeren unter uns niemals das Glücksgefühl erleben werden, das einen überkam, wenn man eine Telefonzelle gefunden hatte, in der der Vorbenutzer weder uriniert noch Kette geraucht hatte, und dann das Gefühl maßloser Enttäuschung, wenn man entdeckte, dass ein übereifriger Elektrobastler den Telefonhörer mitgenommen hatte.--Optimum (Diskussion) 15:41, 23. Mär. 2012 (CET)
Das mit der einen wesentlichen Erfindung pro Generation halte ich für ein Gerücht :) Mein Urgroßeltern sind in einer Welt aufgewachsen in der es ein einzelnes Telefon im nächsten größeren Dorf gab und es ne tolle Sache war wenn jemand ein echtes Auto gesehen hatte und bis zu Ihrem Lebensende gabs Raketen, düsengetriebene Flugzeuge und Fernseher. Kommt uns nur so vor als wäre alles Relevante erst in den letzten 70 Jahren stattgefunden. In jedem stabilen Staatsgebilde das längere Zeit existiert, galopiert der Fortschritt. Vermutlich war das bereits bei den Römern so das die sich bis zu Ihrem Lebensende an lauter erstaunliche neue Sachen gewöhnen mussten, die so im Laufe der Jahrzehnte erfunden wurden und Alltag geworden sind. --85.181.219.233 20:20, 23. Mär. 2012 (CET)
Die Römer-Annahme ist reine Poesie und findet keinerlei Grundlage in Literatur und Funden.
Zutreffend ist allein Deine Annahme, die alten Römer hätten "galoppieren" nicht mit zwei "p" geschrieben. Kirschschorle (Diskussion) 20:31, 23. Mär. 2012 (CET)
Die Neuzeit ist nun schon 300 Jahre alt! Und deine Urgroßeltern gehören auch schon zur Postmoderne. Die haben in ihrem Leben schon mehrere kulturverändernde Erfindungen erlebt. Unsere Generation wird auf schon auf dutzende kommen. -- Janka (Diskussion) 22:09, 23. Mär. 2012 (CET)
Hm. Wie so oft sind wir ja garantiert nicht die ersten, die sich dazu nen Kopp machen. Nun habe ich mir mal den Artikel Technikgeschichte angesehen und bin von dort zu Chronologie der Technik gesprungen. Diese Liste endet erstaunlicherweise schon 1992. Beide Artikel stellen mich (nicht nur wegen der Lücke der letzten 20 Jahre) nicht sonderlich zufrieden und ein Blick in die Kategorie verwirrt und schreckt ab angesichts der großen Zahl der Einträge. Die chronologische Liste, die "wichtige Meilensteine der Entwicklung und Geschichte der Technik" ankündigt, verweist zu allem Überfluß auch noch auf 17 Artikel zur Geschichte ausgewählter Techniken - für mein Anliegen und Interesse eine ausgesprochen ärgerliche Streuung der Information statt Zusammenfassung. Gibt es denn, vergleichbar mit dem Ploetz oder eines der Kulturfahrpläne einen schönen Literaturhinweis? Also ein seriöses Buch, in dem sich auf etwa 30 bis 50 Seiten die relevanten Entwicklungen seit 1900 überblicken lassen? --84.191.143.92 01:14, 24. Mär. 2012 (CET)
Es ist wohl auch schwierig, heute schon zu sagen, welche Erfindungen der z.B. letzten 50 Jahre wirklich relevant für die Menschheitsgeschichte sind. Es gibt schon viele Menschen, die sagen, daß ein Fortschritt der Menschheit heute nur mehr ein Schritt zurück sein kann, wer weiß, vielleicht kündigt das eine langfristige Abkehr von der ganzen Technik an, vielleicht aber auch nicht. Vielleicht fährt in dreißig Jahren keiner mehr Auto - wie relevant war dann die Erfindung, betrachtet aus dem 25. Jahrhundert? Wer soll solche Dinge heute vorhersehen können und damit festlegen, welche der neueren Erfindungen wirklich so relevant sind, daß man die anderen schon einmal getrost unter den Tisch fallen lassen kann? - Aber zurück zum ursprünglichen Thema: Das Foto hab ich gemacht und hochgeladen. (Wie kann ich das intern verlinken, ohne daß es als Bild erscheint?) Liebe Grüße, --Haeferl (Diskussion) 05:48, 24. Mär. 2012 (CET)
Mit Doppelpunkt. --Eike (Diskussion) 09:43, 24. Mär. 2012 (CET)
Danke! :-) --Haeferl (Diskussion) 23:30, 24. Mär. 2012 (CET)
Was relevant für die Menschheitsgeschichte ist, merken immer erst die Menschen mehrere Generationen nach einer solchen Erfindung - denn erst eine Langzeitanalyse kann zeigen, wie einschneidend und umfassend Änderungen gewesen sind. Für uns - also für die Zeitgeschichte - sind viele Dinge relevant, die in ein paar Generationen wieder vergessen sind (da wird dann kaum jemand mehr begreifen, was es damit auf sich hat, wenn einer an beiden Händen zwei Finger abspreizt und dabei "Yo, Man!" sagt). Auch Erfindungen müssen keinen dauerhaften Einfluss auf uns haben - selbst dann nicht, wenn so ein Einfluss ein paar Jahrzehnte lang nicht abzustreiten ist. In der Militärgeschichte etwa gab es reihenweise "Erfindungen" - eigentlich Weiterentwicklungen bekannter Methoden - die in ihrer Zeit eine Verbesserung darstellten - allerdings nur, weil zuvor einige andere Methoden aufgegeben wurden - weshalb dann solche Reiche, die einst die effektivsten Streitkräfte befehligten, irgendwann nichts mehr zu melden hatten, obwohl ihre Gegner nicht so immens viel dazu gelernt hatten. Dazu kommt dann noch, was als Erfindung gelten kann. Ist eine Erfindung tatsächlich für sich allein stehend relevant, wenn sie doch in Wahrheit auf anderen Erfindungen basiert, ohne die es sie nie gegeben hätte? Was wäre denn die Erfindung eines Computers, ohne die Erfindung des Transistors? Und wie hätte man den erfinden können, ohne die Entwicklung der modernen Wissenschaft? Die Erfindung des Rades wird gern für solche Dinge beispielhaft herangezogen - doch wir wissen, dass die südamerikanischen Kulturen das Rad durchaus kannten - es wurde nur eben nicht angewendet. Eine Erfindung allein reicht also auch nicht aus, um sie zu etwas relevantem zu machen. Chiron McAnndra (Diskussion) 15:57, 24. Mär. 2012 (CET)

Das ist hier doch kein Chatforum. Die Frage ist beantwortet! --82.113.98.33 19:28, 25. Mär. 2012 (CEST)

Archivierung dieses Abschnittes wurde gewünscht von: Stimmt. --Eike (Diskussion) 10:52, 26. Mär. 2012 (CEST)

Anpaddeln

Moin,

es wird Zeit... aber vor dem Anpaddeln möchte ich meinem Boot (altes Lettmann-Zweier-Kajak, GFK) mal ein wenig Pflege, fast schon Instandsetzung, gönnen. Das gute Stück ist ein wenig sehr rissig geworden, zwar noch dicht, aber halt von sehr vielen Längsrissen durchzogen. Da ich niemanden fragen kann (kenne keine Paddler hier ;-), hab' ich mich ein wenig schlau gelesen und bin - natürlich - auf den Begriff Gelcoat gestoßen. Meine Frage: Ist dat Zeuchs für eine Gesamtbeschichtung des Rumpfes geeignet (ja, vermute ich), hält das gut und ist elastisch genug, neue Risse und die Vertiefung der alten zu vermeiden (ebenfalls ja, denke ich) und gibt es etwas Besseres/Geeigneteres (keine Ahnung...)? Ein paar Tips/Hinweise/Links wären großartig! Gr., redNoise (Diskussion) 15:34, 25. Mär. 2012 (CEST)

Die Oberfläche von Bauteilen, Booten und Yachten aus GFK besteht aus Gelcoat. I.d. Regel werden diese in sog. Negativ-Formen hergestellt, dazu wird in die mit Trennmittel behandelte Form erst einmal der Gelcoat - die spätere äußerste Schicht eingebracht - anschließend werden die Glasfasermatten Schicht für Schicht eingelegt, mit Kunstharz getränkt und etwaige Luftblasen herausgerollt (Handauflegeverfahren). Nachdem das Bauteil aus der Form draußen ist bildet der Gelcoat die glatte, widerstandsfähige (harte) Oberfläche.
Macken im Gelcoat kann man eigentlich recht gut mit Gelcoat ausbessern - mit einem anderen Material würde ich da absolut nicht dran gehen. Problematisch wirds nur, wenn die Schäden (Kratzer, Risse,...) oberflächlich / großflächig sind, ggf. muß man die schadhaften Stellen erst einmal richtig ausschleifen, den Gelcoat aufbringen und anschließend in Form / glatt schleifen. So ähnliche -nur in größerem Umfang- läuft das z.B. bei Osmose-Sanierungen, hierbei wird der komplette Gelcoat abgefräst und nach der Behandlung wieder neu aufgebracht. --Btr 20:40, 25. Mär. 2012 (CEST)
Ja, danke für die Auskünfte. Kann ich denn davon ausgehen, daß die Oberfläche eines geschätzt 25, eher 30 Jahre alten Bootes aus diesem Material besteht? Und, da es sich (hätte ich ja auch gleich genauer beschreiben können...) nicht nur um Kratzer und Riefen handelt, sondern eben auch um Risse, die z.B. durch unsachgemäße Befestigung (Verformung durch "Walken", dann Riss) auf dem Bootswagen entstanden sind, bildet das (der?) Gelcoat bei nachträglicher Anwendung eine mechanisch wie neue Oberfläche oder werden bei der nächsten mechanischen Verformung die Risse dann durch die neue Schicht durchkommen? Sorry, ich bin da absoluter Laie und in den diversen Foren usw. wird das Verfahren immer als bekannt und unproblematisch beschrieben - das mag ich aus dem Stand nicht so recht glauben... Gr., redNoise (Diskussion) 21:36, 25. Mär. 2012 (CEST)
Glasfaserverstärkter Kunststoff ist immer mit Gelcoat versiegelt. Wenn man sich bei den Segelbooten umsieht (Liste von Bootsklassen) kann man feststellen, dass GFK (neben früher Holz oder Sperrholz) im Bootsbau seit Jahrzehnten etabliert ist. Z.B. der Laser (Bootsklasse) wird von Anfang (Entwurf 1970) an aus GFK gebaut, wobei es mit Sicherheit noch ältere Boote/Bootsklassen aus GFK gibt.
Bei diesem Schaden (Photo) sieht man den Aufbau mit dem eigentlichen GFK (das faserige Zeug), dem Gelcoat (das weiße mit den Bruchkanten) und Farbe (blau).
Mechanische Verformungen sollte es bei GFK eigentlich erst gar nicht geben, allerdings kennt man es bei bestimmten Bootsklassen, wie z.B. der 420er Jolle, 470er Jolle,...und ähnlichen extremen "Leichtbauten", dass die Rümpfe aufgrund der dünnen Materialstärken und den ständigen mechanischen Belastungen an typischen Stellen weich und damit mit der Zeit unbrauchbar werden - diese ausgelutschten Rümpfe werden und können zwar noch problemlos im Freizeitsport gesegelt werden, aber im Regattasport, wo es um Spitzenleistungen geht werden die Boote alle paar Jahre ausgetauscht.
Wenn der GFK keine ausreichende Steifigkeit/Festigkeit mehr besitzt und mechanische Verformungen zuläßt wird meiner Meinung nach der recht harte Gelcoat wieder Risse bekommen. Bestimmte Bereiche, Risse oder Schäden auszubessern ist das eine, aber ob ich einen kompletten Rumpf mit einem neuen Gelcoat versehen würde ..., ob das rentabel ist???--Btr 00:24, 26. Mär. 2012 (CEST)
Gut.Jetzt bin ich schlauer. Da ich aus dem Stand kein neues Kajak kaufen kann und ich mich meiner "Potjomkin" doch sehr verbunden fühle, wird das jetzt eine reine Rechenaufgabe. Vielleicht laminiere ich ihr ein paar Spanten ein. Vielen Dank für die ausführliche Antwort! Gr., redNoise (Diskussion) 05:41, 26. Mär. 2012 (CEST)
Vielleicht laminiere ich ihr ein paar Spanten ein. Ein Tipp, GFK Bauteile werden/wurden oft als Sandwichkonstruktionen gebaut. Besonders bei so Sachen wie Spanten bietet sich das an, dazu sägst du dir den Spant z.B. aus Sperholz aus, fixierst ihn beispielsweise mit ein paar (kleinen) Keilen vorsichtig im Rumpf und muß ihn nur noch mit ein paar Matten GFK einlaminieren bzw. überziehen. Aber Vorsicht beim Laminieren, die Kunstharz-Dämpfe sind nicht ungefährlich - normalerweise wird so etwas unter Atemschutz gemacht, wenn wir an unseren Segelbooten Kleinigkeiten reparieren, dann am besten im Freien bei ausreichender Belüftung! Außerdem bieten sich Einweghandschuhe an - das Kunstharz klebt fürchterlich... --Btr 08:21, 26. Mär. 2012 (CEST)
Danke, laminieren kann ich zum Glück ganz gut (sie hat schon Flaschen/Dosenhalter ). Nochmals, vielen Dank für die umfassenden Hinweise! Gr., redNoise (Diskussion) 10:36, 26. Mär. 2012 (CEST)
Archivierung dieses Abschnittes wurde gewünscht von: Gr., redNoise (Diskussion) 05:41, 26. Mär. 2012 (CEST)

Schwarzmeerflotten Schiff

In diesem Artikel wird die Iman genannt aber ich kann das Schiff bei der Schwarzmeerflotte nicht finden. Als ich in Google die Klasse fand und auch eine Bestätigung dass dieses Schiff doch der Schwarzmeerflotte angehört, las ich allerdings dass es ein Tanker ist. Jetzt stellt sich mir die Frage was dieser Tanker denn nun mit den Marineinftatriesten macht? Oder ist das Normal das mit Tankern auch Truppen verlegt werden.--Sanandros (Diskussion) 11:19, 20. Mär. 2012 (CET)

Tanker gehören auch zur Kriegmarine, sind ja Versorgungsschiffe. So wie Tankflugzeuge auch zur Luftwaffe gehören. -- 208.48.242.106 11:34, 20. Mär. 2012 (CET)
Der Vergleich hinkt, mit Tankflugzeugen werden auf jeden Fall keine Einheiten verlegt.--Sanandros (Diskussion) 20:03, 20. Mär. 2012 (CET)
Die Iman (ein Schiff der Olekma-Klasse), dürfte wohl als Trossschiff bezeichnet werden können. Mit so etwas kann man also schon Personen transportieren und die Russen sind ja sowieso Weltmeister im Improvisieren, bei denen dann also erst recht. --Pilettes (Diskussion) 21:52, 20. Mär. 2012 (CET)
OK werden denn, wie im Artikel Beschrieben, Russische Trossschiffe zivil oder militärisch beflagt?--Sanandros (Diskussion) 09:31, 21. Mär. 2012 (CET)

Die Iman gehört zur Schwarzmeerflotte ([21]) und obwohl zivil anmutend angestrichen, ist sie militärisch spätestens dann, wenn sie Infanteristen in den Einsatz transportiert. --178.196.103.86 12:39, 23. Mär. 2012 (CET)

Aber im Artikel Trossschiff steht das die zivil oder militärisch agieren können, aber wenn sie zivil sind und Truppen mit Waffen (Sturmgewehren usw) transportieren, dann verstösst das gegen das Völkerrecht. Somit kann man nicht einfach sagen sie sind militärisch spätestens dann wenn sie Truppen transportieren.--Sanandros (Diskussion) 13:56, 24. Mär. 2012 (CET)
Als Truppentransporter agieren sie eben nicht mehr zivil. --178.195.182.247 10:42, 25. Mär. 2012 (CEST)
Ja und genau den Hintergrund will ich wissen, ich will genaue Fakten wissen ob sie wirklich nicht zivil sind oder vlt doch. Weil für militärische und zivile Schiffe gelten andere Gesetze.--Sanandros (Diskussion) 16:27, 25. Mär. 2012 (CEST)

Frage zur Steuererklärung

Guten Abend, ich habe eine Frage zur Einkommenssteuererklärung. Ich bezahle monatlich in die Pensionskasse meines Arbeitgebers ein (vom Nettolohn), zusätzlich einen freiwilligen Betrag durch Gehaltsumwandlung (vom Bruttolohn). Letzteres ist ja steuerfrei und daher in der Steuererklärung nicht zu berücksichtigen. Wie verhält es sich jedoch mit dem vom Nettolohn bezahlten Beitrag? Kann ich den geltend machen, z.B. in Zeile 5 der Anlage Vorsorgeaufwand? --82.113.98.191 22:32, 21. Mär. 2012 (CET) P.S.: So verstehe ich zumindest [22], Abschnitt zu Zeile 4 bis 10 auf Seite 9.

Frag dein Finanzamt, die können dir zu konkreten Fragen Auskunft geben und müssen es auch. -- Janka (Diskussion) 23:57, 21. Mär. 2012 (CET)
Berufsständig ist es ja nicht, oder? Das wäre eher sowas wie über die Berufsgenossenschaft. Wie es mit Betriebsrentenkassen aussieht solltest du lieber im zuständigen Finanzamt erfragen. Falls es zutrifft, dann müsstest du, wenn ich das von dir verlinkte Dokument richtig deute, auch den Arbeitgeberanteil mit eintragen. -- 208.48.242.106 09:07, 22. Mär. 2012 (CET)
Es würde mich sehr wundern, wenn das nicht in Zeile 5 gehört. --Grip99 02:34, 24. Mär. 2012 (CET)

Das weiß ganz sicher das Lohnbüro Ihres Arbeitgebers.--Geometretos (Diskussion) 14:22, 22. Mär. 2012 (CET)

Das Finanzamt sagt, das geht nur, wenn der ertse Beitrag vor 2005 gezahlt wurde. Das glaube ich aber nicht so recht (dieses Datum gilt bei Rentenversicherungen mit Kapitalwahlrecht und Kapitallebensversicherungen). Ich trage es jetzt einfach mal in Zeile 5 ein, den AG-Beitrag in gleicher Höhe in Zeile 9. --82.113.98.33 19:24, 25. Mär. 2012 (CEST)

Typografie bei "Dr. h. c.", "Dr. rer. pol." etc.

Setzt man hier zwischen die Bestandteile des Doktorgrades ein ganzes oder halbes Leerzeichen oder gar kein Leerzeichen? Gerade bei "h.c." habe ich schon öfter gesehen, dass es weggelassen wird, bei "rer. pol." sehe ich oft ein ganzes. Kann aber auch Einbildung sein. Nach meinem Gefühl würde ich überall ein halbes Leerzeichen setzen, so wie bei "z._B." (wobei ich dort anders als bei Wikipedia vorgelebt eher gar keines als ein ganzes Leerzeichen setzen würde). 130.149.229.189 09:28, 22. Mär. 2012 (CET)

Leerplatz? Oder keinen drin? So schreibt es die Sekretärin! GEEZERnil nisi bene 09:33, 22. Mär. 2012 (CET)
Zwischen den einzelnen Elementen stet meist ein Leerzeichen, wobei h.c. bzw. e.h. teiweise ein eigenes element darstellen. Also Dr. Ing. Dr. rer. pol. Dr. e.h. Dr. Dr. h.c. Jakobuel Titelhäscher Aber hier steht z.B. Univ.-Prof. Dr.-Ing. habil. Prof. E. h. Dr.-Ing. E. h. Dr. h. c. Reimund Neugebauer -- 208.48.242.106 09:49, 22. Mär. 2012 (CET)
honoris causa sind zwei Wörter, ehrenhalber ist eines. Von daher halte ich es bei „e.h.“ für sinnvoll, das Leerzeichen wegzulassen, bei „h. c.“ eher nicht (auch wenn der Duden es anders sagt). Es müsste aber unbedingt ein nicht umbrechendes und möglichst schmales sein; sonst lieber gar keines als ein ganzes, das womöglich am Zeilenende ein einsames h. stehenlässt. --Jossi (Diskussion) 13:22, 22. Mär. 2012 (CET)
Da Jossi den Duden angesprochen hat (der liefert übrigens auch eine Begründung für "E. h."): Dort heißt es: "E. h. = Ehren halber (frühere Schreibung von ehrenhalber), z. B. in Dr.-Ing. E. h." bzw. "Ehrendoktor, Doktor ehrenhalber oder Ehren halber (Abk. Dr. eh., Dr. e. h. und Dr. E. h.; vgl. E. h.), Doktor honoris causa (Abk. Dr. h. c.)" und "Doktor[in] der Staatswissenschaften (Abk. Dr. rer. pol. oder Dr. sc. pol. oder Dr. oec. publ.)". Im Wörterbuch von Campe ist die Schreibung "Ehren halber" übrigens noch zu finden - ist also schon recht alt, vgl. auch den Eintrag bei Adelung oder in Georges lat.-dt. Handwb. (Lemma "honorarius") (Aufl. 1855, in der von 1913 dann "ehrenhalber"). Die Erklärung für "E. h." ist auch im DDR-Duden von 1969 zu finden (S. 107), dort jedoch ein Druckfehler: "vgl. auch Dr.h. c. [sic] im A[bkürzungs]V[erzeichnis]", dort richtig "Dr. h. c.". Die Auflage von 1984 ist identisch, jedoch ohne den besagten Druckfehler. Das Bertelsmann-Wb. von 1996 bietet "Dr. E. h." und "Dr. h. c." - also auch mit Leerzeichen.--IP-Los (Diskussion)
Was wieder einmal zeigt, dass ich erst nachschauen sollte, bevor ich hier rumschwalle... IP-Los hat wie immer recht: Der Duden (25. Auflage) hat sehr wohl ein Leerzeichen bei „h. c.“, die gegenteilige Behauptung in der von Geezer oben verlinkten Sekretärinnen-Service-Seite ist schlicht falsch (oder veraltet). Und wenn man schon darauf abheben will, dass „ehrenhalber“ ein Wort ist, ist der Duden-Vorschlag „eh.“ natürlich logischer als „e.h.“ Ein nicht umbrechendes Leerzeichen sollte es trotzdem in jedem Fall sein. --Jossi (Diskussion) 16:34, 25. Mär. 2012 (CEST)

Halfenschienen

Gibt es einen herstellerunabhängigen Begriff für Halfenschienen (http://www.halfen.de/t/22_679.html)? So in der Art „einbetoniertes Montageschiene (C-Profil)“? --тнояsтеn 15:21, 23. Mär. 2012 (CET)

Ankerschienen sagt eine bekannte Onleinzykopädie dazu. --Optimum (Diskussion) 15:32, 23. Mär. 2012 (CET)

Danke, habe mal Halfenschiene angelegt als Weiterleitung angelegt. --тнояsтеn 15:50, 23. Mär. 2012 (CET)
Ich kenn die Dinger als C-Schienen. --Rôtkæppchen68 15:33, 23. Mär. 2012 (CET)
Ich kenn die nur als Halfenschiene, daher die Frage ;) Die im o.g. Artikel erwähnte CEN/TS 1992-4 spricht auch von Ankerschienen, das scheint der korrekte Fachbegriff zu sein. --тнояsтеn 15:50, 23. Mär. 2012 (CET)
C-Schienen kenne ich aus Fahrzeugen der Bundeswehr. Entweder als Gitterrost montiert oder als Teil einer Tischplatte o.ä. Da wurden dann Geräte oder auch kleine Kisten (privat) mit befestigt. Der Unterschied ist die Verbindung mit dem Untergrund. --Eingangskontrolle (Diskussion) 21:28, 25. Mär. 2012 (CEST)

Mit einer Webseite via API kommunizieren

Ich bin auf eine Webseite gestoßen, die ein API anbietet, das auf WSDL und XML basiert. (Die Webseite hier zu nennen, wäre unzulässige Werbung, oder?) Es gibt ausführliche Informationen über die Klassen, Methoden etc., die das API zur Verfügung stellt. Da ich Programmierer bin, müsste ich es eigentlich hinbekommen, Code für dieses API zu schreiben. Wozu ich aber nirgends Informationen finde: Wenn ich den Code geschrieben habe, wie kann ich ihn dann testen, d. h. wie übermittle ich ihn zum API, um zu sehen, ob er auf der Webseite den gewünschten Effekt hat? Wie funktioniert diese Übermittlung bei WSDL bzw. XML? Die Betreiber der Webseite waren diesbezüglich leider überhaupt keine Hilfe. --(nicht signierter Beitrag von 212.186.66.201 (Diskussion) 23. März 2012, 18:38 (UTC))

laut WSDL: „Austausch von Nachrichten“... also: lokal läuft die selbstgetippte applikation und schickt nachrichten an den remote host, die durch WSDL nachrichten dort dann das API benutzen... --Heimschützenzentrum (?) 19:56, 23. Mär. 2012 (CET)
Das sollte dann ein Webservice sein. Kommunikation via SOAP. Zum Testen hab ich da früher immer SoapUI genommen. -- TZorn 18:35, 25. Mär. 2012 (CEST)

Kopfhörer

Welche Kopfhörer mit Hörmuscheln (keine Ohrlinge) und sind für mp3player und Laptop geeignet und liefern einen guten Klang? Worauf muss man achten bzw. kann jemand einfach einen Namen sagen. 46.115.19.106 11:02, 24. Mär. 2012 (CET)

Geeignet sind vom Prinzip her alle, und ab einer bestimmten Preisklasse hängt das Meiste von der Qualität der Ausgangsquelle ab (Erwarte vom Notebook mit Standardsoundkarte keine Wunder). Von den technischen Daten her, achte auf Schalldruck (Höher ist besser, besonders wenn am Ausgabegerät etwas wenig kommt) und Frequenzgang. Aber letztlich sind Daten Schall und Rauch, weil Hören eine sehr subjektive Tätigkeit ist; deshalb: Probehören, und wenn man dem eigenen Urteil nicht recht vertraut, Testberichte von Profis lesen. Ein Referenzname, wenn's denn sein muss, fängt mit Senn- an. 85.180.196.206 11:15, 24. Mär. 2012 (CET)
Ja danke. Ich meine mit kleinem Stecker fürs Laptop. Ist das hier [23] was oder ist das Müll? ich habe absolut keine Ahnung. Kann mal jemand einen Link schicken was ich kaufen soll? Wie heißt denn dieser Audiostecker, der ins Laptop rein geht? 46.115.19.106 12:11, 24. Mär. 2012 (CET)
3,5-mm-Klinkenstecker sind mittlerweile Standard; bei Handys gibt es auch mal 2,5 mm, dafür (und für diverse proprietäre Anschlüsse) kann man Adapter kaufen. Das verlinkte Teil kenne ich nicht, die Beschreibung klingt nach Bassgewummer. Wenn das das Gesuchte ist (wie gesagt, subjektiv...), warum nicht. Ansonsten verweise ich noch einmal aufs Probehören im Laden. Gruß 85.180.196.206 12:28, 24. Mär. 2012 (CET)
Empfehlen kann ich da Kopfhörer von Sennheiser. Liefern wirklich guten Klang und satten Bass. Preisklasse um die 50 euro aber das Geld auf jeden fall wert. Hab ich bis jetzt immer benutzt und bin damit sehr zufrieden. --SuburbanZombie (Diskussion) 13:30, 26. Mär. 2012 (CEST)

Wo vor Abflug aus Frankfurt übernachten

Hallo zusammen, im August soll ich meine Freundin um 7:30 an den Frankfurter Flughafen bringen. Da mir der nächtliche Aufbruch gegen vier Uhr zu stressig ist, würden wir gerne irgendwo "vor Frankfurt" in einem preiswerten Hotel, einer Pension o.ä. übernachten. Viele Dinge vor Ort anschauen können, werden wir bei Ankunft gegen 17/18 Uhr ja vermutlich nicht mehr können. Aber ein nettes Restaurant in der Nähe wäre dennoch nett. Hättet ihr eine Idee, wo man das alles aus Richtung Freiburg kommend, erleben könnte? Gruß, --Flominator 12:13, 24. Mär. 2012 (CET)

spart geld. --Janneman (Diskussion) 12:22, 24. Mär. 2012 (CET)
Er hat eine Dame dabei ! Google => Frankfurt nettes kleines Hotel Restaurant nahe am Flughafen <= GEEZERnil nisi bene 12:24, 24. Mär. 2012 (CET)
Da fällt ja jemand aus der Statistik raus: Benutzer mit Weib an der Seite. :) Wobei es nicht der einzige ist. ;) ... 93.134.4.215 12:42, 24. Mär. 2012 (CET)
Falls das Hotel tatsächlich nur fürs Übernachten ist und nicht zuviel kosten darf, würde ich mir Motels am Offenbacher Stadtrand (Kaiserleiviertel) anschauen (etwa Ibis oder Etap); die sind relativ günstig, vor allem, weil man pro Zimmer zahlt. Die Gegend selber ist natürlich trostlos (Büroglasfronten, Baumarkt, Mäckes, ein paar dunkle Ecken), aber: Man kommt direkt von der A661 hin und ist morgens ruckzuck am Flughafen; am Main entlang ist es ein schöner zehnminütiger Spaziergang zur bekannten Gerbermühle, oder, für billiger, ein paar Schritte weiter ins Rudererdorf; oder man setzt sich am Kaiserlei in die S-Bahn und fährt in drei Minuten ins Frankfurter Zentrum. Grüße 85.180.196.206 13:19, 24. Mär. 2012 (CET)
Danke. Das ist in der Tat ein brauchbarer Lösungsansatz. --Flominator 13:57, 24. Mär. 2012 (CET)

Ist das wirklich eine Frage für die Auskunft? Eigentlich sollte die Antwort doch mehrfach am Tag in der TV-Werbung zu sehen sein. Ich komm das sofort auf diverse Angebote zwischen 47-60 Euro [24]. Ist doch eigentlich genau der Fall für Frühbucherrabatt, nicht die Frage, wo es preiswerte Absteigen gibt, die man nicht stundenweise bezahlt.Oliver S.Y. (Diskussion) 13:28, 24. Mär. 2012 (CET)

Es ist insofern eine Frage für die Auskunft, da ich mir noch nicht einmal sicher war, welche Stadt/Gemeinde "vor Frankfurt" sich dafür überhaupt eignet. Gruß, --Flominator 13:57, 24. Mär. 2012 (CET)
Persönliche Erfahrung: Ich hatte ähnliche Erlebnisse / Anforderungen, habe selbst nach FFM 3 Stunden Anfahrt und bin schon öfter früh morgens (USA...) dort abgereist. Ich habe mich in der Jugendherberge eingemietet. Dort sollte man auch Doppelzimmer für die traute Zweisamkeit bekommen. Dann morgens zum Südbahnhof (10 Min. zu Fuss oder mit der Strassenbahn) und von dort mit dem Regionalzug direkt weiter zum Flughafen. Gesamte Reisezeit ca. 30-40 Minuten. War ein Kompromiss aus nah am Flughafen (eher nicht, aber akzeptabel) und preiswert. Ist natürlich Geschmackssache. Heute würde ich vermutlich ein paar Euro drauflegen und in einem Hotel näher des Flughafens - oder, vermutlich günstiger - nahe des Südbahnhofs absteigen. Wenn die JuHe in Frage kommt einfach mal auf bahn.de nach Verbindungen vom Deutschherrnufer 12 nach Frankfurt Flughafen suchen. Die Strecke ist sehr kurz, der Flughafen liegt unmittelbar vor dem Stadtzentrum westlich des Hauptbahnhofs. Über die Verbindungsabfrage bekommt man auch passende Strassenbahnen zur nächsten S-Bahn-Station angezeigt und je nach Anschluss variiert die beste S-Bahn-Station bzw. der Bahnhof. Länger als 45 Minuten ist man aber nie unterwegs. --Manuel Schneider(bla) (+/-) 15:26, 24. Mär. 2012 (CET)
Das Gute daran: Die Jugendherberge liegt am Rande des Apfelweinviertels ,,Alt-Sachsenhausen"! --85.180.138.204 18:59, 24. Mär. 2012 (CET)

das schon gesehn: [25]? und ansonsten war da mal direkt vor diesen check-in schaltern son cooler tunnel an dessen ende eine hotel lobby zu sein schien... vielleicht hatte ich aber auch grad wieder stoffwechselstörungen... :-) --Heimschützenzentrum (?) 17:47, 24. Mär. 2012 (CET)

Nee, nee, stimmt schon: Das Hotel am Ende des coolen Tunnels (klingt wie ein Roman von Douglas Adams) ist das Hilton; wenn man sich's leisten kann, sicher eine gute Adresse. Grüße 85.180.196.206 18:48, 24. Mär. 2012 (CET)

Wir haben in Frankfurt im Hotel Mainstation An der Mainkur 7, 60386 Frankfurt am Main - Fechenheim übernachtet. Nachteil: Das Hotel liegt direkt am Bahnhof Frankfurt-Mainkur (Strecke FFM Süd <=>Hanau), offene Fenster im Sommer nur empfehlenswert, wenn man Güterzüge mag... Vorteil: Das Hotel liegt direkt am Bahnhof Frankfurt-Mainkur, direkte Anbindung an den ÖPNV... Preiswerte Übernachtung, das Frühstück haben wir aber nicht probiert, weil wir aushäusig gebruncht haben... Falls Du doch in Offenbach übernachten willst: Verrat das bloß keinem Frankfurter... Du weißt ja: Autokennzeichen F = Fehler, OF = null Fehler... *schnell in Deckung spring...* Die Frankfurter und Offenbacher lieben sich heiß und innig. Noch lieber, als Badener und Württemberger. Grüße Marcus (Diskussion) 20:14, 24. Mär. 2012 (CET)

Das haste valsch ferstanden, OF = Ohne Ferstand (oder Ohne Führerschein), und FB steht für Forsicht Bauer. :-) --Zerolevel (Diskussion) 19:35, 25. Mär. 2012 (CEST)
Falls du mit dem Auto unterwegs bist und vom Lande kommst, in Frankfurt ist Umweltzone (siehe http://gis.uba.de/website/umweltzonen/pdf/Frankfurt.pdf da), da darfste ohne Pickerl nicht reinfahren. Bliebe also nur Offenbach oder die Gegend. --Hareinhardt (Diskussion) 20:47, 24. Mär. 2012 (CET)
Wie streng wird das kontrolliert? --149.172.233.70 09:30, 25. Mär. 2012 (CEST)
BTW, nicht nur in/bei Frankfurt suchen, auch Neu-Isenburg und Rüsselsheim sind ziemlich pkw-nah am Flughafen. --Zerolevel (Diskussion) 21:50, 24. Mär. 2012 (CET)

Nachtrag: Wir haben nun hier übernachtet und sind ziemlich begeistert. --Flominator 19:45, 5. Aug. 2012 (CEST)

GSG-9 Headset

Weiß jemand welche Art von Headset die GSG-9 verwendet?--79.251.107.152 12:47, 24. Mär. 2012 (CET)

Die GSG-9 weiß das bestimmt. --Sr. F (Diskussion) 14:34, 24. Mär. 2012 (CET)
Einfach mal bewerben, die suchen händeringend gute Leute (überdurchschnittliche Sportlichkeit ist nur ein kleiner Teil der Anforderungen). Seit Jahrzehnten werden die Anforderungen ständig heruntergeschraubt, sonst könnten die 250 Dienstposten nicht besetzt werden. --93.133.196.176 16:05, 24. Mär. 2012 (CET)
Ich könnte mir glatt vorstellen, das behalten die für sich, z. B. um das Bauen eines passenden Störsenders zu erschweren. --Eike (Diskussion) 17:02, 24. Mär. 2012 (CET)
Ich meinte nicht den genauen Typ, sondern ob die eher so große Headsets mit riesigen Kopfhörern oder eher etwas kleineres, z.B. wie für's iPhone verwenden.--79.251.107.152 19:58, 24. Mär. 2012 (CET)
laut Pro7's galileo übertragen die den schall nicht ins ohr, sondern auf den den schädelknochen... also nix, was normale menschen wollen... --Heimschützenzentrum (?) 21:29, 24. Mär. 2012 (CET)
Es gibt wohl Firmen, die glauben, dass auch normale Menschen daran Interesse haben könnten. http://www.bz-berlin.de/ratgeber/computer/musik-bis-auf-die-knochen-article1417186.html --91.66.120.154 17:18, 25. Mär. 2012 (CEST)

Da der Artikel leider völlig unbelegt ist, hier die Nachfrage, ob das Lemma wirklich die Originalbezeichnung ist, und ob man einen Erfinder oder ersten Patentinhaber dafür benennen kann.Oliver S.Y. (Diskussion) 13:08, 24. Mär. 2012 (CET) PS:Ich halte dort auch die Zeitangabe der 50er Jahre für falsch, da ich relativ viele alte Darstellungen kenne, wo schon während des 2.WK diese Schnüre verwendet wurde, Beispiele dafür bei Commons 1944 und 1937.Oliver S.Y. (Diskussion) 13:12, 24. Mär. 2012 (CET)

Hervorragende Eigenschaften zeigt die in letzter Zeit vielfach verwendete Spiralschnur, die sich auf ihre fünffache Länge ausdehnen läßt.Quelle: Bauelemente Nachrichtentechnik von Kurt Pribich in Zusammenabeit mit Dipl.-Ing Helmut Haslinger, 5. Auflage, Bohmann Verlag Heidelberg, 1971 Gruss --Nightflyer (Diskussion) 13:27, 24. Mär. 2012 (CET)
(BK) In diesem Fall handelt es sich um ein normales textilumflochtenes Kabel, das wendelförmig aufgewickelt wurde. Langziehen ist hier weder vorgesehen noch möglich. Bei den Telefonen der Baureihen W38 und W48 gab es das „Spiral“kabel nur als Reparaturteil, original war eine glatte, geflochtene Schnur dran. (Rôtkæppchen68 hat ein funktionsfähiges W48 auf dem Schreibtisch stehen) --Rôtkæppchen68 13:31, 24. Mär. 2012 (CET)
Vor 1940, denn das Patent hier bezieht sich darauf und beschreibt eine Vermeidung des Vertwiddelns dieses spiraligen Kabels. Gefunden mit Google => "coiled telephone cord" "patent" <= GEEZERnil nisi bene 13:58, 24. Mär. 2012 (CET)
Nicht ganz. Dein Patent ist von 1985 und es bezieht sich auf ein Patent von 1940, in dem es um einen drehbaren mehrpoligen Steckverbinder mit Schleifringen geht. --Rôtkæppchen68 14:24, 24. Mär. 2012 (CET)
Bleibe bei "vor 1940": "1938 retractile, spring, or spiral cords were introduced into the Bell System. A single cotton bundle containing the handset's four wires were fashioned into a spiral. This reduced the twisting and curling of conventional flat or braided cords. Spiral cords were popular immediately. AT&T's Events in Telecommunication History reported that introduction began in April, with Western Electric providing 6,000 cords by November. Still, even with W.E. then producing 1,000 cords a week, the cords could not be kept in stock." Natürlich noch nicht Plastik. GEEZERnil nisi bene 16:02, 24. Mär. 2012 (CET)
Ich habe irgendwo gelesen, dass die in ersten Deutschland verwendeten Spiralschnüre schnell ausgeleiert sind, weil deren Elastikkomponente noch aus Gummischnur bestanden hat. Das Weich-PVC wurde erst in den 1930er-Jahren erfunden, wobei die ersten Weichmacher noch nicht so gut waren wie die heutigen, siehe Igelit. Spätestens ab Mitte der 1950er-Jahre waren gewendelte Telefonkabel aus PVC und Telefongehäuse aus ABS dann üblich. --Rôtkæppchen68 16:14, 24. Mär. 2012 (CET)

Danke allen für die sehr erhellenden Detailinformationen zu solchem Randthema. Vieleicht könntet ihr den Artikel nochmal textlich überprüfen, und die ein oder andere Quelle hinzufügen. Es erscheint mir sehr "allgemein" gehalten zu sein.Oliver S.Y. (Diskussion) 20:52, 24. Mär. 2012 (CET)

Erst schreiben wir's an die Tafel - und dann sollen wir den Schulhof fegen ? GEEZERnil nisi bene 10:45, 25. Mär. 2012 (CEST)
 
Western Electric Model 500
Ich hab noch ein bisschen gegoogelt und bin fündig geworden. Das [26] Western Electric 500 gibt es seit 1949. Es ist das älteste mir bekannte Telefon, das serienmäßig eine PVC-Wendelschnur hat. Mein eigenes Western Electric 500, die Militärversion TA 236/FT, ist von Mitte der 1950er-Jahre. Das Telefon liegt leider eingemottet irgendwo in der Abstellkammer. --Rôtkæppchen68 00:16, 26. Mär. 2012 (CEST)
Noch etwas zum Telefongyár-Telefon von 1937: Das gleiche Telefon ist hier als Fernsprechapparat vom Typ ZB 35 mit glatter Handapparateschnur abgebildet. „Unser“ Foto zeigt möglicherweise ein repariertes Telefon. postamuzeum.hu führt den Fernsprechapparat vom Typ ZB 555 von 1955 als erstes Telefon mit gewendelter Handapparateschnur auf. Das Ericsson-Telefon von 1920 ist sehr wahrscheinlich ebenfalls repariert, da das Weich-PVC-Kabel erst in den 30er-Jahren erfunden wurde. Das erste Ericsson-Telefon mit gewendelter Handapparateschnur ist vermutlich das Ericsson Dialog von 1961. Auf ericssonhistory.com ist hier zwar ein Bakelittelefon mit gewendelter Handapparateschnur zu sehen, laut Bildunterschrift war das aber in den 1960er-Jahren, also nach Einführung des ABS/PVC-Telefons Dialog. --Rôtkæppchen68 15:14, 26. Mär. 2012 (CEST)
Habe den ganzen Klumpatsch in die dortige Disk. kopiert. Arbeit für unsere Enkel... GEEZERnil nisi bene 15:25, 26. Mär. 2012 (CEST)

Suche den Internet Explorer 4.01 SP2 auf Deutsch

weiß jemand, wo es den noch gibt? Bei Microsoft habe ich nichts gefunden.

--93.132.170.170 18:36, 24. Mär. 2012 (CET)

sehr gute frage, suche auch schon sehr lange nach allen versionen in deutsch um das feeling der zeit des web erlebins zu erleben xD --Diamant001 (Diskussion) 18:41, 24. Mär. 2012 (CET)
Ich brauche ihn eigentlich nur für den Active Desktop von Windows NT 4.0 --93.132.170.170 18:45, 24. Mär. 2012 (CET)
Keine Ahnung ob es den noch irgendwo gibt, aber es gibt definitiv einfachere Möglichkeiten sich allerlei Digitales Ungeziefer einzufangen!!! --Grüße von Jogo.obb Disk 18:48, 24. Mär. 2012 (CET)
Wie schon gesagt, ich brauche ihn nur für den Active Desktop. Ich möchte nicht damit ins Internet. --93.132.170.170 19:22, 24. Mär. 2012 (CET)
Hier, bitteschön: http://winhistory.de/downloads/update.php/ie40act.exe --85.180.138.204 18:53, 24. Mär. 2012 (CET)
Vielen Dank. Ich frage mich, wie du den gefunden hast. Ich hab mir eine halbe Stunde lang einen Wolf gesucht. --93.132.170.170 19:24, 24. Mär. 2012 (CET)
guck mal bei chip.de nach --178.0.52.96 14:10, 25. Mär. 2012 (CEST)

Alte Versionen gibts immer unter Oldversion.com -- 208.48.242.106 11:42, 26. Mär. 2012 (CEST)

Napoleon und St. Petersburg

Formal war St. Petersburg seit 1712 Hauptstadt von Russland (bzw. residierte der Hof dort). Nur zwischen 1728 bis 1732 zog der Hof komplett zurück nach Moskau, dann aber wieder nach Petersburg. Auch spätere Zaren des 18. Jahrhunderts residierten mal da und mal dort, aber an der offiziellen Residenz des Hofes macht man eben den anachronistischen Begriff "Hauptstadt" fest und so gewinnt man bei der Lektüre unseres Artikels Sankt Petersburg den Eindruck, die Stadt wäre schon kurz nach ihrer Gründung die wichtigste Metropole ganz Russlands gewesen. Was mir nun aber sonderbar erscheint ist, dass Napoleon in seinem Russlandfeldzug 1812 immer nur nach Moskau wollte und an eine Belagerung und Eroberung St. Petersburgs offenbar gar nie gedacht hat. Dabei hat seine Grande Armee von Kaunas aus den Feldzug gestartet und von dort ist St. Petersburg ungefähr gleich weit wie Moskau, sogar ein bisschen näher. Außerdem wäre in diese Richtung die Logistik leichter gewesen, man wäre nie besonders weit von der Ostseeküste entfernt gewesen und hätte Nachschub per Schiff bringen können. Aber Petersburg stand wie es scheint gar nie auch nur zur Diskussion. Offenbar muss Napoleon Moskau so viel bedeutender eingeschätzt haben, denn er erwartete sich ja dem Land durch die Besetzung der größten und wichtigsten Stadt einen Frieden nach seinem Geschmack aufzwingen zu können.

Also entweder war Petersburg damals noch relativ unwichtig, halt mit ein paar Palastbauten, aber sonst ein unbedeutendes Nest, oder Napoleon hat sich hier strategisch massiv verschätzt. Wer weiß dazu näheres? --El bes (Diskussion) 05:40, 25. Mär. 2012 (CEST)

Adolphe Thiers schreibt dazu in "Geschichte des Consulats und Kaiserreichs, Band 14" (1856) auf Seite 96:
"Von Preußen aus hätte er (Napoleon) statt Moskau auch Petersburg zum Ziel nehmen können; dieser Weg durch Kurland und Liefland war um ein Viertel kürzer aber noch schlechter als der durch Litthauen, da er fast gar keine Mittel für den Unterhalt der Menschen und Pferde darbot. Weder diese Richtung nach Petersburg, noch die nach Kiew konnte Napoleon nehmen, ohne sich der Gefahr auszusetzen von den Russen in der Flanke gefaßt zu werden. Der gerade Weg nach Moskau zwischen Düna und Dnjepr behielt also den Vorzug."
Die Seite "petersburg-info.de" interpretiert die Entscheidung so:
"Napoleon wollte Russland nicht am Kopf treffen sondern im Herzen. Sankt-Petersburg war zwar die offizielle Hauptstadt, doch die heimliche Hauptstadt war immer noch Moskau." --Snevern 11:58, 25. Mär. 2012 (CEST)
Ich kann mich noch an den Schulunterricht erinnern. Dort kam das Thema zwar nur kurz dran, aber enthielt ein paar wesentliche Punkte. Demnach hatte Rußland eine sehr schlechte Infrasturktur (wie offenbar auch heute noch). Um das Land zu erobern, genügte nicht wie gegen Preußen der Erfolg in einer Schlacht, und die Eroberung des Machtzentrums, sondern man mußte das infrastrukturelle Zentrum erobern. Und das war nunmal Moskau, da dorthin alle wichtigen Verkehrswege führten, die teilweise keinerlei sonstige Querverbindungen hatten. Darum war auch der Brand von Moskau von so großer Bedeutung, da damit der vorgesehene zentrale Standort für die Militärherschaft über das Land nicht zu halten war. Petersburg wäre ohne Moskau nicht zu halten gewesen, und wäre wahrscheinlich kampflos übergeben worden. Da hätte anders herum nicht funktioniert. Was die Entfernungen angeht, so vergaß Thiers bei seiner Einschätzung die Ostsee als Transportweg, die reine Kilometerangabe ist da nebensächlich, eher wäre es für die Versorgung vorteilhafter gewesen.Oliver S.Y. (Diskussion) 18:33, 25. Mär. 2012 (CEST)
Ich bezweifle, dass Thiers das einfach vergessen hat (und auch Napoleon hat derart wichtige Details zweifellos nicht übersehen). Wir befinden uns in Jahr 7 nach Trafalgar, und es wäre eine über 400.000 Mann starke Armee zu transportieren oder zu versorgen gewesen. Beim besten Willen: Nein. --Snevern 20:22, 25. Mär. 2012 (CEST)
+1 Wie z.B. die Seeschlacht von Kopenhagen zeigt, war die britische Flotte auch in der Ostsee präsent, und die Franzosen waren nicht in der Lage, hier nennenswerte Schiffsbewegungen durchzuführen. Andererseits wurde die "Grande Armée" nicht aus Frankreich oder von sonstwo her versorgt, sondern bezog den Großteil der benötigten Güter aus den eben eroberten Gebieten. --Optimum (Diskussion) 22:17, 25. Mär. 2012 (CEST)
Nach der zweiten Seeschlacht von Kopenhagen von 1807 wandte sich Dänemark aber dem pro-französischen Lager zu und Schweden unter dem Bernadotte-König war zumindest wohlwollend-neutral. Die Ostsee wäre also als Operationsgebiet schon möglich gewesen. Ich glaube aber, dass Petersburg strategisch einfach nicht so wichtig war, obwohl der Zar dort weilte. Mir scheint sogar, dass Riga damals als Stadt noch größer war als Petersburg und auch den bedeutenderen Hafen hatte. Gegenüber Riga hatte Napoleon Eroberungspläne und ließ sogar gewisse Vorbereitungen für eine Belagerung treffen, die er dann aber wieder aufgab. --El bes (Diskussion) 23:21, 25. Mär. 2012 (CEST)

Berichte von Betroffenen zur Verfolgung von Andersdenkenden und Minderheiten im Nationalsozialismus gesucht

Hallo zusammen. Ich benötige für eine Schulstunde Berichte von Betroffenen zur Verfolgung von Andersdenkenden und Minderheiten im Nationalsozialismus. Kennt jemand entsprechende Quellen, am besten online verfügbar (muss aber nicht sein)? Freundliche Grüße, --GGKHorst (Diskussion) 12:34, 25. Mär. 2012 (CEST)

Versuchs mal über die <Kategorie:NS-Opfer> , vielleicht findest du da einen (regionalen) Anhalt. Zudem hat das EL-DE-Haus eine schöne Seite erlebte Geschichte mit Videos. --G-Michel-Hürth (Diskussion) 15:54, 25. Mär. 2012 (CEST)
Verlinkungs-Service: Kategorie:NS-Opfer --84.191.137.216 17:19, 25. Mär. 2012 (CEST)
Außerdem sei auf die sogenannten Deutschland-Berichte der SOPADE hingewiesen, ein riesiger authentischer Fundus zur Situation in Nazi-Deutschland. Korrekter Titel: Deutschland-Berichte der Sopade. Nach dem Exemplar im „Archiv der sozialen Demokratie“ der Friedrich-Ebert-Stiftung neu herausgegeben und mit einem Register versehen von Klaus Behnken im Verlag Petra Nettelbeck, Salzhausen und Zweitausendeins, Frankfurt am Main, 1980 (1. bis 4. Auflage), (7 Bände). --84.191.137.216 17:25, 25. Mär. 2012 (CEST)
Herzlichen Dank an Euch beide für die wertvollen Hinweise! --GGKHorst (Diskussion) 18:13, 25. Mär. 2012 (CEST)
Auf den Seiten von Yad Vashem gibt es viele Berichte als Videos, schau mal hier. --84.191.137.216 19:05, 25. Mär. 2012 (CEST)
Kann ich ebenfalls sehr gut brauchen, in der allernächsten Stunde wird allerdings nur die Verfolgung von Nicht-Juden thematisiert, der Judenverfolgung wird dann im Anschluss eine eigene Stunde gewidmet. Danke für Euer Engagement, Ihr helft mir damit wirklich weiter! Leider steht die Verfolgung von Andersdenkenden und Minderheiten zwar in vielen Lehrplänen, wird aber in den Geschichtsbüchern nur kursorisch behandelt. --GGKHorst (Diskussion) 23:24, 25. Mär. 2012 (CEST)
Ich habe mir den Artikel Oral History nochmal angesehen. Dort wird auf zeitzeugengeschichte.de verlinkt. Schau mal hier. Der Pfiff ist, daß hier die Schülerinnen und Schüler die Befrager sind. --84.191.137.216 01:48, 26. Mär. 2012 (CEST) Addendum: Und schau doch auf dieser Seite der bpb nochmal deren Linkliste durch, das sieht auch vielversprechend aus. --84.191.137.216 01:58, 26. Mär. 2012 (CEST)
Hier gibt es auch einige Zeitzeugenberichte auf Video, sind auch Nicht-Juden wie z.B. Zigeuner oder politische Häftlinge dabei: http://www.mauthausen-memorial.at/index_open.php Liebe Grüße, --Haeferl (Diskussion) 02:59, 26. Mär. 2012 (CEST)

11.september 2001

Ich habe eine frage zu 11 september die situation der armen und die situation der moslems nach dem anschlag in den heimatländern der täter --178.0.52.96 14:08, 25. Mär. 2012 (CEST)

Hmm... Könntest Du die Frage bitte irgendwie verständlich formulieren? So wie das da oben steht ist es ja noch nicht einmal ein vollständiger Satz. Ein oder zwei passende Verben wären nett. -- CC 14:10, 25. Mär. 2012 (CEST)
Die Frage ist doch einigermaßen verständlich, da fehlt nur ein "wie ist": Ich habe eine Frage zum 11. September: Wie ist die Situation der Armen und die Situation der Moslems nach dem Anschlag in den Heimatländern der Täter?
Die Täter stammen mehrheitlich aus Saudi-Arabien, sowie aus den Vereinigten Arabischen Emiraten, Ägypten und dem Libanon. Alle diese Länder haben eine muslimische Bevölkerungsmehrheit (Saudi-Arabien am stärksten, Ägypten und Libanon mit großen christlichen Minderheiten). Daher dürfte sich an der Lage der Moslems in diesen Ländern nichts grundsätzlich geändert haben. Auch auf die Lage der Armen dürfte der Anschlag keine grundsätzliche Wirkung gehabt haben.
Der Fragesteller müsste also etwas konkretisieren, inwiefern er hier einen Zusammenhang annimmt. --::Slomox:: >< 09:43, 26. Mär. 2012 (CEST)
 
SA 330 Puma der R.A.F.
 
Original (ohne Rotorblätter und Räder)

Für welche Umgebung dient diese ungewöhnliches Tarnmuster?! Oder soll das einer optischen Täuschung dienen? --93.133.196.176 15:56, 24. Mär. 2012 (CET)

Tiger, nicht Reptil. Unikat, Aufmerksamkeit, cool, mehr Applikanten: "Die 230 Sqn. der RAF präsentierte diese sehr attraktive Lackierung beim Tigermeet 2005 in Balikesir, Türkei.". GEEZERnil nisi bene 16:15, 24. Mär. 2012 (CET)
Naja, als Säugetier stammt der Tiger ja über ein paar Umwege von den Reptilien ab, also wenn man beide Augen zudrückt kann man ihn immer noch Reptil nennen. Reptil-Katze oder Katzen-Reptil oder wie auch immer. :D -- Chaddy · DDÜP 20:11, 24. Mär. 2012 (CET)
 
Schlange mit schwarz-oranger Musterung

Hier mal ein Reptil in dieser Farbgebung (Schlange) zum Vergleich. --93.133.196.176 16:22, 24. Mär. 2012 (CET)

OK "Tigermeet" >>> Kleine Hommage an diese Zusammenkunft (kein militärisch praktiziertes Tarnmuster wahrscheinlich). Soll wohl nur ein Hingucker sein ... --93.133.196.176 16:24, 24. Mär. 2012 (CET)
Ja, und ein bissl britischer Humor ("Anybody got an idea, how to make the Turks grin?" "Tiger pattern on copter??" "Splendid, Bean! Go ahead!"): "... this event was hosted by the Tiger squadron of the Turkish Air Force". GEEZERnil nisi bene 16:32, 24. Mär. 2012 (CET)
Yeah, seeehr britischer, besser: englischer Humor, denn aus der Kolonialgeschichte dieser kleinen Insel ist ja im Laufe der Jahrhunderte ein ganz besonderes Verhältnis zu Tigern gewachsen. Nicht umsonst lautet einer der bekanntesten Limericks: There once was a lady from Niger / Who smiled as she rode on a tiger / They came back from the ride / With the lady inside / And the smile on the face of the tiger. - Und was könnte englischer sein als ein Limerick... Der getarnte Hubschrauber kann offenbar noch mehr, als man auf den ersten Blick erkennt: Hier z.B. mit einer noch geschickteren Tarnung: der Hubschrauber hat sich in ein Sofa verwandelt und läßt sich energiesparend eine Treppe hinuntertragen. --84.191.136.161 17:06, 24. Mär. 2012 (CET)
Und hier ist plötzlich ein kleines Auto draus geworden. --Rôtkæppchen68 17:14, 24. Mär. 2012 (CET)
Weil noch nicht verlinkt hier: im Artikel NATO Tiger Meet steht ein bisschen zum Ursprung der Lackierung --Hareinhardt (Diskussion) 18:08, 24. Mär. 2012 (CET)
Na aber hoppla den Tigercopter einbauen (Galerie?) - und vielleicht noch ein Bild von einem glücklichen Steuerzahler, der das Boys-will-be-Boys-Treffen und die Bemalung bezahlt ..!?! GEEZERnil nisi bene 18:46, 24. Mär. 2012 (CET)
Hab den Heli mal eingebaut, neben der langen Tabelle war noch Platz. Und glückliche Steuerzahler hats ja ein paar im Hintergrund ;) --Hareinhardt (Diskussion) 16:15, 25. Mär. 2012 (CEST)
Ähm, Tschuldigung: Die Bildbeschreibung lautet Puma, 230 Sqn RAF, Waddington. Static display at RAF Waddington Open Day 2 July 2005, der Artikel behauptet aber 2005 Balikesir/Devrek Türkei - ist das nicht ein bisschen woanders? --88.217.24.167 23:43, 26. Mär. 2012 (CEST)
Das behauptet nicht nur der Artikel, auch deren Homepage: [27]. Das Treffen war Anfang Mai, das Bild ist vom Juli. Hab die Bildbeschreibung etwas abgeändert. --Hareinhardt (Diskussion) 02:10, 27. Mär. 2012 (CEST)

Gender-Mainstreaming

Politik und BRD-Presse propagieren ja mit großem Propagandaaufwand eine Angleichung der Lebensweisen von Frauen und Männern vor allem in der Arbeitswelt. Da stelle ich mir jetzt die Frage ob Frauen, die in einer derartigen Umgebung sozialisiert werden, tatsächlich auch in solchen Berufsgruppen arbeiten die gemeinhin als Männerberufe (Handwerker, Ingenieure, Techniker, Militär...) angesehen werden. Wurde das denn schon mal untersucht? So etwas müsste ja, würde man es ernst nehemn, das Ziel des Gender-Mainstreaming sein, rein subjektiv habe ich jetzt aber das Gefühl das weder solche Mädchen noch deren Eltern das wirklich wollen.--Antemister (Diskussion) 22:27, 24. Mär. 2012 (CET)

Genaue Zahlen habe ich auch nicht, aber so oft wie weibliche Soldaten von der Gorch Fock in den Tod plumpsen, scheint es ja mittlerweile massenhaft Frauen in der Marine zu geben. --176.198.24.224 23:00, 24. Mär. 2012 (CET)
Statistisches Bundesamt 2010. GEEZERnil nisi bene 10:39, 25. Mär. 2012 (CEST) Erstaunt uns nicht: Selbst Personen mit homologer Chromosomenausstattung können ja bereits ein sehr breites Präferenzspektrum aufweisen - wobei der Vatikan eine statistisch nicht relevante Ausnahme darstellt. GEEZERnil nisi bene 10:43, 25. Mär. 2012 (CEST)
Bericht aus dem Jahr 2009 --149.172.233.70 09:25, 25. Mär. 2012 (CEST) Und "gelungen" ist das Ganze, wenn eines Tage nicht mehr in Frauen-/Männerberufe unterteilt wird - das werde ich aber sicher nicht mehr erleben.
Solche Regelungen sind normalerweise nicht der Wunsch der Bevölkerung. siehe auch Fräulein (Wurde abgeschafft obwohl sich praktisch niemand am Wort gestört hat.) Laut Umfrage in Österreich lehnen 80% der Firmen eine Frauenquote ab [28] und die Frauenquote für Führungspersonal ist der Bevölkerung schlichtweg vollkommen egal [29] Warum deutlich mehr Männer in bestimmten handwerklichen Berufen oder als Soldat arbeiten, sagt einem schon Logik. Welches Mädchen träumt schon davon, mit zwei Eimer Zement in den Händen auf Baugerüsten rum zu klettern oder mit 25kg Gepäck auf dem Rücken, hinter einem Panzer her zu rennen. Vor allem da es massenweise andere Handwerkliche Berufe gibt, in denen Frauen schon seit jeher, genau so gerne wie Männer gesehen werden. Bei den Ingenieursberufen (u.ä) gibt es für Frauen immer ein Problem. Die meisten werden nun mal irgend wann schwanger. Das bedeutet häufig einen langfristigen Ausfall. Das ist in Berufen wo man wahlweise im Projekt oder in der technologischen Entwicklung permanent am Ball bleiben muss ein Problem. Wenn sämtliche Gendermainstreaming und Gleichstellungsbeauftragen in Europa ab morgen einen 10 jährigen Urlaub antreten würden, würde das niemand auffallen und Sie würden auch nicht vermisst werden. --85.181.213.139 10:24, 25. Mär. 2012 (CEST)
Versteht jemand die Frage? Was ist eine "derartige Umgebung" und welche "solche Mädchen" sind gemeint?
Oder meinst du schlicht Deutschland und wolltest fragen, ob überwiegend Männer in "Männerberufen" arbeiten?
--Eike (Diskussion) 11:54, 25. Mär. 2012 (CEST)
Ich nehme an, der Fragesteller meint die Regionen und Personen, die der mit großem Propagandaaufwand propagierten Propaganda unterliegen. --176.198.24.224 12:21, 25. Mär. 2012 (CEST)
IP 85.181.213.139, kannst du mir sagen, welcher Junge davon träumt, "mit zwei Eimer Zement in den Händen auf Baugerüsten rum zu klettern oder mit 25kg Gepäck auf dem Rücken, hinter einem Panzer her zu rennen"? Das ist keine Logik.
Welcher Junge und welches Mädchen träumt davon, seinen Arm bis zum Anschlag einer Kuh in den Hintern zu schieben? Und doch ist ausgerechnet der Tierarztberuf in Westdeutschland von einem männerdominierten Beruf vor wenigen Jahrzehnten (die "Blätter zur Berufskunde" schrieben noch Anfang der 1980er Jahre was von den hohen Anforderungen, die an die Ehefrau eines Tierarztes zu stellen seien!) zu einem inzwischen weit überwiegend durch weiblichen Nachwuchs geprägten Beruf geworden.
Anders gesagt: Freuden und Leiden des beruflichen Alltags spielen bei der geschlechterspezifischen Berufswahl keine entscheidende Rolle. --Snevern 12:27, 25. Mär. 2012 (CEST)
Nein sondern der Berufswunsch der Frauen. Also gibt es offensichtlich mehr Frauen die gerne Tierärztin werden wollen, als solche die Maurerin werden wollen. Und wie man sieht hat Sie nicht nur keiner daran gehindert sich Ihren Berufswunsch in einer "Männerdomäne" zu erfüllen, sondern Sie haben dafür auch keinen Gleichstellungsbeauftragten gebraucht. --85.180.184.78 21:47, 25. Mär. 2012 (CEST)
Mit Madame heute morgen diskutiert: Man sollte unterscheiden zwischen Berufen, die rein physisch enorme Ansprüche stellen und "akademisch zugänglichen" Berufen. In letzteren (Forschung und Wissenschaft) haben Frauen (man sehe sich Studentenzahlen an - bis zu 60 % Damen!) "aufgeholt", die werden aber nicht als "rein männliche Berufe" erachtet. GEEZERnil nisi bene 13:23, 25. Mär. 2012 (CEST)
Da solltet ihr aber schon noch zwischen sagen wir Germanistik und Informatik unterscheiden. In meinem Großraumbüro gibt's neuerdings eine (einzige) Frau... --Eike (Diskussion) 13:28, 25. Mär. 2012 (CEST)
Die Frage versteh' ich so, ohne zu werten: Wird hinterfragt/statistisch erhoben, ob Frauen, die ohne Geschlecht-Berufsgruppen-Klischees aufgewachsen sind, davon auch verstärkt und nennenswert Gebrauch machen ? Die Frage deshalh, weil in den Raum gestellt wird, dass trotz neutraler Erziehung letztlich wenig Bedarf besteht. --RobTorgel (Diskussion) 12:34, 25. Mär. 2012 (CEST)

Ich würde gerne haben, dass das Mainstreaming gegen Klischees kämpft, die Männer daran hindern, Kindergärtner zu werden und Frauen die Fähigkeit absprechen, Züge zu steuern. --RobTorgel (Diskussion) 12:47, 25. Mär. 2012 (CEST)

OK, etwas genauer, Robtroge hats erfasst. Der Staat propagiert dieses Gender-Mainstreaming, will (siehe Girls Day usw.) Frauen in typische Männerberufe bringen. Sofern dieses Gender-Mainstreaming Wirkung zeigt, dann müssten gerade doch die Frauen, die Gender-Mainstreaming selbst unterstützen (oder von den Eltern entsprechend beeinflusst wurden), häufiger in Männerberufen arbeiten als andere. Wenn es nicht so ist, dann bleibt Gender-Mainstreaming ja wirkungslos. Hat diese Fragestellung mal jemand untersucht?--Antemister (Diskussion) 13:12, 25. Mär. 2012 (CEST)

Wegen solcher Studien bin ich skeptisch, ein konkreter Berufswunsch wird ja von allem Möglichen und nicht nur vom Elternhaus beeinflusst. Das müsste aber auseinandergehalten werden um eine Aussage in deinem Sinne zu ermöglichen. Eine einfache Statistik, dass es heute mehr Automechanikerinnen gibt als früher, wird's wohl geben. --RobTorgel (Diskussion) 13:56, 25. Mär. 2012 (CEST)
+1. Ich kann ja nur von meine Erfahrungen her sprechen, aber dieser ganze Männer-Frauenberufe-Kram halte ich für ziemlichen Quatsch. Ich war auch mal bei so einem komischen "Girls Day" dabei - und ganz ehrlich, hätte ich vorher gewusst, wie es da abgeht, dann wäre ich da nicht gewesen. Nur irgendwelche Tussis, die ohne Hilfe wahrscheinlich nichtmal den Einschaltknopf am Rechner gefunden hätten. Das war genauso gruselig, als wir in der 7. Klasse in ITG nach Mädchen und Jungen getrennt wurden. War für mich auch die Hölle, wobei ich mich sowieso frage, was Serienbriefschreiben mit Informationstechnischer Grundbildung zu tun hat ;). Ich halte diese ganzen Aktionen also für reinen Blödsinn und wäre so oder so Programmierer geworden. (Und ja, ich sage ganz bewusst Programmierer und nicht Programmiererin;) --declaya red mit mir! 09:54, 26. Mär. 2012 (CEST)
Ich kann nicht beurteilen, wie das gemacht wird, aber ich halte es für sinnvoll, dass da was gemacht wird. Ich bin Programmierer (ohne -in), und wie gesagt: Wir haben hier unter einigen Dutzend Kollegen neuerdings eine einzige Frau. Du bist in der Minderheit. --Eike (Diskussion) 10:14, 26. Mär. 2012 (CEST)

Hmmm, ich hätte iegentlich wissen müssen das die WP hier keine Top-Quelle ist ;-) na ja... Scheint also doch so zu sein das dass ganze doch eher lustig ist, etwa wenn ein Chemie-Lehrstuhl am Görls-Day eine Vorlesung über Kosmetik und Parfüm hält.- Obwohl die Eingangsfrage gar nicht beantwortet wurde, sind wohl die Produkte unserer Gender-Beauftragten so kümmerlich das sie gar nicht auffallen...--Antemister (Diskussion) 23:10, 26. Mär. 2012 (CEST)

Aufwendigster Kriminalfall

Welcher Kriminalfall hatte den größten Personal- und Zeitansatz

  • in Deutschland
  • weltweit?

(egal, ob aufgeklärt oder nicht)? --Apokalypse (Diskussion) 13:07, 25. Mär. 2012 (CEST)

DE: Die Schleyer-Entführung? Int: 11. September 2001? --95.112.138.189 13:10, 25. Mär. 2012 (CEST)
Die Ermordung des österreichischen Thronfolgers. -- Wiprecht (Diskussion) 14:16, 25. Mär. 2012 (CEST)
Der Reichstagsbrand. --Rôtkæppchen68 15:12, 25. Mär. 2012 (CEST)

In der DDR war das wohl der Kreuzworträtselmord. Vor allem wegen der Auswertung von mehr als einer halben Million Schriftproben per Hand. Keine Ahnung, warum die anderen Kriminalfälle besonders aufwendig gewesen sein sollen. Bei Sarajevo und dem Reichstagsbrand wurden die Täter ja sehr schnell ermittelt. Und die anschließende Verfolgung durch die Nazis willst doch hoffentlich nicht als kriminalistische Aufklärung bezeichnen.Oliver S.Y. (Diskussion) 16:07, 25. Mär. 2012 (CEST)

<quetsch> Die Frage galt auch dem Personaleinsatz. Serbien wurde beschuldigt Drahtzieher gewesen zu sein und dies führte letztendlich zum WKI in den Millionen von Menschen verschiedenster Nationen involviert waren. Ursache/Anlass war das Attentat. -- Wiprecht (Diskussion) 20:32, 25. Mär. 2012 (CEST)
Die Auflösung der Reichstagsbrandfrage lässt zu wünschen übrig; die heute favorisierte Version ist wohl die plausibelste, aber keinesfalls nachweisbar richtig. Dies ganz unabhängig von dem politischen Vorteil, den die Nationalsozialisten daraus zogen.
Der Kreuzworträtselfall ist sicherlich auch weltweit unter den alleraufwendigsten.
Kaspar Hauser kann man auch noch nennen; an dem Fall wird seit gut 178 Jahren geforscht - die Aufklärung seines Todes tritt immer ein bisschen zurück vor der Frage, wer er denn wirklich gewesen sei; sie könnte aber mit letzterer zusammenhängen. Das Problem ist, dass die meisten Gegner der Prinzentheorien schlicht abstreiten, dass er ermordet worden sei (weil die Ermordung besser in die Prinzentheorien passt, wenn sie auch diese nicht zwingend voraussetzt) und ein sehr an den Haaren herbeigezogenes Szenario einer Selbstverletzung mit Todesfolge favorisieren. Übrigens ein interessantes Beispiel, wie mit Wikipedia Tatsachen geschaffen werden - der Artikel ist ja fest in der Hand eines bestimmten Ansatzes und hat - auch mit Wirkung nach außen - alle anderen Erklärungsversuche äußerst rabiat niedergemacht (die Diskussionsarchive haben etwa die Länge eines Fernsehturmes, aber einen böseren Stil...).
Es gibt auch noch einen Justizmord, über den seit bald zweitausend Jahren mit eindeutigen politischen Absichten diskutiert wird - haben die (damals dort anwesenden, gerne aber verallgemeinerten) Juden den an sich gutmütigen Besatzungschef Pontius Pilatus dazu aufgehetzt, einen an sich eher harmlosen Wanderrabbi zum Tode zu verurteilen - oder hatte der Besatzungschef selbst Angst vor politischen Wirkungen der Predigten dieses Rabbis, ihn daher verurteilt und das Ganze dann als von den Juden veranlasst hingestellt?
Kirschschorle (Diskussion) 17:19, 25. Mär. 2012 (CEST)
Mein Tip: Der Schmücker-Prozess von 1976 bis 1991. --Φ (Diskussion) 20:37, 25. Mär. 2012 (CEST)
Ist den überhaupt der Personal- und Zeitaufwand bei der Aufklärung (und evt. sonstigen Folgen) gemeint, oder der zur Planung/Durchführung? In letzterem Fall könnte neben diversen Terroranschlägen (siehe oben 11. September) sowas wie der Bankraub in Fortaleza (150 Mann, drei Monate Tunnelbau) eine erste Messlatte legen. --YMS (Diskussion) 21:18, 25. Mär. 2012 (CEST)
Ich meine, es hätte mal eine Kaufhausbetrüger gegeben, der Onkel Dagobert oder so hieß. Die Fahndung nach dem soll auch ziemlich lang gedauert haben und wird auch eine Menge Personal beschäftigt haben. Der Mord an Peggy (9 Jahre alt, nicht alzulange her, irgendwo in Deutschland) hat auch unglaubliche Ressourcen verbraucht (bis zu RECCE-Tornados!). Grüße--SkyhawkFour Disk BW 21:29, 25. Mär. 2012 (CEST)
Im wahren Leben heißt er Arno Funke. --Eike (Diskussion) 21:43, 25. Mär. 2012 (CEST)
Die Frage bezieht sich nicht auf die Akteure sondern auf die Strafverfolgungsbehörden. Allerdings ist die andere Seite natürlich auch interessant... --Apokalypse (Diskussion) 22:26, 25. Mär. 2012 (CEST)
Die Fahndung nach dem soll auch ziemlich lang gedauert haben und wird auch eine Menge Personal beschäftigt haben. --Eike (Diskussion) 23:01, 25. Mär. 2012 (CEST) Nachwurf: Da die Zeiten der Anrufe bekannt waren, ließ die Berliner Polizei einmal 1.100 Telefone vergeblich überwachen, ein anderes Mal 3.900 Apparate. Der Personaleinsatz mit mehreren tausend Polizeibeamten war enorm. Zufälligerweise benutzte Funke in einem Fall ein nicht überwachtes Telefon. Grüße--SkyhawkFour Disk BW 23:07, 25. Mär. 2012 (CEST)
Beim Googlen hab ich das hier gefunden, vielleicht hilft dir das weiter. Grüße--SkyhawkFour Disk BW 23:05, 25. Mär. 2012 (CEST)
Die Ermittlungen zu den Finanzierungen, Telefonrechnungen, Audi-Fahrzeugen und Urlauben des Christian Wulff hatten mit Sicherheit den größten Personal- und Zeitansatz in der Geschichte Deutschlands. Über 80 Mio Ermittler arbeiteten wochenlang in jeder freien Minute in der Soko Stammtisch an der Aufklärung der Taten. --91.56.154.62 00:40, 26. Mär. 2012 (CEST)
Es gibt eine korrekte Antwort auf die Frage. Aber die ist gar nicht lustig. Dasjenige Verbrechen der Menschheitsgeschichte, das die größte Untersuchungs- und Aufklärungsarbeit sowie auch die insgesamt größte (wenn auch nur einzelne Tatbereiche und einzelne Täter abdeckende) juristische Aufarbeitung nach sich zog und zieht (und auch als Verbrechen an sich zu den größten der Geschichte gehört) nennt man gemeinhin Shoa, früher auch Holokaust (und andere Bezeichnungen). Manchmal findet man es unter "Kriegsverbrechen", aber das ändert nichts an der Sache - ein Kriegsverbrechen war es nur insofern, als der Krieg und die Umstände des Krieges die Durchführung des Verbrechens - des Mordes an etlichen Millionen Menschen, dazu etliche Nebenverbrechen in Tateinheit - begünstigten. Die Absicht, das Verbrechen zu begehen, hatte aber mit dem Krieg nicht viel zu tun.
Um einen Eindruck der Größenordnung der Aufklärungsarbeit zu bekommen, sei daran erinnert, dass man seit vielen Jahrzehnten auch nur die Namen aller Ermordeten zusammenzubekommen versucht.
Eine Schätzung des Prozentsatzes der Täter, die man zumindest angeklagt hat, kenne ich nicht. Er dürfte aber klein sein.
Um die Frage wieder lustiger zu gestalten, sollte man vielleicht Massenmorde / Völkermorde draußen halten...
Kirschschorle (Diskussion) 02:59, 26. Mär. 2012 (CEST)
Kriegsverbrechen sind Verstöße gegen das Völkerrecht, Kriminalfälle gegen das Strafrecht. --81.200.198.20 13:38, 26. Mär. 2012 (CEST)
Wie schon angedeutet, kommt mir die Bezeichnung "Kriegsverbrechen" für die Shoa immer seltsam verharmlosend vor. Nicht, dass man sich unter "Kriegsverbrechen" irgendwas Nettes vorstellte. Aber irgendwie denkt man dabei an Gewalthandlungen, die aus dem Kontext des Krieges entstehen, die (vielleicht auch im Eifer des Gefechts) die als nötig empfundenen kriegerischen Handlungen vielleicht in Richtung unnötige Grausamkeit oder in Richtung unbeteiligte Zivilisten übertreiben, et c. All das scheint mir auf die Shoa nicht wirklich zuzutreffen.
Aber ich bin kein Experte. Kirschschorle (Diskussion) 23:11, 26. Mär. 2012 (CEST)

AAA-Batterien und -Akkus

Die haben bekanntlich unterschiedliche Spannungen (1,5 V vs. 1,2 V). Warum das so ist, konnte ich rausfinden. Aber kann es in einer von beiden Richtungen schädlich sein, Akkus statt Batterien oder umgekehrt zu verwenden? 79.203.199.71 19:44, 24. Mär. 2012 (CET)

  • Wenn Du einen Akku in einem Gerät für Batterien verwendest, macht es lediglich schneller schlapp (weil die Spannung zu schnell zu niedrig wird). Im Extremfall geht es erst gar nicht an. Wobei meine Erfahrung sagt die meisten neueren Geräte sind schon im Hinblick auf Akkus ausgelegt. Passieren tut hier nix.
  • Andersrum? Ich glaube kaum, dass ein Gerät dass AA/AAA schluckt nicht so ausgelegt ist, dass es sich auch aus Batterien versorgen kann kann. Hinweise dazu gibt vermutlich die Anleitung. Ausnahme sind hier Geräte die die Akkus auch laden, dort nie andere als den vorgesehenen Akkutyp verwenden! Wenn Du da statt Akkus nun Batterien reinpackst und das Gerät versucht (trotz der schon zu hohen Spannung) diese zu laden, kann die Batterie zerstört werden, und dadurch mit hoher Wahrscheinlichkeit auch das Gerät. Geräte mit vernünftiger Ladeelektronik sollten das zwar erkennen, in vielen (teilweise einfach zu billigen) Geräten die auf Akkus ausgelegt sind gibt es jedoch keinerlei Überwachung. .. Windharp (Diskussion) 19:52, 24. Mär. 2012 (CET)
Zum zweiten Punkt angemerkt: es schadet nicht, Batterien zu benutzen (auch nicht andere Akkutypen), nur sollte man Batterien dort nicht laden. Insofern zu deiner Frage: du kannst in der Regel auch alles mit Batterien oder Akkus verwenden. Nur halt Akkus geben etwas weniger Leistung ab, sind dafür jedoch etwas konstanter. -- Quedel Disk 19:48, 26. Mär. 2012 (CEST)
Es kann auch noch andere Effekte geben, wenn man Batterien statt Akkus verwendet: z.B. leuchten Taschenlampen, die auf 1,5V ausgelegt sind und Glühlampen verwenden generell mit 1,2V Akkus viel schwächer als mit 1,5V (eine Formel dafür findet man unter en:Incandescent_light_bulb#Light_output_and_lifetime, laut dem ist eine Glühlampe mit 1,5V statt 1,2V mehr als doppelt so hell). Für Geräte, die 1,5V brauchen um richtig zu funktionieren oder mit 1,2V schnell schlapp machen gibt es aber mittlerweile auch RAM-Zellen (wiederaufladbare Alkalines) mit 1,5V. Auch normale Alkalines lssen sich mit RAM-Ladegeräten wiederaufladen, aber nicht so oft (max. 10 mal, RAM nach Herstellerangaben bis zu 250 mal). --MrBurns (Diskussion) 17:57, 27. Mär. 2012 (CEST)

Komponist gesucht

Von wem entstammte das Infanterielied "Gäb es nur eine Krone"? --Kompanievorstand (Diskussion) 23:27, 25. Mär. 2012 (CEST)

Vielleicht mal in einem Bundeswehrforum anfragen. --87.122.112.30 11:07, 26. Mär. 2012 (CEST)
Zuständig wäre GEMA bzw. AKM ("Titelauskunft") --RobTorgel (Diskussion) 11:22, 26. Mär. 2012 (CEST)
Bei solchen Fragen hilft manchmal ISWC-Net. Diesmal leider nicht. --Komponistenarchiv (Diskussion) 19:07, 27. Mär. 2012 (CEST)

alte rtf Datei öffnen

Wir haben hier eine alte rtf Datei die mit einem Word Pad von ca 2000 geschrieben wurde. Nur leider bekommen wir weder mit Open Office mit dem aktuelle Word Pad noch mit Word 2010 das alte Layout, sondern nur ein völlig Chaotisches Layout. Wie bekommen wir das originale Layout wieder? thx--Sanandros (Diskussion) 16:24, 25. Mär. 2012 (CEST)

Die Formatierung hängt ggf. auch von der Druckereinstellung (Seitenränder/bedruckbarer Bereich) und davon ab, ob ihr unter Ansicht/Zeilenumbruch "Kein Umbruch", "Am Fenster umbrechen" oder "Am Lineal umbrechen" gewählt habt. -- 188.99.203.199 16:43, 25. Mär. 2012 (CEST)
(BK) Such Dir jemand, der noch ein Windows XP hat, oder ein Windows 7 mit XP Mode, und wandel die Datei dort ins wri- oder doc-Format. --Rôtkæppchen68 16:45, 25. Mär. 2012 (CEST)
@IP:Es handelt sich nicht um die Druckereinstellung sonder ist schon beim öffnene der Datei nicht mehr richtig.
@Rot:Warum XP? 2000 war Win Milenium und Win 2000 auf den Rechnern. Ausser dass ich gerade niemand kenne der XP hätte (aber müsste auch mal fragen)--Sanandros (Diskussion) 20:19, 25. Mär. 2012 (CEST)
Die Druckereinstellung beeinflusst eben auch die Darstellung am Bildschirm, sonst hätte ich das nicht geschrieben. Das ist ein altes Kriegsleiden der Microsoft-Textverarbeitungen. -- 188.99.203.199 20:57, 25. Mär. 2012 (CEST)
Weil viele Leute noch XP haben, aber fast keiner mehr Windows 2000 oder gar ME? Weil Windows 2000 oder gar ME gar nicht mehr unterstützt werden, z. B. mit Sicherheitsupdates? Weil Windows 7 nun mal einen XP- und keinen ME-Modus hat? --Eike (Diskussion) 21:23, 25. Mär. 2012 (CEST)
Genau. Windows 7 Professional mit XP Mode wird es noch ein paar Jährchen geben. Windows 98, ME oder 2K würde ich höchstens noch als Testinstallation auf eine SD-Karte installieren. --Rôtkæppchen68 22:35, 25. Mär. 2012 (CEST)
Ich hätte hier noch einen Win 98 Rechner, sowie Win XP und gar Office 2000. Ich könnte es mal für dich versuchen? Win 98-Rechner braucht man für die Dos- und Windows-Spiele, die Dosbox und Win XP nicht können. Man muss damit ja nicht ans Netz gehen, auch wenn Win98 in der Praxis inzwischen wohl das sicherste Win-System ist, da mit gängiger Schadsoftware nicht kompatibel und nicht Ziel von Angriffen. (Wofür man higegen Win7 braucht, wüsste ich nicht, aber das war ja nicht Thema) --Robin Goblin (Diskussion) 15:58, 26. Mär. 2012 (CEST)
Thx Robin ich werde mich melden und Thx an alle ich versuche es mal mit XP versuchen da ich doch noch jemand mit XP gefunden habe.--Sanandros (Diskussion) 08:09, 29. Mär. 2012 (CEST)
Archivierung dieses Abschnittes wurde gewünscht von: тнояsтеn 08:25, 29. Mär. 2012 (CEST)

Chinesische Kaiser

Im Artikel der chinesischen Kaiser Jingtai und Tianqi befinden sich verschiedene Ausschnitte des selben Bildes (und das nicht nur in der deutschen Wikipedia). Wer ist auf diesem Bild wirklich dargestellt? --Hareinhardt (Diskussion) 13:59, 25. Mär. 2012 (CEST)

Hmm, ich glaube beide Bilder zeigen Kaiser Jingtai, der von 1449 bis 1457 regiert hat. Das erste Bild (links, tschechischer Uploader) hat auch eine Quelle, die genau das sagt, das zweite Bild (rechts) hat keine Quelle und der russische Uploader dürfte da etwas verwechselt haben. Das erste Bild ist auch im chinesischen Wikipedia-Artikel von Jingtai eingebunden, siehe zh:明代宗, während im chinesischen Artikel zum viel späteren Kaiser Tianqi (1620-1627) ein ganz anderes Bild ist, nämlich dieses (Datei:Jianqi Emperor.jpeg), siehe zh:明熹宗. --El bes (Diskussion) 14:36, 25. Mär. 2012 (CEST)
3 x Chapeau für Hareinhardt, dem es aufgefallen ist ! GEEZERnil nisi bene 16:05, 25. Mär. 2012 (CEST)
Das erinnert mich an eine ähnliche Konstellation, die ich vor einiger Zeit erfolglos unter Portal Diskussion:Biografien/Archiv#Bebilderung Polemon von Laodikeia vs. Polemon von Ilion zur Diskussion gestellt hatte: Wer sich im Stande sieht, aufzulösen, ob die in beiden Artikeln abgebildete Büste Polemon von Laodikeia oder Polemon von Ilion darstellt, gewinnt zwei frei verwendbare Karmapunkte. --YMS (Diskussion) 16:34, 25. Mär. 2012 (CEST)
Der korrekte Ansatz - in beiden Fällen - wäre die Entfernung des Bildes (z.B. Parking in der Disk.), bis die Frage eindeutig geklärt ist. Etwas "auf Verdacht" reinsetzen, bringt ganz schlechtes Karma. GEEZERnil nisi bene 17:07, 25. Mär. 2012 (CEST)
Denk ich auch, die sollten raus. Und es kommt noch schlimmer: ich hab noch dieses Bild gefunden
 
Zur Zeit nicht in de:Wikipedia verwendet
, das zeigt in den Einbindungen den Lebensdaten nach einen dritten (siehe die internationale Einbindung, zur Zeit ohne deutschem Artikel). Und wieder keine richtige Quelle angegeben. Ein Chaos.....--Hareinhardt (Diskussion) 17:22, 25. Mär. 2012 (CEST)
Die chin:WP verwendet für Jingtai dasselbe Bild wie wir, für Tianqi zeigen sie aber ein anderes Bild. GEEZERnil nisi bene 19:57, 25. Mär. 2012 (CEST)
Auf die Chinesen ist aber auch kein Verlass. In Jingtai sind zwei verschiedene Bilder. Das alternative Datei:Jingtai.jpg wird in der Liste der Ming-Herrscher für Jingtai verwendet, während in selbiger Liste fragliches Bild einmal bei Tianqi und einmal bei Ming Rui Zong (der ohne deutschem Artikel, lebte von 1476 -1519) eingeordnet ist. Tianqi ist es wahrscheinlich nicht, wenn man die Anzahl der Fundstellen zählt. Ich glaub, wir brauchen was gedrucktes... --Hareinhardt (Diskussion) 01:06, 26. Mär. 2012 (CEST)
Völlig klar. Mal bei der BiBa anfragen ?
  • Das hier einsehen: Das Kommunikationsamt <T'ung-cheng shih ssu> der Ming-Dynastie <1368 - 1644>, Verfasser: Paul, Carmen, Wiesbaden : Harrassowitz, 1996, 272 S. Freiburger fernöstliche Forschungen - und
  • das hier: National Geographic History: Das chinesische Kaiserreich: Die Herrscher auf dem Drachenthron (Originaltitel: Dragon Throne); Gebundene Ausgabe: 256 Seiten, Verlag: NATIONAL GEOGRAPHIC Deutschland; Auflage: 1., Aufl. (1. März 2010), ISBN-10: 3866901631, ISBN-13: 978-3866901636 GEEZERnil nisi bene 08:32, 26. Mär. 2012 (CEST) Warum assoziiere ich beim Bild des unbekannten Herrn Kaisers immer Yellow Submarine ...?
Der Link auf den Stabi-Katalog geht nicht. --тнояsтеn 08:46, 26. Mär. 2012 (CEST)
Soooo, ich habs, danke allen Helfern. Das Bild hängt im Palastmuseum Peking, siehe hier. Der gute Mann ist der Ming-Prinz Zhu Youyuan (朱祐杬; 1522-1556) in Zeremonialuniform, von einem anonymen Ming-Künstler, entstanden von etwa 1521-1524. Auf gehts ans ausbessern....--Hareinhardt (Diskussion) 00:59, 27. Mär. 2012 (CEST)
Es war KEINER von den beiden Initialbebilderten ?!? Grossartig !! Schlage Grosses Ostereier-Suche vor und mit den beiden Polemonen (oben) weiter zu machen. Spricht hier jemand Griechisch und kann auf griech. Webseiten nachlesen oder herausfinden, wo diese Büste steht? GEEZERnil nisi bene 16:46, 29. Mär. 2012 (CEST)


Die Seite ist zur Zeit eine Falschschreibungsweiterleitung. Ist das so richtig? Bevor ich die ganzen Links auf die Seite umbiege, würde ich dies gerne absichern. Viele Grüße --Christian1985 (Diskussion) 18:39, 25. Mär. 2012 (CEST)

Französisch jedenfalls mit zwei n: http://www.cnrtl.fr/definition/salonnière. Im Deutschen gibt es wohl häufiger die Schreibweise mit einem n. Belege bei: http://dict.leo.org/forum/viewWrongentry.php?idThread=1086321&idForum=17&lp=frde&lang=de oder: http://www.zeit.de/2007/07/KA-Mittelstueck. Angesichts der dünnen Beleglage muss das nichts heißen, es kann sich auch um Falschschreibungen handeln. Andererseits hat Google bei „Seiten auf Deutsch“ 62700 Fundstellen für „Salonière“ und nur 5070 für „Salonnière“, was zwar auf den ersten Blick recht eindeutig erscheint, aber unberücksichtigt lässt, dass Google unter „Salonière“ auch die ebenfalls vorkommende Schreibung ohne Akzent („Saloniere“) aufführt (die ich, nebenbei bemerkt, ganz schrecklich finde – „Salo-Niere“?). Hier und hier erscheint die Schreibung mit „n“ sogar in Buchtiteln. Gegen eine 1:1-Übernahme aus dem Französischen würde auch die unterschiedliche Aussprache sprechen (siehe hier und hier). Im Großen Duden Fremdwörterbuch kommt das Wort nicht vor, auch nicht bei Duden Online oder im DWDS. Insofern fehlt es wohl an einem anerkannten Standard für das Deutsche. In der WP war die Schreibweise mit einem n jahrelang akzeptiert, die Änderung zur Falschschreibungs-Weiterleitung war eine Aktion des erst seit einer Woche aktiven Benutzers Kulturkampfforschung. Der derzeit feststellbare tatsächliche Sprachgebrauch im Deutschen scheint mir eher in Richtung „Salonière“ zu gehen, von daher würde ich dafür plädieren, das rückgängig zu machen. Ansonsten könnte man vielleicht mal eine Mailanfrage an die Duden-Redaktion schicken. --Jossi (Diskussion) 19:10, 25. Mär. 2012 (CEST)
Wobei es doch recht merkwürdig wäre, einem Wort, das am Akzent klar als französisch erkennbar ist, eine spezielle deutschsprachige Rechtschreibung zuzuweisen. Anders wäre es bei einer Eindeutschung, aber dann würde man wohl auf den Akzent verzichten. Ein ähnliches Beispiel ist übrigens Saisonier/Saisonnier. --= (Diskussion) 20:25, 25. Mär. 2012 (CEST) Sic --Kulturkampfforschung (Diskussion) 15:43, 27. Mär. 2012 (CEST)
Entsprechend gibt es auch noch die Falschschreibungsseite Saloniere. --Christian1985 (Diskussion) 20:29, 25. Mär. 2012 (CEST)
Natürlich ist das merkwürdig, aber Sprache ist nun einmal oft merkwürdig. Der Normalfall ist, dass bei der Eindeutschung der Akzent wegfällt, aber es gibt auch Beispiele dafür, dass französische Fremdwörter unter Beibehaltung des Akzents in der Schreibung verändert wurden, z.B. Dekolleté oder Kommuniqué. Das Problem scheint mir hier hauptsächlich darin zu liegen, dass das Wort im Deutschen so extrem selten ist, dass sich (noch) keine verbindliche Schreibung herausbilden konnte. Im ähnlichen Fall der chiffonnière hat der Duden sich für „Chiffonniere“ entschieden, obwohl es durchaus auch „Chiffoniere“ gab und gibt. Wie auch das Beispiel Saisonier/Saisonnier erkennen lässt, gibt es im Deutschen in solchen Fällen durchaus eine Tendenz zum Wegfall des zweiten n. Ein anderes Beispiel ist der Pontonpionier, der im Französischen pontonnier, im Deutschen „Pontonier“ heißt. --Jossi (Diskussion) 23:24, 25. Mär. 2012 (CEST)
Noch ein weiterer Gedanke: Das Stichwort war doch ohnehin nie mehr als ein Redirect auf Literarischer Salon. Was bringt uns denn eine Falschschreibungsweiterleitung auf einen Begriff, den wir überhaupt nicht als Lemma haben? (Außer vielleicht dem Versuch, eine Version als richtig vorzuschreiben – was aber nicht Aufgabe der Wikipedia sein kann.) Wenn wir sowohl „Salonnière“ als auch „Salonière“ (und meinetwegen in Gottes Namen auch noch „Saloniere“) einfach als normale Redirects auf „Literarischer Salon“ stehen lassen, brauchen wir diese momentan offenbar nicht lösbare Frage gar nicht zu lösen, sondern können sie elegant umschiffen. --Jossi (Diskussion) 00:08, 26. Mär. 2012 (CEST)
Ich halte deinen Gedankengang für sehr vernünftigt. Stellt man allerdings die Weiterleitung auf Salonière wieder her, dann muss man aus formalen Gründen auch aus Saloniere eine Weiterleitung machen. Die Wikipedia-Regularie Wikipedia:Weiterleitung#Sonderzeichen verstehe ich so, dass eben auch für das Sonderzeichen è eine Weiterleitung ohne dieses Sonderzeichen angelegt werden soll. Oder missinterpretiere ich dort etwas? Ansonsten wären ja auch solche Weiterleitungen wie Poincare-Lemma oder Henri Poincare nicht zulässig.
Ich habe mal Kulturkampfforschung auf seiner Diskussionsseite angesprochen. Bevor wir hier nun wieder etwas ändern, sollten wir noch abwarten, wie er die Sache sieht. --Christian1985 (Diskussion) 00:33, 26. Mär. 2012 (CEST)

@Jossi, danke, das war gute Vorarbeit. Ich habe mich mit dem Thema hier irgendwo schon einmal auseinandergesetzt und bin bereit, es in Kürze noch ausgiebiger zu tun, da mich der Begriff "Salonnière" in der deutschen WP schon lange stutzig macht. Sämtliche oben genannte Schreibweisen (Salonnière, Salonière, Saloniere) sind in deutschen Publikationen jüngeren Datums zu finden. Auf Anhieb zur Orthographie: das doppelte n wird in der frz. Rechtschreibung nur gesetzt, weil andernfalls ein Nasal ausgesprochen werden müsste, was vor einem Vokal unmöglich/unaussprechbar ist. In der deutschen Rechtschreibung fällt in der Regel das zweite n (und sofern vorhanden auch der Akzent) fort. Siehe fr:Pionnierde:Pionier und wie schon von Dir angegeben fr:Saisonnierde:Saisonier, fr:Chiffonnièrede:Chiffoniere (hier überrascht mich der Duden), beispielsweise auch fr:Impressionnismede:Impressionismus, fr:Cloisonnismede:Cloisonismus. Danach sollte die deutsche Schreibweise de:Saloniere (ein n, ohne Akzent) sein, die in deutschen Büchern ebenfalls weit verbreitet ist. Dahingegen ist de:Salonière ein deutsch-französischer Zwitter und IMHO eine Falschschreibung, was aber zu belegen wäre. --Désirée2 (Diskussion) 17:21, 26. Mär. 2012 (CEST)

Heilt DekolleTee die SaloNiere?
Ich wurde gewarnt, dass es hier Diskussionen geben könne, wenn ich mit den Korrekturen weitermache. So habe ich nach Setzung des Falschreibungshinweises nicht systematisch, sondern nur beiläufig korrigiert. Aber mir scheint die Lage hier ziemlich eindeutig: Wenn es keine Eindeutschung gibt, ist die fremdsprachige Originalorthografie zu übernehmen. Die amtliche Regelung der Rechtschreibung (2006) hält in den Erörterungen zu "Regelteil und Wörterverzeichnis" fest: "Hingegen kann die Schreibung vieler Fremdwörter nur durch Einzelfestlegungen im Wörterverzeichnis erfasst werden; es gibt dazu weder Regeln noch ist es sinnvoll, lange Ausnahmelisten im Regelteil anzulegen." Ich wäre bei Neologismen viel liberaler als bei nicht-kreativen Heterografien. Eindeutschung und Orthografiereform sind aber nicht die Aufgabe der WP. Falschschreibungen in Redirects zu promoten ist gerade das Gegenteil einer eleganten Lösung (Jossi), nämlich ein Kurs in Falschschreibung: denn wenn der falsch geschriebene Begriff unter der Sucheingabe angewählt werden kann und dann eine Weiterleitung glückt, die womöglich gar nicht als solche bemerkt wurde, gibt das der Falschschreibung den Anschein von Korrektheit. Im Übrigen (Christian1985) ist eine Weiterleitung von Poincare zu Poincaré meines Erachtens schlicht überflüssig: nach der Sucheingabe von Poincaré oder Poincare oder sogar auch Poincarè erscheinen dieselben LemmaNamen zum Anwählen. Man tippt etwas Falsches, wählt dann aber etwas richtiges - oder einen Falschschreibungshinweis (Hinweis für die Software: da wäre es schön, wenn diese und BKLs, wenn sie als Belegstellen unter der Sucheingabe auftauchen, entsprechend markiert würden, zum Beispiel durch einem vorangehenden fetten Punkt/BKL oder ein ≠/FS). Duden Universalwörterbuch gibt neben Dekolletee auch das unproblematische Décolleté ("besonders schweizerisch") sowie Dekolleté an. Nicht aber Renommé, Renommée oder Erdbéren. Desiderabel also bleibt das angekündigte Ausgiebigere in Kürze. Über die französischen Akzente im Dèutschen. Und zum wichtigen Hinweis von Jossi, dass das lautliche Pendant zu der Unschönschreibung "Saloniere" längst eingedeutscht ist, der Duden also sprüchig werden müsste.
Im Übrigen ist Salonnière ein Lemma wert, mit besonderer Berücksichtigung von Gender-Gesichtspunkten, der Bildungsökonomie, des Wissensnetzwerkens und der Statusproduktion. --Kulturkampfforschung (Diskussion) 15:56, 27. Mär. 2012 (CEST)

Ja die Weiterleitungen zu Poincaré halte ich eigentlich auch für Überflüssig, aber irgendwer sah das wohl anders. Bei den persönlichen Einstellungen unter Helferlein ist es möglich einzustellen, dass Links auf BKLs und Falschschreibungsseiten optisch hervorgehoben werden. --Christian1985 (Diskussion) 16:49, 27. Mär. 2012 (CEST)
Es ist nicht Aufgabe von Wikipedia, auf eigene Faust Schreibweisen als „richtig“ oder „falsch“ zu klassifizieren. Wenn, wie Désirée2 zutreffend feststellte, derzeit alle drei Schreibweisen nebeneinander verwendet werden und wenn, wie dein Zitat aus den Amtlichen Regeln belegt, dazu keine Regelung existiert, müssen wir (im Deutschen) alle drei Schreibweisen so lange gelten lassen, bis es eine Regelung geben wird. Es kommt uns nicht zu, aus eigener Machtvollkommenheit einzelne davon als „Falschschreibungen“ zu deklarieren (und wenn das eigene Sprachgefühl noch so sehr aufjault). Eine Regel „Wenn es keine Eindeutschung gibt, ist die fremdsprachige Originalorthografie zu übernehmen“ kann hier nicht in Betracht kommen, da es ja eingedeutschte Formen gibt (nur eben mehrere), die neben der Originalschreibweise (und zwar deutlich häufiger als diese) in Gebrauch sind. --Jossi (Diskussion) 22:17, 27. Mär. 2012 (CEST)
Das Substantiv Salonnière (alternativ: Salonière, und doch seltener Saloniere) glänzt definitiv durch Abwesenheit in deutschen Wörterbüchern (was die unsichere/uneinheitliche Rechtschreibung erklärt) und hat in französischen eine ganz andere Bedeutung. Dort ist salonnière allerdings als Adjektiv - des Öfteren mit negativer Konnotation - vorhanden. Fazit: ein Neologismus (+ 1 Jossi: keine etablierte Rechtschreibregelung, folglich vorläufig auch keine Falschschreibung, was + 2 Jossi, da es sich um eine Weiterleitung handelt, mir nicht so besonders dramatisch zu sein scheint). Ich bin dem genaueren Ursprung (und entsprechenden Quellen) noch beharrlich auf der Fährte und bitte um Geduld. Es wäre nett, wenn dieser Absatz vorläufig nicht abgespeichert würde. Danke. --Désirée2 (Diskussion) 15:59, 29. Mär. 2012 (CEST)
Da die Auskunftsseite einen eher ephemeren Charakter hat, habe ich diesen Abschnitt nach Diskussion:Salonière kopiert, wohin er inhaltlich auch gehört, und schlage vor, die Diskussion dort fortzusetzen. --Jossi (Diskussion) 18:54, 29. Mär. 2012 (CEST)

Schaltung

Gibt es Menschen, die lieber mit Gangschaltung fahren als mit Automatik? Wenn, ja: Wieso? Hat man bei manueller Schaltung irgendwelche Vorteile, die sich mir nicht erschließen? --85.180.138.204 19:07, 24. Mär. 2012 (CET)

Ja, gibt es. Um eine überflüssige Was-ist-besser-Debatte zu vermeiden: Man hat ein anderes Fahrgefühl. Gruß --79.213.112.245 19:11, 24. Mär. 2012 (CET)
Mich z.B., weil ich gerne selbst fahre und nicht fahren lasse, ansonsten kannst du ja mal Google mit der Thematik füttern!. Ich weiß nicht in wie weit das Stimmt aber Automatik schluckt glaub ich mehr. -- Viele Grüße von Jogo.obb Disk 19:18, 24. Mär. 2012 (CET)
Manche Menschen finden Automatik langweilig. --93.132.170.170 19:19, 24. Mär. 2012 (CET)
ein Automatikgetreibe wiegt auch mehr. Heisst ein handgeschalteter wiegt weniger, und hat ind er Regel dadurch mehr Zuladung. Gerade bei Handwerken durchaus ein Argument für ein handgeschalteten.--Bobo11 (Diskussion) 19:33, 24. Mär. 2012 (CET)
Schaltgetriebe fahren macht Spaß. Aber steh' mal ein paar Stunden bei stop and go im Stau - dann sehnst du dich auch nach einem Automatikgetriebe. --Snevern 19:40, 24. Mär. 2012 (CET)
Man gibt beim Automatikgetriebe die gesamte Traktionskontrolle des Fahrzeuges an das Getriebe ab. Dadurch verliert man auch einen Teil der Möglichkeit, vorausschauend zu fahren. Das Automatikgetriebe kann nicht wissen, dass da in 50m Entfernung eine Kurve kommt und nun runterschalten gut wäre. Die einzige Kommunikationsmöglichkeit des Fahrers mit dem Getriebesteuergerät sind Gaspedal und Bremsen. Wobei beides nur indirekt ist und es sehr auf den Aufwand ankommt, den der Getriebehersteller in die Elektronik, vor allem in deren Einstellungen, gesteckt hat, ob man damit eine ähnlich gute Kontrolle über das Fahrzeug bekommt wie mit Handschaltung. Das ist auch der Grund, warum fast alle Automatikgetriebe heutzutage auch mit manueller Gangwahl (Tiptronic) angeboten werden. Geschmackssache halt. -- Janka (Diskussion) 21:59, 24. Mär. 2012 (CET)
Auf Youtube gibt es hunderte Videos von Autounfällen mit sehr niedriger Geschwindigkeit, die nur passiert sind, weil die Amis mit ihren Automatik-Autos nicht umgehen können. Da rollen sie über Brückengeländer, fahren Parkgaragenschranken nieder, stürzen in Swimmingpools, fahren rückwärts gegen Garagentore, verwechseln Gas und Bremse und crashen gegen Tankstellen-Zapfsäulen, etc. Alle Unfälle wo man sieht, wie das Auto wo ankracht und gleich nochmal ankracht und nochmal, sind typische Automatikunfälle. Hier ein Clip, wo mindestens bei der Hälfte mangelnde Fähigkeit mit Automatikgetriebe umzugehen die Ursache ist. --El bes (Diskussion) 01:41, 25. Mär. 2012 (CET)
Wobei es auf der anderen Seite genauso Unfälle geben dürfte, die entstanden sind, weil jemand mit dem Schalten überfordert war, der Gang nicht rein ging oder beim Pedalwechsel mit den Füßen durcheinander kam. Von unzähligen Abwürgern an der Ampel oder missglückten Überholmanövern im fünften Gang mal abgesehen. --YMS (Diskussion) 21:10, 25. Mär. 2012 (CEST)
Ich würde nie auf meine Handschaltung verzichten wollen.
Einfache Situation: Du hängst auf der Landstraße hinter einem LKW, und wartest darauf den überholen zu können.
Handschaltung: Du schaltest runter, um im rechten Moment genügend Drehzahl zu haben um möglichst schnell zu beschleunigen und vorbei zu ziehen.
Automatikschaltung: Gemütlich im höchsten Gang. Dann scherst du aus und latschst drauf, und das auto kommt nicht so schnell aus dem Knick, weil es erst dann irgenwann wenn du schon ausgeschert hast und drauflatschst merk, dass es vllt mal runterschalten müsste.
Außerdem: Wenn man nicht mal mehr Schalten muss, wasbleibt einem dann noch vom Fahren? Dann ist der Spaß ja ganz weg. Außerdem bestimm ich lieber selbst über das Verhalten meines Autos und lasse nicht bestimmen. Denn: Ich sehe was die nächsten hundert Meter vor mir auf der Straße passiert und kann vorausschauend schalten. Die Automatik kann stets erst dann reagieren, wenn die Situation eingetreten ist. -- 208.48.242.106 11:51, 26. Mär. 2012 (CEST)
Autsch. "Wenn man nicht mal mehr Schalten muss, was bleibt einem dann noch vom Fahren? Dann ist der Spaß ja ganz weg. ": Du fährst ganz offensichtlich die falschen Autos. Wenn du einen Sportwagen mit ausreichend Leistung unter der Haube fährst, dazu mit einer guten Automatik (die hat so ein Auto in der Regel ohnehin - nur Anfänger fahren sowas mit Schaltgetriebe), dann macht das definitiv Spaß, und zwar so richtig und ohne Schalterei. Als ich vor Jahrzehnten zum ersten Mal selbst ein Mercedes Coupé fahren durfte, war das für mich ein ähnlich denkwürdiges Erlebnis wie mein erster Start mit einem Passagierflugzeug.
Ja, Schaltgetriebe fahren macht Spaß. Aber Sportwagen fahren macht auch ohne Schaltgetriebe Spaß. --Snevern 13:45, 26. Mär. 2012 (CEST)
Wer einen Sportwagen mit Automatik fährt ist ein Poser. Aber das sind ja ohnehin ~100% der Sportwagenfahrer. -- Janka (Diskussion) 14:18, 26. Mär. 2012 (CEST)
Die von dir als "Poser" beschimpften Sportwagenfahrer ruinieren sich jedenfalls nicht das Getriebe ihres Jaguars durch Rumgemurkse mit pseudo-sportlichem Fahren. Und, seien wir mal ehrlich: Solche Äußerungen setzen dich dem Verdacht des Sozialneids aus.
Ich selbst fahre übrigens keinen Sportwagen (hey - ich habe Hunde und ein kleines Kind!), sondern ein Fahrzeug mit Schaltgetriebe. Im übrigen kann ich das Posting der IP 82.207.169.66 direkt unterhalb bestätigen (ausgenommen die technischen Feinheiten, von denen ich nichts verstehe). --Snevern 16:19, 26. Mär. 2012 (CEST)
Wenn ich es mir nicht leisten kann, das Getriebe meines Jaguars durch wirklich sportliche Fahrweise zu ruinieren, sollte ich mir einen solchen nicht kaufen. Es ist der Sinn und Zweck sportlichen Fahrens, das Fahrzeug bis zur Grenze auszuloten, und da gehört die Verschleißgrenze eindeutig dazu. Der Motor eines Formel-1-Boliden ist nach dem Rennen genauso schrottreif wie das Fahrwerk eines Ralley-Touareg. Wenn man das Material zu sehr schont, kommt man nicht auf die vorderen Plätze. Wer das nicht will, fährt tatsächlich "Pseudo-Sportlich" und braucht daher auch keinen Sportwagen. So wie eben ~100% aller Sportwagenfahrer.
Und dann es gibt auch noch die Porsche-Cayenne-Fahrer, die sich beschweren, dass das Automatikgetriebe bei Vollgas erst im roten Bereich hochschaltet, aus Unkenntnis, dass dieser Bereich sehr wohl zulässig ist, nur eben höchstens einige Sekunden. Für diese Deppen haben wir dann das Drehzahlsignal zum Multiinstrument so zurechtgefaked, dass es im oberen Bereich nur 95% der tatsächlichen Drehzahl anzeigt. Daran muss ich jetzt immer denken, wenn ein Cayenne an mir vorbeifährt.
"Sozialneid": Da hab ich keine Hemmungen, mich dem "Verdacht eines solchen" auszusetzen. Warum sollte ich auf Leute hören, die ein Produkt kaufen, das sie nicht brauchen, um andere Leute zu beeindrucken, die sie eigentlich nicht ausstehen können? -- Janka (Diskussion) 01:17, 27. Mär. 2012 (CEST)
Wer gerne sportlich auf der Straße unterwegs ist (auf der Straße - nicht im Gelände! Der ganze SUV-Mist und die Geländewagenflut, egal mit wie viel PS, ist eine Seuche, die hoffentlich bald ausgerottet wird) und es sich leisten kann, kauft gerne ein Auto mit ein paar PS mehr. Es macht einen erheblichen Unterschied, ob man bei 130 auf der Autobahn schon am Limit fährt oder noch Reserven hat - auch wenn man sein Auto nicht bis an die Verschleißgrenze prügeln will (zum Beispiel, weil man nicht nach jeder Fahrt ein neues hingestellt bekommt, und weil es im Straßenverkehr in Wahrheit gar keine "vorderen Plätze" zu erringen gibt). Mit einem Formel-1- oder einem anderen Motorsport-Wagen hat das nichts zu tun - gar nichts. Obwohl...: Die "Poser" in der Formel-1 fahren auch Automatikgetriebe... Da muss doch irgendwo ein Denkfehler in deinen Überlegungen sein - nur wo!?
Wer nicht fahren kann, egal ob mit Getriebe oder Automatik, sollte es besser ganz lassen. --Snevern 10:47, 27. Mär. 2012 (CEST)
In der Formel 1 wird üblicherweise ein Doppelkupplungsgetriebe mit reiner Tiptronic benutzt. Von Hand gekuppelt wird nicht mehr. Eine Gangwahlautomatik würde da aus den eingangs von mir genannten Gründen (fehlendes vorausschauendes Schalten) keiner haben wollen. -- Janka (Diskussion) 14:18, 27. Mär. 2012 (CEST)

Ich habe den Eindruck, hier haben einige Schaltgetriebenutzer vor zwanzig Jahren das letzte mal einen Automatikwagen ausprobiert. Moderne "Automatiken" haben nichts mehr mit den alten Wandlerautomatiken des letzten Jahrtausends zu tun. Mein Auto besitzt ein DSG Doppelkupplungsgetriebe (streng genommen keine Automatik, daher das Wort vorhin in Anführungsstrichen) mit Schaltwippen am Lenkrad und ich kann jederzeit, auch im Automatikmodus, für das Überholen, für die Motorbremse oder für das extreme Beschleunigen in Sekundenbruchteilen die Gänge wechseln. Einen Gangwechsel in 0.1 Sekunden hat bislang jedenfalls noch kein Schalterfahrer hinbekommen. Im Halbschlaf kann ich aber auch alles alles der Automatik (normal und Sportmodus) überlassen. Dazu gibt es sogar auch noch eine Launch Control für die sportlichere Anfahrt von der Ampel. Und der Wagen hat dabei einen erheblich geringeren Verbrauch als die Schalterversion (knapp 1 Liter im Normzyklus). Ich sehe fast nur Vorteile. 82.207.169.66 15:55, 26. Mär. 2012 (CEST)

DK-Getriebe gibt es aber nur für sehr wenige Fahrzeuge. Das übliche sind weiterhin Drehmomentwandler+Simpsonsatz. Tiptronic ist hingegen inzwischen Standard. Mir erschließt sich allerdings nicht ganz der Sinn, ein Automatikgetriebe zu kaufen und dann doch von Hand zu schalten. Naja, wer's mag. -- Janka (Diskussion) 01:21, 27. Mär. 2012 (CEST)
Also wir haben im Haushalt 2 PKW. Ich fahr nen 1.2er TSI (105PS bei 1000kg) und der andere ist ein 1.4er TSI (122PS bei 1400kg). Meiner hat 5-Gang Handschaltgetriebe, der andere 7-Gang DSG-Getriebe (allerdings ohne Schaltwippen). Die Schaltzeiten vom DSG sind genial, sehr sparsam ist er auch. Ab und zu feht mir da aber das vorausschauende Schalten, was ich mit meinem halt machen kann. Bis auf dass mir bei meinem ein 6. Gang fehlt, fahr ich lieber mit meinem, und werde auch von Zeit zu Zeit um meine Handschaltung beneidet. Letztendlich ist es bei den Modernen DSG-Getrieben aber auch nur gewohnheit und vorliebe, sehr gut fahren kann man mit beidem. Aber ich sehe nicht ein, warum ich so viel mehr für das DSG-Getriebe ausgeben soll, wenn sich dadurch für mich kein wirklicher Vorteil ergibt. Und irgendwie gehört das Schalten auch zum Autofahren dazu. Ich schalte gerne. -- 208.48.242.106 11:20, 27. Mär. 2012 (CEST)

Antwort: Automatik schluckt auf jeden Fall deutlich mehr. Auch Tempomat ist kostenintensiver. Ein Grund für Standard ist das feinere Fahrgefühl, aber auch dass man das KFZ besser unter Kontrolle hat, also Fahrsicherheit in komplexen Situationen. Ein Nachteil bei Standard ist der deutlich höhere Aufwand beim Erlernen. Ein Fahrschüler muss ja nicht nur die Verkehrsregeln verinnerlichen, sondern auch noch auf die Signale des Autos achten. Im Laufe der Fahrpraxis automatisiert sich das aber beim Standard-Fahrer. Bestimmte Manöver sind allerdings bei Automatik schwerer zu erlernen als für jemanden, der bei Standard schon erfahren ist. Der Lernaufwand wird also bei Automatik zeitlich nach hinten verschoben im Vergleich zum Standard-Fahrer, der zwar zunächst viel lernen muss, aber später seine Karre schon so beherrscht, wie es der Automatikfahrer erst nachträglich lernt. Dazu kommt, dass ein Standardfahrer die Geschwindigkeit intuitiv durch die Gänge mitgeteilt bekommt. Auch die professionelle Nutzung des Drehzahlenmessers, die einen guten Standard-Fahrer ausmacht, ist bei Automatik kaum möglich. Gerade der Drehzahlenmesser ist aber das Instrument, das richtig satt Sprit spart und es ermöglicht guten Standard-Fahrern auch den Verzicht auf das Kuppeln. (Kuppelloses Schalten durch Drehzahlangleichung, eine eher hohe Schule des Fahrens) Das erspart einen Wechsel der Kuppelscheibe bei Zweitwagen, die bekanntlich teuer ist. Das realisiert man nur mit einem Standardgetriebe. Insgesamt betrachtet sind die Möglichkeiten hier viel höher, aber auch der Lernaufwand ist intensiver. Fahren ist mehr als nur gasgeben und bremsen. Ökonomische Fahrweise kann enorm Geld sparen, auch zusätzlich, wenn sie mit weiteren bewussen Gewohnheiten kombiniert wird: geeignete Reifen, Reifendruck, Brems- und Beschleunigungsverhalten, Ausrollenlassen im Stadtverkehr, angepasste Fahrdynamik, verminderte Ballastzuladung, Ausbau von Rückbänken bei Fahrzeugen von Singles bringen oft 100 kg, Verzicht auf Ersatzräder im Stadtverkehr 50 kg, untertouriges Schalten beim Standardgetriebe. Da ist ordentlich was drin.

Ein Vorteil der Automatik besteht unter sehr extremen Stress-Situationen, da dann Automatikfahrer einen Vorteil haben: Stichwort "Verlust erlernter Verhaltensweisen" oder "Rückfall in naive Verhaltensmuster bei Gefahr", weshalb auch Panzer und militärische Fahrzeuge heute meist Automatik haben. Hier ist das Feingefühl im Gelände zwar herab gesetzt, in schweren Gefechten und großer Gefahr aber sind die Fahrer dann etwas entlastet und müssen nur noch beschleunigen und bremsen. Also einfach nur "abhauen" oder auch bei leistungsstarkem Aggregat, wenn Zähne im Getriebe ausbrechen könnten oder auch bei Antrieb mit einer Turbine, die mehrere Vorschaltungen aufweist. Bei bestimmten landwirtschaftlichen Nutzfahrzeugen oder bei LKW.

Die richtige Antwort lautet: Es kommt auf den Verwendungszweck an und gute Fahrer können beide Formen beherrschen. Je nach Verwendung und Spar-Absichten sind Standard, Automatik, Tempomat oder Drehzahlenmesser die Mittel der Wahl. :-)

Ein wirklich guter Fahrer setzt sich bei jedem KFZ - vom Kleinstwagen bis zum Sattelschlepper - selbstbewusst ans Steuer und bringt des je nach Möglichkeit sparsam, sicher und souverän von A nach B. 2.211.116.166 20:49, 27. Mär. 2012 (CEST)

"... ermöglicht guten Standard-Fahrern auch den Verzicht auf das Kuppeln. (Kuppelloses Schalten durch Drehzahlangleichung, eine eher hohe Schule des Fahrens) Das erspart einen Wechsel der Kuppelscheibe bei Zweitwagen, die bekanntlich teuer ist."
Wow. Ich habe noch das Schalten mit Zwischengas gelernt (unsynchronisiertes Getriebe, also Drehzahlangleichung durch Zwischengasgeben). Aber das man bei Zweitwagen die teure Kuppelscheibe sparen kann, indem man auf das Kuppeln verzichtet... wow. Wieder was gelernt. --Snevern 23:14, 27. Mär. 2012 (CEST)
Im Ernst, das geht! Wenn beim rollenden Wagen kein Kraftfluss besteht kuppelt man aus, indem man den Kupplungshebel sehr leicht unter Spannung hält und das Gas abflauen lässt. Geht an einem Punkt butterweich. Die Drehzahl pro aktuelle Geschwindigkeit hat man mit der Zeit im Gefühl. Einkuppeln lernt man, indem man beim rollenden Auto den Gang "anspricht" (betätigt bis kurz bevor sich die Zahnräder im Getriebe berühren) und die Drehzahl angleicht. Da die Drehzahl aber dynamischer als die Fahrgeschwindigkeit ist, lässt man die Drehzahl langsam von oben herunter kommen und wartet bis man auf dem Wert ist, der bei der aktuellen Geschwindigkeit synchron ist. Dann Einkuppeln, fertig! Das zu lösende Problem: Drehzahl und Geschwindigkeit sind beides Variablen, die pro Gang jeweils eine Funktion im Sinne einer Kurve bilden. Mir gelingt nach jahrelanger Routine das Einkuppeln in allen Gängen gut, auch bei fremden Autos, denn der Vorgang ist auch vom Fahrzeugtyp abhängig. Wer das lernen will, bekommt gerade beim einkuppeln immer mal einen sogenannten "Gruß vom Getriebe", der tunlichst zu vermeiden ist. Man muss also ganz ordentlich Feingefühl entwickeln, das man aber später einfach intuitiv drin hat. Ergebnis: Damit braucht man die Kupplung nur noch zum Anfahren, Einparken und natürlich in Gefahrensituationen, aber nicht mehr im Routinebetrieb. Ärger mit Kupplungswechsel gehören komplett der Vergangenheit an. Auch bei völlig fertiger Kupplung kann man den Wagen noch jahrelang betreiben. Das Problem gibts dann eben einfach nicht mehr. :-) Ich bin auf diese Form aufmerksam geworden, als mir Anfang der Neunziger jemand einen alten Citroen BX mit frischem TÜV angedreht hat, der aber nur noch schlecht ausgekuppeln konnte. Damals habe ich die Seile sehr fest nachgespannt und die Kupplung auf diese Weise geschont. (Damals war ich Student und brauchte die Kohle für anderen Kram.) Seither mache ich das nur noch so. Gibts niemanden, der das auch so macht? Hinweis: Ich fahre seit 27 Jahren unfallfrei auf Krad, LKW, PKW, Panzer und Nutzfahrzeugen. Die Fleppen habe ich bei der GST und der Asche gemacht.46.115.42.125 19:49, 28. Mär. 2012 (CEST)
Ich fahre auch so, schon seit Motoradzeiten (da geht das auch), und jede Menge anderer Leute auch, wie ich gerade über Google gelernt habe. Ich hatte anfangs immer ein schlechtes Gewissen wegen Abrieb oder was auch immer, es hat den Kupplungen aber nie geschadet, neulich erst bestätigte ein Mechaniker mir auch die Unbedenklichkeit. Beim Tempomaten allerdings bilde ich mir ein, dass es sich damit sparsamer fährt (und von Automatikgetrieben höre ich seit einigen Jahren, dass es dort inzwischen auch so sein soll). Die Beschleunigung ist zwar lahmer, allerdings zeigt mein Multidingensinstrument nie exorbitante Sachen an wie beim beschleunigen mittels Gaspedal (wo gerne, gerade beim Anfahren, gerne mal 50l/100km angezeigt werden... jaja, ist nur für den Moment, aber beim Tempomaten macht er das nie). Tempomat ist übrigens auch eine feine Sache in Tempo-30-Zonen. Ein Klick und man kann auf die Straße und spielende Kinder achten, statt in Angst vor dem nächsten Blitzer ständig auf’s Tacho schauen zu müssen.
Und Snevern stimme ich mit der Automatik in Sportwagen zu. Ich hatte einmal das Vergnügen, für zwei Tage einen Porsche Carrera fahren zu dürfen. Ich fand es zwar uncool, dass wirklich alles elektronisch geregelt wurde (war aber auch abschaltbar, soweit ich erinnere), war andererseits aber sehr froh, mir über durchdrehende Räder etc keine Gedanken machen zu müssen, sondern einfach nur heizen zu können. -- Ian Dury Hit me  10:23, 29. Mär. 2012 (CEST)
Abrieb bei kuppellosem Schalten ist kein Problem und auch keine Gefahr. Ein Problem ist m.E. nur das Selbstvertrauen des Jeweiligen. Man muss es sich halt zutrauen ohne Kupplung zu fahren. Gib jedem Neuling zwei dutzend Getriebegrüße, danach hat er es drin. Klingt auch häßlich und sehr aversiv, da lernt man es ohnehin schneller.
Tempomat ist m.W. in jedem Fall kostenintensiver. Ich habs mal mit einem Mazda 626 auf der Autobahn durchgerechnet. Da bin ich sechs Wochen jeden Tag zweimal ziemlich genau 100 km Nurautobahn gefahren, das waren 2 Liter mehr im Standardbetrieb. Also überhaupt nicht gut. Das sind verbürgbare Erfahrungswerte. Wie es in der Stadt ist, weiß ich nicht. denk mal, eher noch schlechter, oder?. Aber wie Dury hinweist ist der Sicherheitsaspekt in jedem Falle wichtiger. Ich nehme trotzdem den Tempomaten nur auf Langstrecke in der Ebene, wenn überhaupt, und schleiche in der 30er Zone. Tempomat ist einfach teurer und mir persönlich ist schlichtweg der Sprit zu schade; bin persönlich eher auf Sparen aus. Weiß nicht wie es Euch geht?
Ohne Tempomat in der Stadt (Berliner Berufsverkehr) rechne man so per Daumenregel durch Ausbau Rückbank und Reserverad mit 1,2 Litern runter. (Twingo 99 für Parklückenfreaks) Manchmal auch mehr. Das merkste richtig an der Tanksäule. Bleibt einfach mehr Geld für schöne Dinge übrig und mal ehrlich, hat schon mal jemand das Ersatzrad gebraucht? Gibt natürlich auch ordentlich Stauraum frei. 46.115.37.254 19:11, 29. Mär. 2012 (CEST)
Mmmh... das würde bedeuten, dass wenn ich ohne Tempomat fahre, statt 9 nur 7l/100km verbrauchen würde? Und das im Stadtverkehr? Kann ich mir nicht vorstellen, bei einer ’nem Van, zumal ich mit den 9 Litern schon unter den Herstellerangaben liege. -- Ian Dury Hit me  10:13, 30. Mär. 2012 (CEST)
Hmja, das habe ich auch nicht geschrieben. Ich habe auf der Autobahn auf 100 km mit dem 626 mit Tempomat 2 Liter mehr verbraucht, das kann ich ganz sicher verbürgen. Stadtverkehr weiß ich jedoch nicht. Da kann man es nicht so gut abschätzen, unter anderem weil man den Tempomat nicht 100 km am Stück anhaben kann und weil da der Sprit ja ins Anhalten/Anfahren geht. Ist natürlich bei jedem Auto anders, aber die Tendenz ist klar: Atomatische Systeme sind teurer. Und, um auf die Ausgangsfrage zurück zu kommen, Automatik ist eben auch teurer. Menschliche Fahrerfahrung kann eben nicht mit so einfachen Methoden ersetzt werden, obwohl ich vermute, dass es vielleicht auch computergestützte Systeme geben kann, die sogar noch mehr sparen als gute Fahrer. Aber ich kenne die nicht und ich würde sie auch nicht als Neuwagen kaufen, weil sie garantiert teurer als die Einsparung wären. Das ist genau wie bei 3-Liter-Autos oder LED-Birnen. Klar brauchen LEDs nur 4 Watt, aber sie kosten auch 15 Euro, wo die Leuchtstoffbirnen von OBI nur 1,99 kommen bei 9 Watt. Man soll eben seinen eigenen Rechenfähigkeiten mehr trauen als den mitgelieferten Rehenbeispielen der Hersteller. Im Fall des Automatikautos oder bei Tampomat ist es eben so, dass er mehr frisst. Aber da sage ich halt nur: Ausprobieren! Selbst an der Tanke Notizen machen und nicht nur einfach rein gluckern lassen und hinterher drüber schimpfen. 2.212.79.168 17:02, 30. Mär. 2012 (CEST)
"Wie es in der Stadt ist, weiß ich nicht. denk mal, eher noch schlechter, oder?" Ich finde im Netz nur Aussagen, die meine Vermutung bestätigen, vorausgesetzt, du schaltest den Tempomaten bergauf ab. Wenn du mit "steifen Fuß" fährst, hast du ja auch einen gleichbleibenden Verbrauch und nimmst in Kauf, bergauf langsamer zu werden. Mein Mechaniker erklärte mir übrigens auch, dass kaum ein Auto ohne den ganzen Elektronik-Schnickschnack überhaupt noch effizient fahren würde. Der Kram wird, wie er sagte, eingebaut, damit "hinten nur noch bunte Blümchen rauskommen". Tempomat kostet übrigens nur etwa 50 € im Teileverkauf, bei einem Neuwagen ist die Anschaffung also fast zu vernachlässigen, Wert steigernd ist es nicht unbedingt, wenn der Gebrauchte einen hat. Bei den LEDs wäre interessant wie die Bilanz bzgl. der Herstellung im Vergleich zu den anderen Leuchten aussieht. Der Preis spielt dann – in Anbetracht knapper werdender Ressourcen – keine Rolle, wenn der Unterschied im Energieaufwand sich rechnet, unterschiedlicher Haltbarkeiten etc. natürlich mit berücksichtigt. Ich meine, da sind LEDs am Ende günstiger, nicht unbedingt für den eigenen Geldbeutel, aber die Preise bewegen sich da ja auch schon nach unten. -- Ian Dury Hit me  19:57, 30. Mär. 2012 (CEST)
Die chinesischen LED sind deutlich einfacher herzustellen als alle anderen Leuchtmittel. Es handelt sich um einen Lack, der gleißend grell leuchtet, wenn er von Strom durchflossen wird, sich dabei nicht erwärmt und nie kaputt geht. Er wird auf Dioden getropft, die dann in Kinderarbeit oder von Schuldensklaven in Fassungen gebaut werden. 1cm² diesen Lacks leuchtet mit 36 Watt so grell wie eine 1000 Watt-Lampe. Man könnte auch eine ganze Wand damit streichen, wenn das nicht zu hell wäre. Aber der Vorteil wird nicht an den Verbraucher weiter gegeben. Im Gegenteil: (1) die kommen so teuer auf, dass es ein deutlicher Geldverlust ist, sie anzuschaffen (2) der Preis sinkt langsam auf genau das selbe Niveau der älteren Technologie (3) die Ressourcekosten (Strom) steigen um den Effektivitätsgewinn weg zu fressen. Man gibt zu jedem Zeitpunkt etwa gleich viel Geld für Beleuchtung aus, vorausgesetzt man kauft die Lampen nicht zu früh, dann gibt man mehr Geld aus..
Es geht mir nicht um die Umwelt oder den gesamtgesellschaftlichen Nutzen oder sowas. Von mir aus können die auch Atommüll über Flüsse entsorgen oder Affenköpfe auf Babys transplantieren. Ich will einfach nur kein Sinnlosgeld für andere bezahlen, das ist alles. So wie ich auch nicht sinnlos meine Lebenszeit für andere verarbeiten will. Und das ist auch der Grund, weshalb ich niemals ein Automatik-Auto anschaffen werde. Es ist sinnlos teurer. 2.212.79.168 08:26, 31. Mär. 2012 (CEST)